c. motor recovery pictures removed - ebrsrpr).pdf · - 3 - c1. assessment of motor function post...

112
- 1 - 1 C. Motor Recovery Post Stroke Educational Supplement Robert Teasell MD FRCPC, Andrew McClure, Manuel Murie-Fernandez MD C1. Assessment of Motor Function Post Stroke…………………………………………………… 2 C2. Prognosis of Motor Recovery……………………………………………………………...…... 10 C3. Gait Retraining Post Stroke…………………………………………………………………….. 15 C4. Balance Retraining Post Stroke…………………………………………………………..……. 21 C5. The Bobath Approach / Neurodevelopmental Technique of Motor Therapy……………... 25 C6. Task-Specific Therapy………………………………………………………………………….. 33 C7. Treadmill Training and Partial Weight Support………………………………………………. 37 C8. Functional Electrical Stimulation of Lower Extremity………………………………......…… 46 C9. Rehabilitation of the Upper Extremity……………………………………………………….… 50 C10. Constraint-Induced Movement Therapy…………………………………………………........ 54 C11. Mental Practice Post Stroke……………………………………………………………………. 63 C12. Spasticity Post Stroke…………………………………………………………………………... 67 C13. Hemiplegic Shoulder Pain…………………………………………………………………….... 78 C14. Case Study: Post-Stroke Motor Recovery……………………………………………………. 87 C15. Case Study: Lower Extremity and Mobility………………………………………………….... 92 C16. Case Study: Upper Extremity……………………………………………………………......97 112 pages

Upload: haduong

Post on 18-Nov-2018

215 views

Category:

Documents


0 download

TRANSCRIPT

Page 1: C. Motor Recovery pictures removed - EBRSRPR).pdf · - 3 - C1. Assessment of Motor Function Post Stroke C1.1 Assessments of Motor Function Post Stroke Case Study A 65 year old male

- 1 - 1

C. Motor Recovery Post Stroke Educational Supplement Robert Teasell MD FRCPC, Andrew McClure, Manuel Murie-Fernandez MD C1. Assessment of Motor Function Post Stroke…………………………………………………… 2 C2. Prognosis of Motor Recovery……………………………………………………………...…... 10 C3. Gait Retraining Post Stroke…………………………………………………………………….. 15 C4. Balance Retraining Post Stroke…………………………………………………………..……. 21 C5. The Bobath Approach / Neurodevelopmental Technique of Motor Therapy……………... 25 C6. Task-Specific Therapy………………………………………………………………………….. 33 C7. Treadmill Training and Partial Weight Support………………………………………………. 37 C8. Functional Electrical Stimulation of Lower Extremity………………………………......…… 46 C9. Rehabilitation of the Upper Extremity……………………………………………………….… 50 C10. Constraint-Induced Movement Therapy…………………………………………………........ 54

C11. Mental Practice Post Stroke……………………………………………………………………. 63 C12. Spasticity Post Stroke…………………………………………………………………………... 67

C13. Hemiplegic Shoulder Pain…………………………………………………………………….... 78 C14. Case Study: Post-Stroke Motor Recovery……………………………………………………. 87 C15. Case Study: Lower Extremity and Mobility………………………………………………….... 92 C16. Case Study: Upper Extremity……………………………………………………………......… 97 112 pages

Page 2: C. Motor Recovery pictures removed - EBRSRPR).pdf · - 3 - C1. Assessment of Motor Function Post Stroke C1.1 Assessments of Motor Function Post Stroke Case Study A 65 year old male

- 2 - 2

C1. Assessment of Motor Function Post Stroke

Page 3: C. Motor Recovery pictures removed - EBRSRPR).pdf · - 3 - C1. Assessment of Motor Function Post Stroke C1.1 Assessments of Motor Function Post Stroke Case Study A 65 year old male

- 3 - 3

C1. Assessment of Motor Function Post Stroke C1.1 Assessments of Motor Function Post Stroke Case Study A 65 year old male suffers a left hemispheric stroke and presents with a right hemiplegia. At the time of his admission to rehabilitation 2 weeks later he still presents with weakness on his right side, he is a two person pivot transfer with returning tone. Q1. Name some commonly used assessments of motor function post stroke. Answers Some commonly used assessments of motor function post stroke include: 1. The Berg Balance Score 2. Chedoke-McMaster Stages of Motor Impairment 3. Clinical Outcomes Variable Scale (COVS) C1.2 The Berg Balance Score Q2. Describe the Berg Balance Score including its strengths and limitations. Answers 1. Assessment of balance. 2. 14 items from 0-4 per item for maximum score of 56. 3. Score < 45 is at risk of falling. 4. Measure static and dynamic balance – little equipment, space or training needed. 5. Decreased sensitivity early on among severe stroke patients. Discussion Questions Answer What does it measure?

Quantitative assessment of balance in older adults (Berg et al. 1989).

What is the scale? 14 items requiring subjects to maintain positions or complete movement tasks of varying levels of difficulty. Items receive a score of 0-4 based on ability to meet the specific time and distance requirements of the test. 0 = inability to complete the item; 4 = ability to complete the test independently.

Page 4: C. Motor Recovery pictures removed - EBRSRPR).pdf · - 3 - C1. Assessment of Motor Function Post Stroke C1.1 Assessments of Motor Function Post Stroke Case Study A 65 year old male

- 4 - 4

What are the key scores?

Maximum score = 56. A score of less than 45 is indicative of balance impairment (Berg et al.1992a, Zwick et al. 2000).

What are its strengths?

Measures a number of different aspects of balance, both static and dynamic (Whitney et al. 1998). Requires little equipment or space and no specialized training. High levels of reliability even when test is administered by an untrained assessor. Particularly well suited to acute stroke rehabilitation, as the majority of patients do not obtain maximum scores on admission to rehabilitation (Wee et al. 1999).

What are its limitations?

Takes somewhat longer to administer than other balance measures (Whitney et al. 1998) and may not be suitable for the evaluation of active, elderly persons, as the item included are not sufficiently challenging for this group (Berg et al. 1989, Zwick et al. 2000). As no common standards for interpretation of BBS scores exist, their relationship to mobility status and the requirement for mobility aides (Wee et al. 2003). Decreased sensitivity in early stage post-stroke among severely affected patients as scale includes only one item relating to balance in the sitting position (Mao et al. 2002).

Summary Reliability Validity Responsiveness Rigor Results Rigor Results Rigor Results Floor/Ceiling ++ +++ (TR)

+++ (IO) +++ (IC)

+++ +++ +++ +++ varied

NOTE: +++=Excellent; ++=Adequate; +=Poor; n/a = insufficient information; TR = Test re-test; IC = Internal consistency; IO = Interobserver; varied (re floor/ceiling effects; mixed results) Q3. Describe the Benefits of the Berg Balance Score in Terms of Prognosis of Stroke? Answers 1. On the positive side, not many stroke patients have received maximum scores on admission

to rehabilitation. 2. Often correlates well with functional mobility gains on rehabilitation. 3. On the negative site, there is decreased sensitivity in the early stages of stroke for severely

affected patients as there is only one item related to balance in the sitting position. Discussion Wee et al. (1999) have suggested that the Berg Balance Scale may be particularly well suited for use in acute stroke rehabilitation, as the majority of patients do not obtain maximum scores on admission to rehabilitation. However, the BBS may suffer from decreased sensitivity in the early stages of stroke among severely affected patients as the scale includes only one item relating to balance in the sitting position.

Page 5: C. Motor Recovery pictures removed - EBRSRPR).pdf · - 3 - C1. Assessment of Motor Function Post Stroke C1.1 Assessments of Motor Function Post Stroke Case Study A 65 year old male

- 5 - 5

Case Study (continued) The 65 year old male who suffered a left hemispheric stroke responds well to rehabilitation and after extensive rehabilitation he is up walking with a cane. His Berg Balance score is 42/56. Q4. What are the Implications of a Berg Balance Score of 42/56 with Regard to Risk of Falling? What Level of Assistance Should He Require? Answer 1. Berg Balance score less than 45 means the patient is at significant risk of falling. 2. He will likely only require supervision and his cane while walking or transfering. C1.3 The Chedoke-McMaster Stages of Motor Impairment Q5. Describe the Chedoke-McMaster Stages of Motor Impairment. Answer 1. The Chedoke-McMaster Stroke Assessment Scale (CMSA) physical impairment has 6

dimensions: shoulder pain, postural control, arm, hand, leg and foot movements. 2. Each dimension is rated on a 7-point scale corresponding to Brunnstrom’s 7 stages of motor

recovery (where 1 = flaccid paralysis & 7 = normal). 3. The CMSA is primarily an assessment of mobility. 4. CMSA also has a disability inventory which consists of a gross motor index (10 items) and a

walking index (5 items). 5. With the exception of the 2-minute walk test, items are scored according to the same 7 point

scale as the FIM (1 = total assistance, 7 = total independence). Discussion Questions Answer What does it measure?

The Chedoke-McMaster Stroke Assessment Scale (CMSA) is a 2-part assessment consisting of a physical impairment inventory and a disability inventory. The impairment inventory is intended to classify patients according to stage of motor recovery while the disability inventory assesses change in physical function (Gowland et al. 1993).

What is the scale? The scale’s impairment inventory has 6 dimensions; shoulder pain, postural control, arm movements, hand movements, leg movements, and foot movements. Each dimension (with the exception of ‘shoulder pain’) is rated on a 7- point scale corresponding to Brunnstrom’s 7

Page 6: C. Motor Recovery pictures removed - EBRSRPR).pdf · - 3 - C1. Assessment of Motor Function Post Stroke C1.1 Assessments of Motor Function Post Stroke Case Study A 65 year old male

- 6 - 6

stages of motor recovery. The disability inventory consists of a gross motor index (10 items) and a walking index (5 items). With the exception of a 2-minute walking test (which is scored as either 0 or 2), items are scored according to the same 7-point scale where 1 represents total assistance and 7 represents total independence.

What are the key scores?

The impairment inventory yields a total score out of 42 while the disability inventory yields a total score out of 100 (with 70 points from the gross motor index and 30 points from the walking index).

What are its strengths?

The use of Brunnstrom staging and FIM scoring increases the interpretability of the CMSA and may facilitate comparisons across groups of stoke patients (Gowland et al. 1993). The CMSA is relatively comprehensive and has been well studied for reliability and validity (Poole and Whitney 2001).

What are its limitations?

Taking approximately 1 hour to complete, the length and complexity of the CMSA may make the scale less useful in clinical practice (Poole and Whitney 2001). As primarily a measure of motor impairment, the CMSA should really be accompanied by a measure of functional disability such as the BI or the FIM (Poole and Whitney 2001).

Summary Reliability Validity Responsiveness Rigor Results Rigor Results Rigor Results Floor/Ceiling + +++ (TR)

+++ (IC) +++ (IC)

+ +++ + +++ n/a

NOTE: +++=Excellent; ++=Adequate; +=Poor; n/a = insufficient information; TR = Test re-test; IC = Internal consistency; IO = Interobserver; varied (re floor/ceiling effects; mixed results) Q6. Describe the 7 Brunnstrom Stages of Motor Recovery. Answer 1. Flaccid paralysis. No reflexes. 2. Some spastic tone. No voluntary movement. Synergies elicited through facilitation. 3. Spasticity is marked. Synergistic movements may be elicited voluntarily. 4. Spasticity decreases. Synergistic movements predominate. 5. Spasticity wanes. Can move out of synergies although synergies still present. 6. Coordination and movement patterns near normal. Trouble with more rapid complex

movements. 7. Normal Discussion Stages of Motor Recovery of the Chedoke McMaster Stroke Impairment Inventory (Gowland et al. 1993) Stages Characteristics

Page 7: C. Motor Recovery pictures removed - EBRSRPR).pdf · - 3 - C1. Assessment of Motor Function Post Stroke C1.1 Assessments of Motor Function Post Stroke Case Study A 65 year old male

- 7 - 7

1 Flaccid paralysis is present. Phasic stretch reflexes are absent or hypoactive. Active movement cannot be elicited reflexively with a facilitatory stimulus or volitionally.

2 Spasticity is present and is felt as a resistance to passive movement. No voluntary movement is present but a facilitatory stimulus will elicit the limb synergies reflexively. These limb synergies consist of stereotypical flexor and extensor movements.

3 Spasticity is marked. The synergistic movements can be elicited voluntarily but are not obligatory.

4 Spasticity decreases. Synergy patterns can be reversed if movement takes place in the weaker synergy first. Movement combining antagonistic synergies can be performed when the prime movers are the strong components of the synergy.

5

Spasticity wanes, but is evident with rapid movement and at the extremes of range. Synergy patterns can be revised even if the movement takes place in the strongest synergy first. Movements that utilize the weak components of both synergies acting as prime movers can be performed.

6

Coordination and patterns of movement can be near normal. Spasticity as demonstrated as resistance to passive movement is no longer present. Abnormal patterns of movement with faulty timing emerge when rapid or complex actions are requested.

7

Normal. A “normal” variety of rapid, age appropriate complex movement patterns are possible with normal timing, coordination, strength and endurance. There is no evidence of functional impairment compared with the normal side. There is a “normal” sensory-perceptual motor system.

C1.4 Clinical Outcomes Variables Scale (COVS) Q7. Describe the COVS Assessment Tool Including Strengths and Weaknesses. 1. Designed to assess functional mobility status. 2. 13-items selected in so as to be representative of outcomes associated with a regular

physiotherapy caseload within the general rehabilitation population. 3. Each item receives a score from 1-7 with a composite score of 13-91. 4. Provides detail in areas of mobility not assessed by functional assessments such as FIM. 5. Monitors motor tasks retrained by physiotherapists and includes use of assistive devices

and ability to negotiate environmental barriers. 6. Overall, good reliability and user friendly. Discussion Questions Answer What does it measure?

Designed to assess functional mobility status in order to identify treatment goals and initiate treatment protocols

What is the scale? The 13-items comprising the COVS were selected in such a way as to be representative of outcomes associated with a regular physiotherapy caseload within the general rehabilitation population (Seaby and Torrance 1989, Finch et al. 2002). The concept of environmental barriers and the ability to negotiate within the environment is incorporated into the test items (Seaby and Torrance 1989), which

Page 8: C. Motor Recovery pictures removed - EBRSRPR).pdf · - 3 - C1. Assessment of Motor Function Post Stroke C1.1 Assessments of Motor Function Post Stroke Case Study A 65 year old male

- 8 - 8

include assessments of transfer abilities to and from bed and from the floor, and wheelchair skill (Low Choy et al. 2002). Each item or functional task has its own 7-point rating scale based on the Patient Evaluation Conference System (PECS) with 1 representing the worst possible outcome and 7 the best possible outcome (i.e. the highest amount of function). The COVS is usually administered by a trained physiotherapist and a full assessment can take 15-45 minutes to complete.

What are the key scores?

Items can be considered individually or summed to provide a composite score ranging from 13 – 91. Items can also be summed in various combinations to provide assessments of ambulation (4 items), mobility in bed (2 items), transfers (2 items) and arm function (2 items) (Seaby and Torrance 1989).

What are its strengths?

Provides detail in areas of mobility not assessed by global functional assessments such as the FIM (Barclay-Goddard 2000, Low Choy et al. 2002). Monitors motor tasks retrained by physiotherapists and includes both the use of assistive devices and the ability to negotiate environmental barriers. Overall, has demonstrated good reliability and is user friendly in that it is designed to be performed as part of a routine physiotherapy assessment (Huijbregts 1996).

What are its limitations?

Administration of the COVS requires a fairly lengthy list of equipment and a substantial amount of time. There is ongoing need for further validation of the COVS, which is relatively widely used.

Summary Reliability Validity Responsiveness Rigor Results Rigor Results Rigor Results Floor/Ceiling + +++ (TR)

+++ (IO) ++ (IC)

++ +++ + + n/a

NOTE: +++=Excellent; ++=Adequate; +=Poor; n/a = insufficient information; TR = Test re-test; IC = Internal consistency; IO = Interobserver; varied (re floor/ceiling effects; mixed results) References Barclay-Goddard R. Physical function outcome measurement in acute neurology. Physiotherapy Canada 2000;52:138-145. Berg KO, Wood-Dauphinee S, Williams JL. The Balance Scale: Reliability assessment with elderly residents and patients with acute stroke. Scan J Rehab Med 1995; 27:27-36. Berg KO, Wood-Dauphinee S, Williams JL, Maki B. Measuring balance in the elderly: validation of an instrument. Can J Public Health 1992; 83:S7-S11. Berg KO, Wood-Dauphinee S, Williams JL, Maki B. Measuring balance in the elderly: preliminary development of an instrument. Physiotherapy Canada 1989; 41:304-311.

Page 9: C. Motor Recovery pictures removed - EBRSRPR).pdf · - 3 - C1. Assessment of Motor Function Post Stroke C1.1 Assessments of Motor Function Post Stroke Case Study A 65 year old male

- 9 - 9

Finch E, Brooks D, Stratford PW, Mayo NE. Physical Rehabilitations Outcome Measures. A Guide to Enhanced Clinical Decision-Making., 2nd ed. Toronto, Ontario: Canadian Physiotherapy Association, 2002. Gowland C, Stratford PW, Ward M, et al. Measuring physical impairment and disability with the Chedoke-McMaster Stroke Assessment. Stroke 1993; 24:58-63. Huijbregts M. The physiotherapy clinical outcome variables (COVS) reliability testing videotape. Physiotherapy Canada 1996;48:285. Juneja G, Czyrny JJ, Linn RT. Admission balance and outcomes of patients admitted for acute inpatient rehabilitation. Am J Phys Med Rehabil 1998; 77:388-393. Low Choy N, Kuys S, Richards M, Isles R. Measurement of functional ability following traumatic brain injury using the Clinical Outcomes Variable Scale: A reliability study. Australian Journal of Physiotherapy 2002;48:35-39. Mao HF, Hsueh IP, Tang PF, Sheu CF, Hsieh CL. Analysis and comparison of the psychometric properties of three balance measures for stroke patients. Stroke 2002; 33:1022-1027. Poole JL, Whitney SL. Assessments of motor function post stroke: A review. Physical and Occupational Therapy in Geriatrics 2001; 19:1-22. Seaby L, Torrance G. Reliability of a physiotherapy functional assessment used in a rehabilitation setting. Physiotherapy Canada 1989;41:264-271. Wee JYM, Bagg SD, Palepu A. The Berg Balance Scale as a predictor of length of stay and discharge in an acute stroke rehabilitation setting. Arch Phys Med Rehabil 1999; 80:448-452. Wee JYM, Wong H, Palepu A. Validation of the Berg Balance Scale as a predictor of length of stay and discharge destination in stroke rehabilitation. Arch Phys Med Rehabil 2003; 84:731-735. Whitney SL, Poole JL, Cass SP. A review of balance instruments for older adults. Am J Occup Ther 1998; 52:666-671. Zwick D, Rochelle A, Choksi A, Domowicz J. Evaluation and treatment of balance in the elderly: A review of the efficacy of the Berg Balance Test and Tai Chi Quan. Neurorehabilitation 2000; 15:49-56.

Page 10: C. Motor Recovery pictures removed - EBRSRPR).pdf · - 3 - C1. Assessment of Motor Function Post Stroke C1.1 Assessments of Motor Function Post Stroke Case Study A 65 year old male

- 10 - 10

C2. Prognosis for Motor Recovery

Page 11: C. Motor Recovery pictures removed - EBRSRPR).pdf · - 3 - C1. Assessment of Motor Function Post Stroke C1.1 Assessments of Motor Function Post Stroke Case Study A 65 year old male

- 11 - 11

C2. Prognosis for Motor Recovery Q1. Describe the two most important factors which predict motor recovery and describe their role. Answers 1. Stroke severity: The most important predictive factor which reduces the capacity for brain

reorganization. 2. Age: Younger patients do better than older patients. Discussion Severity of the Stroke This is the most important prognostic factor. As a general rule, the severity of the initial deficit is inversely proportional to the prognosis for recovery. Most spontaneous recovery occurs during the first 3-6 months following the stroke. The course of recovery negatively accelerates as a function of time and is a predictable phenomenon (Skilbeck et al. 1983). Jorgensen et al. (1995a, 1995b) studied 1,197 acute stroke patients in what is referred to as the Copenhagen Stroke Study. This study consisted of a large unselected community-based population who were admitted to a 63-bed stroke unit. Impairments were classified using the Scandinavian Neurological Stroke Scale (SSS) and functional disability was defined according to the Barthel Index (BI). Typically, recovery for impairment and functional disability meant the highest recorded score in SSS and BI, respectively, with no further improvement. At the time of initial assessment, 41% of patients had mild strokes, 26% moderate and 19% severe, reflecting the severity of their neurological impairment as measured by the SSS. As a group, 95% of all patients reached their best neurological level wihin 11 weeks, on average. 95% of patients with mild strokes had reached their maximal neurological recovery within six weeks; for patients with moderate, severe and very severe strokes, 95% of the group had achieved their maximal recovery within 10, 15 and 13 weeks respectively. Neurological recovery occurred on average two weeks earlier than functional recovery. The specific timeline for neurological and functional disability recovery is presented in Tables below. In surviving patients, the best neurological recovery occurred within 4.5 weeks in 80% of the patients, while best ADL function was achieved by 6 weeks. For 95% of the patients, best neurological recovery was reached by 11 weeks and best ADL function within 12.5 weeks. Impairment and Neurological Recovery of Stroke Patients in the Copenhagen Stroke Study

Category (SSS) Admission1

(%) Discharge2

(%) Survival

(%) Weeks to 80%

Best Recovery3 Weeks to 95%

Best Recovery3 Very Severe (0-14)

19 4 38 10 13 (11.6-14.4)

Severe (15-29) 14 7 67 9 15 (13-17) Moderate (30-44)

26 11 89 5.5 10.5 (9.5-11.5)

Mild/No (45-58 41 78 97 2.5 6.5 (5.4-7.6)

Page 12: C. Motor Recovery pictures removed - EBRSRPR).pdf · - 3 - C1. Assessment of Motor Function Post Stroke C1.1 Assessments of Motor Function Post Stroke Case Study A 65 year old male

- 12 - 12

1 Percentage patient distribution on admission, grouped by stroke severity sub groups, as measured by SSS (scores range from 0-58 points). 2 Percentage distribution of survivors (79% of initial group) after completion of stroke rehabilitation. 3 Neurological recovery as measured by SSS. Disability and Outcome of Stroke Patients in the Copenhagen Stroke Study

Category (BI) Discharge1 (%)

Survival (%)

Weeks to 80% Best Recovery2

Weeks to 95% Best Recovery2

Very Severe (0-20) 14 50 11 17 (15-19) Severe (25-45) 6 92 15 16 (13.5-18.5) Moderate (50-70) 8 97 6 9 (7.5-10.5) Mild (75-95) 26 98 2.5 5(4-6) No (100) 46 1 Percentage patient distribution on admission, grouped by stroke severity sub groups, as measured by BI 2 Functional recovery as measured by BI. In another study, Jorgensen et al. (1995c) reported that the best walking function was reached within 4 weeks for patients with mild paresis of the affected lower extremity, 6 weeks for those with moderate paresis and 11 weeks for severe paralysis. Consequently, the time course of both neurological and functional recovery was strongly related to both initial stroke severity and functional disability. Jorgensen et al. (1995a-c), as noted above, two-thirds of all stroke survivors have mild to moderate strokes and are able to achieve independence in ADL Based on these observations one can safely conclude that the initial severity of the stroke is inversely proportional to the final functional outcome, with the majority of patients who demonstrate mild strokes demonstrating no or only mild disabilities, while the majority of patients suffering from very severe strokes still experience severe or very severe deficits even after the completion of rehabilitation. There is evidence that structural changes occur in the uninjured cortical tissue surrounding the stroke. Nudo (2003a) has noted that, “the mechanisms that underlie functional changes in the motor cortex of normal animals during motor skill learning are likely the same mechanisms that play a role in recovery after damage to the motor cortex.” Frost et al. (2003) have suggested two principles to explain how plasticity is related to the amount of brain damage: “1) reorganization of secondary cortical areas is a general feature of injury-induced plasticity, and 2) remote reorganization is directly related to reciprocal connectivity of the various motor areas.” The latter concept implies that there needs to be some form of connection to the damaged motor area, i.e. secondary motor areas, for any opportunity for functional reorganization to occur. Thus, larger strokes, which damage primary and secondary motor areas, severely limit the capacity for compensatory reorganization. Hence, the greater the damage to reciprocal intracortical pathways, the greater plasticity seen in secondary intact areas. Age Age influences motor recovery with younger patients making a greater motor recovery than older patients. The impact of stroke and recovery with age in animals is not entirely clear. In one study examining the effects of age on the development of ischemic injury in rats, the authors discovered that young rats were more affected by the stroke than old rats as exhibited by more pronounced neurological impairments and poorer performance in a water maze task (Shapira et al. 2002). In the rats, the duration of motor impairment post brain lesion increases with age (Brown et al. 2003). The regenerative response of neurons and glial cells, though largely

Page 13: C. Motor Recovery pictures removed - EBRSRPR).pdf · - 3 - C1. Assessment of Motor Function Post Stroke C1.1 Assessments of Motor Function Post Stroke Case Study A 65 year old male

- 13 - 13

preserved with age, appears to be delayed or occurs at a diminished rate the older the animal (Popa-Wagner et al. 1999, Whittemore et al. 1985). Reactive neuronal synaptogenesis declines (Scheff et al. 1978), sprouting responses are less robost (Schauwecker et al. 1995, Whittemore et al. 1985) and synaptic replacement rates diminish (Cotman and Anderson 1988). In humans, age has long been thought to diminish post-stroke neurological recovery (Nakayama et al. 1994). In a cohort study of 2219 patients, Kugler et al. (2003) studied the effect of patient age on early stroke recovery. The authors found that relative improvement decreased with increasing age: patients younger than 55 years achieved 67% of the maximum possible improvement compared with only 50% for patients above 55 years (p<0.001). They also found that age had a significant but relatively small impact on the speed of recovery with younger patients demonstrating a slightly faster functional recovery (p<0.001). The authors concluded that although age had a significant impact it nevertheless was a poor predictor of individual functional recovery after stroke and could not be regarded as a limiting factor in the rehabilitation of stroke patients. However, younger patients did demonstrate a more complete recovery. A prospective study that included 561 patients admitted to an inpatient stroke rehabilitation program found that age alone was a significant predictor of total FIM score and Motor FIM score at discharge, but not of FIM change (Bagg et al. 2002). For both total FIM score and Motor FIM score at discharge, age alone accounted for only 3% of the variance in outcome. The odds of successful discharge home was 18 times greater if the stroke patient was under 65 when compared to those patients over 85 years of age (Herman et al. 1984). Kotila et al. (1984) compared 77 stroke patients 65 or older versus 77 patients younger than 65. Younger patients had a better outcome in terms of return to home and ADLs. Bogousslavsky and Pierre (1992) found 202 or 1638 (12.3%) of patient from the Lausanne stroke registry with first ever ischemic stroke were 45 years or younger. They were divided into two groups: 1) Group 1 – age 16-30, n=56 (28%); 2) Group 2 – age 21-45, n=146 (72%). Prognosis was better for the Group 1 patients. Alexander (1994) reported that all patients under 55 years of age in his study group were discharged home. More importantly there was a significant greater FIM change in those patient aged <55 years than for the patients > 55 years and even a significantly greater FIM change for groups aged 55 to 74 years than for the group >75 years of age (Alexander 1994). Nakayama et al. (1994) reported that older stroke patients made the same degree of neurological recovery than younger patients but had a much lower functional gain. It was suggested that the more elderly patients had less compensatory abilities than younger stroke patients with comparable neurogical impairments. Kalra (1994) also reported that younger patients enjoyed greater functional recovery and higher rates of home discharge than elderly stroke survivors. References Alexander MP. Stroke rehabilitation outcome: A potential use of predictive variables to establish levels of care. Stroke 1994; 25(1):128-134. Bagg S, Pombo AP, Hopman W. Effect of age on functional outcomes after stroke rehabilitation. Stroke 2002; 33(1):179-185. Bogousslavsky J, Pierre P. Ischemic stroke in patients under age 45. Neurol Clin 1992; 10(1):113-124.

Page 14: C. Motor Recovery pictures removed - EBRSRPR).pdf · - 3 - C1. Assessment of Motor Function Post Stroke C1.1 Assessments of Motor Function Post Stroke Case Study A 65 year old male

- 14 - 14

Brown AM, Marlowe KJ, Bjelke B. Age effect on motor recovery in a post-acute animal stroke model. Neurobiol Aging 2003; 24(4):607-614. Cotman CW, Anderson KJ. Synaptic plasticity and functional stabilization in the hippocampal formation: Possible role in Alzheimer’s disease. Adv Neurol 1988; 47:313-335. Frost SB, Barbay S, Griel KM, et al. Reorganization of remote corical regions after ishchemic brain injury: a potential substrate for stroke recovery. J Neurophysiol 2003; 89(6):3205-3214. Herman JM, Culpepper L, Franks P. Patterns of utilization, disposition, and length of stay among stroke patients in a community hospital setting. J AM Geriatr Soc 1984; 32(6):421-426. Jorgensen HS, Nakayama H, Raaschou HO, Olsen TS. Recovery of walking function in stroke patients: The Copenhagen Stroke Study. Arch Phys Med Rehabil 1995(a); 76(1):27-32. Jorgensen HS, Nakayama H, Raaschou HO, et al. Outcome and time course of recovery in stroke. Part I: Outcome. The Copenhagen Stroke Study. Arch Phys Med Rehabil 1995(b); 76(1):27-32. Jorgensen HS, Nakayama H, Raaschou HO, et al. Outcome and time course of recovery in stroke. Part II: Time course of recovery. The Copenhagen Stroke Study. Arch Phys Med Rehabil 1995(c); 76(5):406-412. Kalra L. Does age affect benefits of stroke unit rehabilitation? Stroke 1994; 25(2):346-351. Kotila M, Waltimo O, Niemi ML, et al. The profile of recovery from stroke and factors influencing outcome. Stroke 1984; 15(6):1039-1044. Kugler C, Altenhoner T, Lochner P, Ferbert A. Does age influence early recovery from ischemic stroke? A study from the Hessian Stroke Data Bank. J Neurol 2003; 250(6):676-681. Nakayama H, Jorgensen HS, Raaschou HO, Olsen TS. Recovery of upper extremity function in stroke patients: the Copenhagen Stroke Study. Arch Phys Med Rehabil 1994;75:394-398. Nudo RJ. Adaptice plasticity in motor cortex: implication for rehabilitation after bain injury. J Rehabil Med 2003; (41Suppl):7-10. Popa-Wagner A, Schroder E, Schmoll H, et al. Upregulation of MAP1B and MAP2 in the rat brain after middle cerebral artery occlusion: Effect of age. J Cereb Blood Flow Metab 1999; 19(4):425-434. Schauwecker PE, Cheng HW, Serquinia RM, et al. Lesion-induced sprouting of commissural/associational axons and induction of GAP-43 mRNA in hilar and CA3 pyramidal neurons in the hippocampus are diminished in aged rats. J Neurosci 1995; 15(3 Pt 2):2462-2470. Scheff SW, Bernardo LS, Cotman CW. Decrease in adrenergic axon sprouting in the senescent rat. Science 1978; 202(4369):775-778. Shapira S, Sapir M, Wengier A, et al. Aging has a complex effect on a rat model of ischemic stroke. Brain Res 2002; 925(2):148-158. Skilbeck CE, Wade DT, Hewer RL, Wood VA. Recovery after stroke. J Neurol Neurosurg Psychiatry 1983; 46(1):5-8. Whittemore SR, Nieto-Sampedro M, Needles DL, Cotman CW. Neuronotrophic factors for mammalian brain neurons: Injury induction in neonatal, adult and aged rat brain. Brain Res 1985; 352(2):169-178.

Page 15: C. Motor Recovery pictures removed - EBRSRPR).pdf · - 3 - C1. Assessment of Motor Function Post Stroke C1.1 Assessments of Motor Function Post Stroke Case Study A 65 year old male

- 15 - 15

C3. Strength Training Post Stroke

Page 16: C. Motor Recovery pictures removed - EBRSRPR).pdf · - 3 - C1. Assessment of Motor Function Post Stroke C1.1 Assessments of Motor Function Post Stroke Case Study A 65 year old male

- 16 - 16

C3. Strength Training Post Stroke Q1. What is the evidence that strength training improves outcomes following stroke? Answers 1. There is mixed evidence that strength training improves outcomes post stroke. Some of the

studies were positive while others were not. Discussion Given that paresis or weakness is a common source of impairment and subsequent disability, strength training has been examined as a therapeutic approach. The central or upper motor neuron etiology of weakness makes stroke related weakness less amenable to strength training than other approaches such as deconditioning. An examination of all of the RCTs which evaluated strengthening programs (isokinetic or resistance training) reveals that there was variability in the length of time the treatment were provided which lasted from three to 12 weeks. Most subjects recruited were independent community ambulators. Heterogeneity of the types of interventions provided, their intensity and the outcomes assessed make it difficult to formulate conclusions as to the overall effectiveness of strength training treatment. Many studies failed to demonstrate a significant treatment effect, although the numbers of patients recruited were generally small. The studies with greater methodological rigour failed to demonstrate significant treatment effect. Summary of Studies Investigating Strength Training Post Stroke

Author PEDro Score N Control Comparison Outcome

Mead et al. 2007 8 66 Relaxation - Ouellette et al. 2004

7 42 Upper limb therapy -

Kim et al. 2001 7 20 Therapeutic Exercise - Flansbjer et al. 2008 6 24 Usual daily activities + Moreland et al. 2003 6 133 Therapeutic Exercise - Duncan et al. 1998 6 20 Therapeutic Exercise - Teixeira-Salmela et al. 1999

5 13 No Intervention +

Dean et al. 2000 5 12 Upper limb therapy + Bourbonnais et al. 2002

5 12 Untreated Limb +

Inaba et al. 1973 4 176 Functional training & selective stretching

+

Page et al. 2008 4 7 Home exercise program + Glasser 1986 4 20 Therapeutic Exercise - A further assessment of the outcomes of gait and ADL performance is presented below.

Page 17: C. Motor Recovery pictures removed - EBRSRPR).pdf · - 3 - C1. Assessment of Motor Function Post Stroke C1.1 Assessments of Motor Function Post Stroke Case Study A 65 year old male

- 17 - 17

Additional Summary of RCTs Examining Strength Training

Study Length of Intervention

Independence in Gait at Randomization

(+/- aid)

Significant Difference in Functional Outcome(s) Between Groups at End

of Treatment Inaba et al. 1974 1 mos Yes + (ADL) Glasser et al. 1986 5 weeks No - (Functional Ambulation Profile) Duncan et al. 1998 8 weeks Yes - (Gait velocity)

- (6 minute walk test) Teixeira-Salmela et al. 1999

10 weeks Yes + (Nottingham Health Profile)

Dean et al. 2000 4 weeks Yes + (Walking speed) Kim et al. 2001 12 weeks Yes - (Walking speed) Bourbonnais et al. 2002

3 weeks Not stated No b/w group comparisons reported

Moreland et al. 2003

Hospital stay Not stated - (Disability Inventory)

Ouellette et al. 2004 12 weeks Yes - (Late Life Function and Disability Instrument)

Mead et al. 2007 12 weeks Yes - (FIM) - (Walking speed)

Flansbjer et al. 2008

10 weeks Yes - (Fast gait speed, 6 minute walk test) + (Timed up and Go)

The SREBR (11th edition) concluded that there is conflicting evidence that strength training results in improvements in ADL performance, distance walked or gait speed (Teasell et al. 2008). References Bourbonnais D, Bilodeau S, Lepage Y, Beaudoin N, Gravel D, Forget R. Effect of force-feedback treatments in patients with chronic motor deficits after a stroke. Am J Phys Med Rehabil 2002; 81: 890-897. Dean CM, Richards CL, Malouin F. Task-related circuit training improves performance of locomotor tasks in chronic stroke: a randomized, controlled pilot trial. Arch Phys Med Rehabil 2000; 81:409-417. Duncan P, Richards L, Wallace D, et al. A randomized, controlled pilot study of a home-based exercise program for individuals with mild and moderate stroke. Stroke 1998; 29:2055-2060. Flansbjer UB, Miller M, Downham D, Lexell J. Progressive resistance training after stroke: effects on muscle strength, muscle tone, gait performance and perceived participation. J Rehabil Med 2008; 40:42-48. Glasser. Effects of isokinetic training on the rate of movement during ambulation in hemiparetic patients. Physical Therapy 1986; 66(5):673-676. Inaba M, Edberg E, Montgomery J, Gillis MK. Effectiveness of functional training, active exercise and resistive exercise for patients with hemiplegia. Physical Therapy 1973; 53:28-35.

Page 18: C. Motor Recovery pictures removed - EBRSRPR).pdf · - 3 - C1. Assessment of Motor Function Post Stroke C1.1 Assessments of Motor Function Post Stroke Case Study A 65 year old male

- 18 - 18

Kim CM, Eng JJ, MacIntyre DL, Dawson AS. Effects of isokinetic strength training on walking in persons with stroke: a double-blind controlled pilot study. Journal of Stroke and Cerebrovascular Disease 2001; 10(6):265-273. Mead GE, Greig CA, Cunningham I, et al. Stroke: a randomized trial of exercise or relaxation. J Am Geriatr Soc 2007; 55:892-899. Moreland JD, Goldsmith CH, Huijbregts MP, Anderson RE, Prentice DM, Brunton KB, O’Brien A, Torresin WD. Progressive resistance strengthening exercises after stroke: a single-blind randomized controlled trial. Arch Phys Med Rehabil 2003; 84:1433-40. Ouellette MM, LeBrasseur NK, Bean JF, et al. High-intensity resistance training improves muscle strength, self-reported function, and disability in long-term stroke survivors. Stroke 2004; 35:1404-1409. Page SJ, Levine P, Teepen J, Hartman EC. Resistance-based, reciprocal upper and lower limb locomotor training in chronic stroke: a randomized, controlled crossover study. Clin Rehabil 2008; 22:610-617. Teasell RW, Foley NC, Salter K, Bhogal SK, Jutai J, Speechley MR. Evidence-Based Review of Stroke Rehabilitation (11th edition). Canadian Stroke Network; 2008. www.ebrsr.com Teixeira-Samela LF, Olney SJ, Nadeau S, Brower B. Muscle strengthening and physical conditioning to reduce impairment and disability in chronic stroke survivors. Arch Phys Med Rehabil 1999; 80:1211-8.

Page 19: C. Motor Recovery pictures removed - EBRSRPR).pdf · - 3 - C1. Assessment of Motor Function Post Stroke C1.1 Assessments of Motor Function Post Stroke Case Study A 65 year old male

- 19 - 19

Key Study: Strength Training Post Stroke

Moreland JD, Goldsmith CH, Huijbregts MP, Anderson RE, Prentice DM, Brunton KB, O’Brien A, Torresin WD. Progressive resistance strengthening exercises after stroke: a single-blind randomized controlled trial. Arch Phys Med Rehabil 2003; 84:1433-40. Author / Year Country PEDro score

Methods Outcome

Moreland et al. 2003 Canada 6 (RCT)

133 stroke patients were randomized to the experimental group, receiving 9 lower-extremity progressive resistance exercises or to the control group who performed the same exercises without resistance. All patients received conventional physiotherapy.

No significant difference on the rate of change on the Disability Inventory or the 2-minute walking test was found between the groups.

Results of Unadjusted Primary Outcomes

25

30

35

40

45

50

55

60

65

Baseline 4 week Discharge 6 mo

2MW

T (m

)

Experimental Control

Importance: This RCT tested the benefit of two strengthening programs on gait. Unfortunately, strength training did not improve gait when compared to an exercise program which did not involve strength training. Although strength training has been shown to be helpful in a number of studies, the benefit is by no means consistent. Relevant SREBR Conclusions: There is strong evidence to the overall benefit of strengthening exercises for hemiparetic stroke patients, but moderate evidence that compared to therapeutic exercises, strength training does not provide additional benefits. Related References Badics E, Wittmann A, Rupp M, Stabauer B, Zifko UA. Systematic muscle building exercises in the rehabilitation of stroke patients. NeuroRehabilitation 2002;17(3):211-214.

Page 20: C. Motor Recovery pictures removed - EBRSRPR).pdf · - 3 - C1. Assessment of Motor Function Post Stroke C1.1 Assessments of Motor Function Post Stroke Case Study A 65 year old male

- 20 - 20

Bourbonnais D, Bilodeau S, Lepage Y, Beaudoin N, Gravel D, Forget R. Effect of force-feedback treatments in patients with chronic motor deficits after a stroke. Am J Phys Med Rehabil 2002;81:890-897. Dean CM, Richards CL, Malouin F. Task-related circuit training improves performance of locomotor tasks in chronic stroke: a randomized, controlled pilot trial. Arch Phys Med Rehabil 2000;81:409-417. Duncan P, Richards L, Wallace D, et al. A randomized, controlled pilot study of a home-based exercise program for individuals with mild and moderate stroke. Stroke 1998;29:2055-2060.

Inaba M, Edberg E, Montgomery J, Gillis MK. Effectiveness of functional training, active exercise and resistive exercise for patients with hemiplegia. Physical Therapy 1973;53:28-35. Sharp SA, Brouwer BJ. Isokinetic strength training of the hemiparetic knee: effects on function and spasticity. Arch Phys Med Rehabil 1997;78:1231-1236. Teixeira-Samela LF, Olney SJ, Nadeau S, Brower B. Muscle strengthening and physical conditioning to reduce impairment and disability in chronic stroke survivors. Arch Phys Med Rehabil 1999; 80:1211-8. Weiss A, Suzuki T, Bean J, Fielding RA: High intensity strength training improves strength and functional performance one year after stroke. Am J Phys Med Rehabil 2000;79:369-376.

Page 21: C. Motor Recovery pictures removed - EBRSRPR).pdf · - 3 - C1. Assessment of Motor Function Post Stroke C1.1 Assessments of Motor Function Post Stroke Case Study A 65 year old male

- 21 - 21

C4. Balance Retraining Post Stroke

Page 22: C. Motor Recovery pictures removed - EBRSRPR).pdf · - 3 - C1. Assessment of Motor Function Post Stroke C1.1 Assessments of Motor Function Post Stroke Case Study A 65 year old male

- 22 - 22

C4. Balance Retraining Post Stroke Q1. Is balance is a predictor of mobility post stroke? Answers 1. Improvement in balance has been identified as the strongest predictor of distance walked. Discussion Pohl et al. (2004) identified improvement in balance as the strongest predictor of distance gained in walking among poor performers (those who walked less than 213 metres) three months following stroke. Trunk flexion, extension muscle weakness, and asymmetric weight bearing may contribute to difficulties in walking and performing ADLs following a stroke. Q2. What evidence is there to treat balance problems post stroke. Answers 1. There are a number of treatments which have been shown to improve balance. 2. These therapy approaches are highly variable and include visual feedback, platform training,

strength training and cycle training. All have been shown to have an impact on outcomes. Discussion A variety of therapy approaches were assessed including visual feedback, task-specific methods, platform training, additional strength training, and cycle training. All of them were found to have a similar significantly positive impact on outcomes (Teasell et al. 2008). Summary of RCTs Examining Balance Treatments

Author Pedro Score ‘N’ Treatment Outcome

Van Nes et al. 2006 9 53 Whole body vibration - Mudie et al. 2002

8 40 Task Specific; Bobath methods; Balance Performance Monitor

(BPM) feedback training

+ BPM

Howe et al. 2005 7 35 Additional Therapy - Cheng et al. 2001

6 54 Symmetrical standing training

and repetitive sit-to-stand training

+

Morioka et al. 2003 6 28 Perceptual Learning + Bonan et al. 2004 6 20 Visual Cue Deprivation + De Seze et al. 2001 6 20 Bon Saint Come device + Sackley & Lincoln 1997 6 26 Visual biofeedback vs. sham

treatment +

Eser et al. 2008 6 41 Visual feedback using Nor-Am

Page 23: C. Motor Recovery pictures removed - EBRSRPR).pdf · - 3 - C1. Assessment of Motor Function Post Stroke C1.1 Assessments of Motor Function Post Stroke Case Study A 65 year old male

- 23 - 23

Target Balance Training System Grant et al. 1997 5 16 Biofeedback Balance Training - Allison et al. 2007 5 17 Standing practice - Yavuzer et al. 2006 5 50 Nor-Am Target Balance Training

System +

Wong et al. 1997 5 60 Standing training device vs. visual feedback training device

+ (feedback)

Walker et al. 2000 5 54 Visual Feedback Training - References Allison R, Dennett R. Pilot randomized controlled trial to assess the impact of additional supported standing practice on functional ability post stroke. Clin Rehabil 2007; 21:614-619. Cheng PT, Wu SH, Liaw MY, Wong AM, Tang FT. Symmetrical body-weight distribution training in stroke patients and its effect on fall prevention. Arch Phys Med Rehabil 2001; 82:1650-1654. Bonan IV, Yelnik AP, Colle FM et al. Reliance on visal information after stroke. Part II: Effectiveness of a balance rehabilitation program with visual cue deprivation after stroke: A randomized controlled trial. Arch Phys Med Rehabil 2004; 85:274-278. De Seze M, Wiart L, Bon-Saint-Come A, Debelleix S, de Seze M, Joseph PA, Mazaux JM, Barat M. Rehabilitation of postural disturbances of hemiplegic patients by using trunk control retraining during exploratory exercises. Arch Phys Med Rehabil 2001; 82:793-800. Eser F, Yavuzer G, Karakus D, Karaoglan B. The effect of balance training on motor recovery and ambulation after stroke: a randomized controlled trial. Eur J Phys Rehabil Med 2008; 44:19-25. Grant T, Brouwer B, Culham E, Vandervoort A. Balance retraining following acute stroke: a comparison of two methods. Canadian Journal of Rehabilitation 1997; 11:69-73. Howe TE, Taylor I, Finn P, Jones H. Lateral weight transference exercises following acute stroke: a preliminary study of clinical effectiveness. Clin Rehabil 2005; 19:45-53. Morioka S, Yagi F. Effects of perceptual learning exercises on standing balance using a hardness discrimination task in hemiplegic patients following stroke: a randomized controlled pilot trial. Clinical Rehabilitation 2003; 17:600-607. Mudie MH. Training symmetry of weight distribuation after stroke: a randomized controlled pilot study comparing task-related reach, Bobath and feedback training approaches. Clinical Rehabilitation 2002;16:582-592. Pohl PS, Perera S, Duncan PW, Maletsky R, Whitman R, Studenski S. Gains in distance walking in a 3-month follow-up poststroke: what changes? Neurorehabil Neural Repair 2004;18:30-36. Teasell RW, Foley NC, Salter K, Bhogal SK, Jutai J, Speechley MR. Evidence-Based Review of Stroke Rehabilitation (11th edition). Canadian Stroke Network; 2008. www.ebrsr.com Sackley CM, Lincoln NB. Single blind randomized controlled trial of visual feedback after stroke: effects on stance symmetry and function. Disabil Rehabil 1997; 19:536-546. van Nes IJ, Latour H, Schils F, Meijer R, van Kuijk A, Geurts AC. Long-term effects of 6-week whole-body vibration on balance recovery and activities of daily living in the postacute phase of stroke: a randomized, controlled trial. Stroke 2006;37:2331-2335.

Page 24: C. Motor Recovery pictures removed - EBRSRPR).pdf · - 3 - C1. Assessment of Motor Function Post Stroke C1.1 Assessments of Motor Function Post Stroke Case Study A 65 year old male

- 24 - 24

Walker C, Brouwer BJ, Culham EG. Use of visual feedback in retraining balance following acute stroke. Phys Ther 2000; 80:886-895. Wong AM, Lee MY, Kuo JK, Tang FT. The development and clinical evaluation of a standing biofeedback trainer. J Rehabil Res Dev 1997; 34:322-327. Yavuzer G, Eser F, Karakus D et al. The effects of balance training on gait late after stroke: a randomized controlled trial. Clin Rehabil. 2006; 20:960-969.

Page 25: C. Motor Recovery pictures removed - EBRSRPR).pdf · - 3 - C1. Assessment of Motor Function Post Stroke C1.1 Assessments of Motor Function Post Stroke Case Study A 65 year old male

- 25 - 25

C5. The Bobath Approach / Neurodevelopmental Technique of Motor Therapy

Page 26: C. Motor Recovery pictures removed - EBRSRPR).pdf · - 3 - C1. Assessment of Motor Function Post Stroke C1.1 Assessments of Motor Function Post Stroke Case Study A 65 year old male

- 26 - 26

C5. The Bobath Approach / Neurodevelopmental Technique of Motor Therapy C5.1 The Bobath Approach Case Study A 70 year female suffered a large hemispheric stroke with a significant hemiplegia. On admission to rehabilitation 14 days following the onset of the stroke the patient has shown some improvement, with no recovery in the upper extremity and CMS of 3 in the leg and 2 in the foot. The therapists enter you into a discussion about the use of the Bobath technique and its use in this case. Q1. Describe the Bobath Approach/Neurodevelopmental Technique for the therapy of stroke patients. Answers 1. Bobath Approach/Neurodevelopmental Technique (Bobath 1978) 2. The goal of NDT is to normalize tone, to inhibit primitive patterns of movement, and to

facilitate automatic, voluntary reactions and subsequent normal movement patterns. 3. Based on the concept that pathologic movement patterns (limb synergies and primitive

reflexes) must not be used for training because continuous use of these pathologic pathways may make it too readily available at the expense of the normal pathways.

4. The goal is to suppress abnormal muscle patterns before normal patterns are introduced 5. Mass synergies are avoided, although they may strengthen weak, unresponsive muscles,

because these reinforce abnormally increased tonic reflexes, spasticity. 6. Abnormal patterns modified at proximal key points of control (e.g., shoulder and pelvic

girdle). Discussion The neurodevelopmental treatment approach, commonly referred to as NDT or “Bobath”, is based on the philosophy of Bertha Bobath. Price and Reding (1994) have noted that, “Bobath based her approach to hemiplegia on the assumption that ‘spasticity is caused by the release of an abnormal postural reflex mechanism.’ Bobath put forth this view after considering contemporary physiology … She acknowledged that the approach was developed empirically, and the underlying theory was developed ‘as a working hypothesis to explain the observed facts.’ Based on this theory, Bobath emphasizes inhibition of the abnormal reflex patterns, and facilitation of normal, volitional movement patterns. Bobath suggests that proper handling of the

Page 27: C. Motor Recovery pictures removed - EBRSRPR).pdf · - 3 - C1. Assessment of Motor Function Post Stroke C1.1 Assessments of Motor Function Post Stroke Case Study A 65 year old male

- 27 - 27

hemiplegic patient will direct such patterns ‘into the channels of the higher integrated and complex patterns of more normal coordination.’ Again, there is a belief in the potential for neurological recovery. Bobath writes ‘Experience has shown that there is in every patient some untapped potential for more highly organized activity.’ … Assessment of the patient with hemiplegia emphasizes analysis of motor patterns and reflex responses, rather than weakness (weakness or paralysis is considered a consequence of deficient postural reflex patterns). … Treatment is then planned to restore (facilitate) the normal patterns, and to eliminate the abnormal patterns. Exercises that take advantage of overflow, associated reactions, synergy or any ‘pathological’ reflex or pattern, are strictly avoided. In NDT, such methods are viewed as a cause of spasticity, and are believed to interfere with recovery. The resulting exercises are intended to put the patient through ‘reflex-inhibiting movement patterns’ with the twofold purpose of suppressing spasticity and facilitating normal voluntary movement.” Price and Reding (1994) have also noted, “Specific therapeutic exercises depend on whether the patient’s hemiplegia is flaccid, spastic, or resolving with relative recovery. Treatment in the flaccid stage consists largely of positioning and activities intended to prepare the patient for sitting,s tanding and ambulation. As tone returns, treatment begins to incorporate activities carried out while the patient is sitting or standing. If spasticity interferes, it is viewed as a consequence of abnormal postural reflexes. The therapist must inhibit these reflexes, or defer the activity involved until the patient is better prepared. The goal of treatment is a normal quality of gait and upper extremity movement. Emphasis is placed on bearing weight and using the extremities on the affected side to prevent learning to compensate with the intact side. As gait training begins, walking is accomplished with the aid of the therapist but without devices or bracing, encouraging the patient to bear weight and walk symmetrically. The philosophy of NDT is summarized in Berta Bobath’s statement that ‘it must always be remembered that the aim of this type of treatment is to improve the quality of movement on the affected side.’ … ‘In order to prepare for a reasonably normal gait, balance, stance and weight transfer should be practiced. If all this is first practiced while in the standing position, [the patient] will develop a better walking pattern than if he is made to walk immediately without the necessary control of his leg.’” C5.2 Neurodevelopmental Approaches to Motor Recovery Post Stroke Q2. Describe the various Neurodevelopmental Approaches to Motor Recovery Post Stroke. Answer 1. Bobath 2. Brunnstrom’s Movement Therapy 3. Proprioceptive Neuromuscular Facilitation

Page 28: C. Motor Recovery pictures removed - EBRSRPR).pdf · - 3 - C1. Assessment of Motor Function Post Stroke C1.1 Assessments of Motor Function Post Stroke Case Study A 65 year old male

- 28 - 28

Discussion The concept of NDT emphasizes that abnormal muscle patterns or muscle tone have to be inhibited and that normal patterns should be used in order to facilitate functional and voluntary movements. There are a number of approaches that fall under the heading of neurodevelopmental techniques, including the Bobath, Brunnstrom and Proprioceptive Neuromuscular Facilitation approaches. These techniques are described below: Neurodevelopmental Training (NDT) Approaches

Approach Description Bobath Aims to reduce spasticity and synergies by using inhibitory postures and

movements in order to facilitate normal autonomic responses that are involved in voluntary movement (Bobath 1990).

Brunnstrom’s Movement Therapy

Emphasis on synergistic patterns of movement that develop during recovery from hemiplegia. Encourages the development of flexor and extensor synergies during early recovery, assuming that synergistic activation of the muscle will result in voluntary movement (Brunnstrom 1970).

Proprioceptive Neuromuscular Facilitation (PNF)

Emphasis on using the patient’s stronger movement patterns for strengthening the weaker motions. PNF techniques use manual stimulatin and verbal instructions to induce desired movement patterns and enhance motor function (Myers 1995).

C5.3 The Evidence Supporting Neurodevelopmental/Bobath Thearpy Q3. The physical and occupational therapists on the rehabilitation team have been trained in neurodevelopmental therapy (NDT), in particular the Bobath technique. What is the evidence with regard to using NDT in the treatment of the hemiparetic upper extremity following a stroke? Answer 1. Strong evidence that NDT is not superior to other approaches. 2. Moderate evidence that Motor Relearning Program (task-specific training) results in short-

term improvements in motor functioning and shorter lengths of hospital stay when compared to NDT.

Discussion A variety of treatment approaches are currently in use. Arguably, the Bobath approach (a neurodevelopmental technique) is the most commonly used, although other methods such as motor re-learning and orthopedic or mixed technique are also used. The Bobath approach aims to reduce spasticity and synergies by using inhibitory postures and movements in order to facilitate normal autonomic responses that are involved in voluntary movement (Bobath 1990). The results from two systematic reviews authored by Barreca et al. (2003) and Paci et al. (2003) indicated that no one method emerged as superior. The SREBR identified two high quality RCTs that compared Motor Relearning Programme (MRP) and Bobath treatment approaches. The results from these trials conflicted.

Page 29: C. Motor Recovery pictures removed - EBRSRPR).pdf · - 3 - C1. Assessment of Motor Function Post Stroke C1.1 Assessments of Motor Function Post Stroke Case Study A 65 year old male

- 29 - 29

Two high quality studies compared the neurodevelopmental approach to the motor relearning approach. Langhammer and Stanghelle (2000) reported improvements in upper extremity function and a shorter length of hospital stay was associated with motor relearning while Van Vliet et al. (2005) did not report any significant difference between treatment approaches. Van Vliet et al. (2005) speculated that earlier, more intensive training provided in the Langhammer and Stanghelle (2000) study as well as a higher (albeit non-statistically significant) baseline difference may have contributed to the differences. Platz et al. (2005) failed to demonstrate an effect of augmented arm therapy (in addition to regular rehabilitation) upon motor recovery, regardless of the treatment approach (BASIS arm training or Bobath). Hafsteinsdottir et al. (2007) also reported that the Bobath approach was not superior to that of the non-NDT approach. There were no differences between the groups on any of the outcome measures assessed including the FIM, quality of life, health-related quality of life, shoulder pain or depression at up to 12 months following stroke. Conclusions from the Stroke Rehabilitation Evidence-Based Review (11th edition) (Teasell et al. 2008) noted that there is strong evidence that neurodevelopmental techniques are not superior to other therapeutic approaches. There was moderate evidence from one “good” RCT that indicated that when compared to the Bobath treatment approach, the Motor Relearning Programme may be associated with improvements in short-term motor functioning and is associated with shorter lengths of hospital stay. References Barreca S, Wolf S L, Fasoli S, Bohannon R. Treatment interventions for the paretic upper limb of stroke survivors: a critical review. Neurorehabil Neural Repair 2003;17(4):220-226. Bobath, B. (Ed.). Adult Hemiplegia: evaluation and treatment. London: Heinemann Medical Books; 1990. Bobath B. Hemiplegia: evaluation and treatment. London: Butterworth-Heinemann, 1978. Brunnstrom S. Movement Therapy in Hemiplegia. New York: Harper & Row; 1970. Hafsteinsdottir TB, Kappelle J, Grypdonck MH, Algra A. Effects of Bobath-based therapy on depression, shoulder pain and health-related quality of life in patients after stroke. J Rehabil Med 2007;39:627-632. Langhammer B, Stanghelle JK. Bobath or motor relearning programme? A comparison of two different approaches of physiotherapy in stroke rehabilitation: a randomized controlled study. Clin Rehabil 2000;14(4):361-9. Myers BJ. Proprioceptive neuromuscular facilitation approach. In Trombly CA, ed. (Ed.), Occupational Therapy for Physical Dysfunction: 474-498. Baltimore, MD: Williams & Wilkins; 1995. Paci M. Physiotherapy based on the Bobath concept for adults with post-stroke hemiplegia: a review of effectiveness studies. J Rehabil Med 2003;35:2-7. Platz T, Eickhof C, van Kaick S, et al. Impairment-oriented training or Bobath therapy for severe arm paresis after stroke: a single-blind, multicentre randomized controlled trial. Clin Rehabil 2005;19:714- 724. Price SJ, Reding MJ. Physical therapy philosophies and strategies. In Good DC, Couch JR Jr. (Ed.), Handbook of Neurorehabilitation (pp. 181-196). New York: Marcel Dekker, 1994.

Page 30: C. Motor Recovery pictures removed - EBRSRPR).pdf · - 3 - C1. Assessment of Motor Function Post Stroke C1.1 Assessments of Motor Function Post Stroke Case Study A 65 year old male

- 30 - 30

Teasell RW, Foley NC, Salter K, Bhogal SK, Jutai J, Speechley MR. Evidence-Based Review of Stroke Rehabilitation (11th edition). Canadian Stroke Network; 2008. van Vliet PM, Lincoln NB, Foxall A. Comparison of Bobath based and movement science based treatment for stroke: a randomised controlled trial. J Neurol Neurosurg Psychiatry 2005;76:503-508.

Page 31: C. Motor Recovery pictures removed - EBRSRPR).pdf · - 3 - C1. Assessment of Motor Function Post Stroke C1.1 Assessments of Motor Function Post Stroke Case Study A 65 year old male

- 31 - 31

Key Study: Bobath vs. Motor Learning Physiotherapy in Stroke Langhammer B, Stanghelle JK. Bobath or motor relearning programme? A comparison of two different approach of physiotherapy in stroke rehabilitation: a randomized controlled study. Clinical Rehabilitation 2000;14:361-369. Langhammer B, Stanghelle JK. Bobath or Motor Relearning Programme? A follow-up one and four years post stroke. Clinical Rehabilitation 2003; 17:731-734. Author / Year Country PEDro score

Methods Outcome

Langhammer et al. 2000, 2003 Norway 8 (RCT)

A double blind trial of 61 stroke patients randomized to receive therapy based on Bobath concept (represents a theoretical framework in a reflex-hierarchical theory) or to receive a Motor Relearning Programme (based on system theory and task oriented) (MRP). All patients received physiotherapy 5 days weekly with a minimum of 40 minutes duration as long as they were hospitalized and the same comprehensive multidisciplinary treatment for stroke patients from doctors, nurses, occupational therapist and speech therapist according to recommendation for stroke units in Norway.

Length of stay was significantly shorter in MRP group compared to Bobath group, 21 vs. 34 days.

Motor Learning vs. Bobath

Langhammer et al. 2000 Mean MAS scores at 3 Tests for Physiotherapy using the Motor Relearning Programme

(MRP) vs. Bobath

p=.0001

p=.0001

p=.05 p=.0001

NS

p=.0001NS

0

10

20

30

40

Test 1 Test 2 Test 3

Mean Motor Assessment Scale (MAS)

Out

com

e M

easu

re

Valu

es

MRP Bobath

Gender Difference using the Life Quality Test (NHP)

p=.0003p=.0007NS

p=.0006

p=.01

p=.001NS

0

20

40

60

80

Energy Sleep Emotions Mobility Pain Social Life Total

NHP Categories

NH

P Va

lues Women Men

Page 32: C. Motor Recovery pictures removed - EBRSRPR).pdf · - 3 - C1. Assessment of Motor Function Post Stroke C1.1 Assessments of Motor Function Post Stroke Case Study A 65 year old male

- 32 - 32

21

34

p=.008

0 5 10 15 20 25 30 35

Length of Stay (days)

MRP

Bobath

LOS for Physiotherapy using the Motor Relearning Programme (MRP) vs. Bobath

Langhammer et al. 2003

Outcome Measures at 3 Assessments Following Stroke: Motor Relearning Programme (MRP) vs. Bobath Group

Outcome Measures:

MAS=Motor Assessment Scale

SMES=Sodring Motor Evaluation

ScaleNHP=Nottingham

Health Profile0

102030405060708090

3 M

o

1 Yr

4 Yr

3 M

o

1 Yr

4 Yr

3 M

o

1 Yr

4 Yr

3 M

o

1 Yr

4 Yr

3 M

o

1 Yr

4 Yr

3 M

o

1 Yr

4 Yr

MAS SMES1 SMES2 SMES3 Barthel NHPOutcome Measure

Note: No significant differences were found between groups

Sco

re

MRPBobath

Importance: One of the great debates in physiotherapy is whether the neuro-developmental (or restorative) approach, is preferred or whether the compensatory, task-focused, adaptive approach is superior. The most common restorative technique is the Bobath approach that is based upon a theoretical framework in a reflex-hierarchical therapy. Synergistic movements are supported while normal movements are facilitated and encouraged. Langhammer and Stronghelle (2000, 2003), in a RCT, compared the Bobath approach to the Motor Relearning Programme and found the latter resulted in shorter hospital stays and improved motor function. Relevant SREBR Conclusions: there is strong evidence that the Bobath approach is not superior to other therapy approaches. There is conflicting evidence that the Motor Relearning approach is superior to the Bobath approach for achieving improvements in functional outcome. There is moderate evidence that the Motor Relearning Programme reduced length of hospital stay. Related References Dean CM, Shepherd RB. Task-related training improves performance of seated reaching tasks after stroke. A randomized controlled trial. Stroke 1997;28:722-728. van Vliet PM, Lincoln NB, Foxall A. Comparison of Bobath based and movement science based treatment for stroke: a randomised controlled trial. J Neurol Neurosurg Psychiatry 2005;76:503-508. Wang RY, Chen HI, Chen CY, Yang YR. Efficacy of Bobath versus orthopaedic approach on impairment and function at different motor recovery stages after stroke: a randomized controlled study. Clin Rehabil 2005;19:155-164.

Page 33: C. Motor Recovery pictures removed - EBRSRPR).pdf · - 3 - C1. Assessment of Motor Function Post Stroke C1.1 Assessments of Motor Function Post Stroke Case Study A 65 year old male

- 33 - 33

C6. Task-Specific Therapy

Page 34: C. Motor Recovery pictures removed - EBRSRPR).pdf · - 3 - C1. Assessment of Motor Function Post Stroke C1.1 Assessments of Motor Function Post Stroke Case Study A 65 year old male

- 34 - 34

C6. Task-Specific Therapy C6.1 Task-Specific Training for Mobility Q1. Why is Task-Specific Training for Mobility Being Increasingly Recommended? Answers 1. Task-specific training has been shown to be associated with improved gait-specific

outcomes. 2. Task-specific training has been shown to have longer-lasting cortical reorganization. Discussion According to Classen et al. (1998), focal transcranial stimulation and functional magnetic resonance imaging have shown that task-specific training, in comparison to traditional stroke rehabilitation, yields long-lasting cortical reorganization specific to the corresponding areas being used. Indeed, in a focal Cochrane review, French et al. (2007) showed that mobility task-specific training was associated with improvements in walking distance and speed and performance in sit-to-stand. Salbach et al. (2004) demonstrated task-specific training of lower extremities designed to strengthen the lower extremities and enhance walking balance, speed, and distance showed improvement in a number of gait parameters. C6.2 Motor Relearning Program Q2. Describe the Motor Relearning Program (Carr and Kenney 1992, Carr et al. 1985). Answers 1. Based on cognitive motor relearning theory. 2. Goal is for the patient to relearn how to move functionally and how to problem solve during

attempts at new tasks. 3. Instead of emphasizing repetitive performance of a specific movement for improving skill, it

teaches general strategies for solving motor problems. 4. Emphasizes functional training of specific tasks, such as standing and walking, and

carryover of those tasks. References Carr EK, Kenney FD. Positioning of the stroke patient: a review of the literature. Int J Nurs Stud 1992;29(4):355-369. Carr JH, Shepherd RB, Nordholm L, Lynne D. Investigation of a new motor assessment scale for stroke patients. Physical Therapy 1985; 65: 175-80.

Page 35: C. Motor Recovery pictures removed - EBRSRPR).pdf · - 3 - C1. Assessment of Motor Function Post Stroke C1.1 Assessments of Motor Function Post Stroke Case Study A 65 year old male

- 35 - 35

Classen J, Liepert J, Wise SP, Hallett M, Cohen LG. Rapid plasticity of human cortical movement representation induced by practice. J Neurophysiol 1998;79(2):1117-23. French B, Thomas LH, Leathley MJ, et al. Repetitive task training for improving functional ability after stroke. Cochrane Database Syst Rev 2007;CD006073. Salbach NM, Mayo NE, Wood-Dauphinee S, Hanley JA, Richards CL, Cote R. A task orientated intervention enhances walking distance and speed in the first year post 9. Mobility and the Lower Extremity pg. 107 of 110 www.ebrsr.com stroke: a randomized controlled trial. Clin Rehabil 2004;18:509-519.

Page 36: C. Motor Recovery pictures removed - EBRSRPR).pdf · - 3 - C1. Assessment of Motor Function Post Stroke C1.1 Assessments of Motor Function Post Stroke Case Study A 65 year old male

- 36 - 36

Key Study: Task Specific Therpay and Walking Salbach NM, Mayo NE, Wood-Dauphinee S, Hanley JA, Richards CL, Cote R. A task-orientated intervention enhances walking distance and speed in the first year post stroke: a randomized controlled trial. Clinical Rehabilitation 2004;18:509-519. Author / Year Country PEDro score

Methods Outcome

Salbach et al. 2004 Canada 8 (RCT)

91 community-dwelling subjects with a residual walking deficit within one year of a first or recurrent stroke were randomized to an intervention which comprised 10 functional tasks designed to strengthen the lower extremities and enhance walking balance, speed and distance or to a control intervention focusing on upper extremity activities, 3 days a wk x 6 wks. The main outcomes assessed were 6-minute walk test (SMWT), 5-m walk (comfortable and maximum pace), Berg Balance Scale and timed 'up and go' test.

Following treatment patients in the intervention group had attained achieved greater improvements on the following outcomes measures: SMWT (40 m vs. 5m); comfortable walking speed (0.14 vs. 0.03 m/s); maximum walking speed (0.20 vs. –0.01 m/s); TUG (-1.2 vs. 1.7sec).

Performance on measures of Walking Speeds

-0.05

0

0.05

0.1

0.15

0.2

0.25

ComfortableWalking Speed

Maximum WalkingSpeed

Cha

nge

in S

peed

Ove

r 6 w

eeks

of

Tra

inin

g (m

/s)

Mobility Training UE Training

Performance measures for the Six-minute Walking Test

200

210

220

230

240

250

260

Pre Post6 week Training Period

Dis

tanc

e (m

)

Mobility Training UE Training

Importance: Results from the studies of Dean et al. (2003) and Salbach et al. (2004) suggest that therapy designed to improve the strength and endurance of the affected lower limb and functional performance demonstrated improvement that was specific to the training. Relevant SREBR Conclusion: There is strong evidence that task-specific gait training improves gait post-stroke. Related Reference Dean CM, Richards CL, Malouin F. Task-related circuit training improves performance of locomotor tasks in chronic stroke: a randomized, controlled pilot trial. Arch Phys Med Rehabil 2000;81:409-417.

Page 37: C. Motor Recovery pictures removed - EBRSRPR).pdf · - 3 - C1. Assessment of Motor Function Post Stroke C1.1 Assessments of Motor Function Post Stroke Case Study A 65 year old male

- 37 - 37

C7. Treadmill Training and Partial Weight Support

Page 38: C. Motor Recovery pictures removed - EBRSRPR).pdf · - 3 - C1. Assessment of Motor Function Post Stroke C1.1 Assessments of Motor Function Post Stroke Case Study A 65 year old male

- 38 - 38

C7. Treadmill Training and Partial Weight Support C7.1 Treadmill Training Q1. What Evidence is there for the Effectiveness of Treadmill Training in the Absence of Partial Body Weight Support? Answer 1. The evidence does not support it as necessarily more effective than standard gait training. Discussion Treadmill training has been used as a form of gait specific training despite conflicting evidence as to whether or not it is more beneficial than standard gait training. Moseley et al. (2003) conducted a meta-analysis to determine the effectiveness of treadmill training and body weight support for walking after stroke. The authors found that, as compared to other physiotherapy interventions, treadmill training (with and without other task-oriented exercises) had no significant effects on walking speed. Nevertheless, there has been some indication that more aggressive progressive training programs may help to improve the effectiveness of treadmill training. For example, Pohl et al. (2002) found that stroke patients who received structured speed-dependent treadmill training scored significantly higher on measures of walking speed, cadence, stride length, and functional ambulation after 4 weeks of training. C7.2 Treadmill Training and Partial Body Weight Support Q2. What evidence is there for the effectiveness of treadmill training and partial body weight support? Answers 1. The evidence of PBWS and treadmill training is mixed but the weight of the evidence is

moving towards supporting PBWS. 2. Is supported by the general trend towards task-specific therapies. 3. Does require some equipment and can be therapist intensive. Discussion Partial body weight support and treadmill training is a promising new approach that enables non-ambulatory stroke survivors the repetitive practice of complex gait cycles rather than single-limb gait preparatory manoeuvres. Patients are able to walk more symmetrically with less spasticity and improved cardiovascular efficiency, which the treadmill provides when compared to floor walking (Hesse et al. 2003). Among the trials we reviewed, the results were ambiguous, although the differences in the duration and intensity of the treatment protocols limit interpretability of the findings. Nilsson et al. (2001), a “good” quality trial”, found no significant differences between the groups on a variety of outcomes either at discharge or at the 10-month

Page 39: C. Motor Recovery pictures removed - EBRSRPR).pdf · - 3 - C1. Assessment of Motor Function Post Stroke C1.1 Assessments of Motor Function Post Stroke Case Study A 65 year old male

- 39 - 39

follow-up. Eich et al. (2004), Visintin et al. (1998), Werner et al. (2002), Barbeau et al. (2003) and Tong et al. (2006) also “good” quality trials, found the combination of treadmill training and partial weight-bearing support resulted in significant benefits in motor recovery and walking speed, which were still present at the 3-month follow up. In one of the more recent trials Sullivan et al. (2007) reported that treadmill training was more effective than resisted cycle training for improving gait speed in ambulatory, chronic stroke subjects. The authors argue that body-weight supported treadmill training provides a unique form of treatment that combines forms task-specific and strength training, resulting in clinically significant improvements in walking speed. Some of the reviewed studies evaluated variations of treadmill training, which were peripheral to the question of whether treadmill training is an efficacious treatment. For example, Eich et al. (2004) used an inclined treadmill and a predetermined defined heart rate as part of their protocol. In other cases, the choice of control condition complicated the process of reaching conclusions. Sullivan et al. (2002) noted that significant gait improvements were achieved at higher training speed, compared with slower speeds; however, this study did not include a true control condition. Kosak et al. (2000) found no significant benefit of treadmill training compared to aggressive bracing assisted walking. The most common control condition was that of routine rehabilitation. Among the four studies that used a two-group design with conventional rehabilitation as the control condition, (daCuhna et al. 2001, Nilsson et al. 2001, Werner et al. 2002, Dias et al. 2007) only one demonstrated significantly greater improvement with treadmill training. Therefore, although this appears to be a promising treatment approach, the evidence is conflicting, with regard to the evidence of partial body-weight support in gait training. Yaugra et al. (2006) evaluated the additional effect of the facilitation technique upon treadmill training with body weight support, whereby “a therapist presses the hip of the patients forward and backward to ensure stable swing and stance of the paretic leg” and reported no additional benefit associated with the intervention. Given the uncertainty surrounding the treatment effect of treadmill training as a tool to enhance and promote gait restoration following stroke, Ada et al. (2009) proposed a study protocol which seeks to further our understanding. The proposed randomized controlled trial will recruit a target of 130 acute stroke survivors, with a 6-month follow-up period. Details of timelines for recruitment and study completion were not provided. Duncan et al. (2007) have also published a trial protocol with similar objectives and outcome measurements-the Locomotor Experience Applied Post-Stroke (LEAPS) trial is to determine if there is a difference in the proportion of participants who recover walking ability at one year post-stroke compared with subjects who receive a home-based exercise program. This is a phase III, 5-year trial. This trial will also recruit acute subjects, but will attempt to recruit larger numbers (400). The SREBR (11th edition) notes that there is conflicting evidence that the combination of partial body weight support and treadmill training results in improved gait performance compared with other physiotherapy interventions (Teasell et al. 2008). Thus, while there is continued uncertainty regarding the effectiveness of PWBS, further research is required to determine whether or not it is beneficial and in which circumstances. References

Page 40: C. Motor Recovery pictures removed - EBRSRPR).pdf · - 3 - C1. Assessment of Motor Function Post Stroke C1.1 Assessments of Motor Function Post Stroke Case Study A 65 year old male

- 40 - 40

Ada L, Dean CM, Lindley R, Lloyd G. Improving community ambulation after stroke: the AMBULATE trial. BMC Neurology 2009; 9(8). Barbeau H, Visintin M. Optimal outcomes obtained with body-weight support combined with treadmill training in stroke subjects. Arch Phys Med Rehabil 2003; 84:1458-65. da Cunha, I. T., Jr., Lim, P. A., Qureshy, H., Henson, H., Monga, T., & Protas, E. J. Gait outcomes after acute stroke rehabilitation with supported treadmill ambulation training: a randomized controlled pilot study. Arch Phys Med Rehabil 2002; 83(9):1258-1265. Dias D, Lains J, Pereira A, et al. Can we improve gait skills in chronic hemiplegics? A randomised control trial with gait trainer. Eura Medicophys 2007; 43:499-504. Duncan PW, Sullivan KJ, Behrman AL, et al. Protocol for the Locomotor Experience Applied Post-stroke (LEAPS) trial: a randomized controlled trial. BMC Neurol 2007; 7:39. Eich HJ, Mach H, Werner C, Hesse S. Aerobic treadmill plus Bobath walking training improves walking in subacute stroke: a randomized controlled trial. Clin Rehabil 2004;18:640-651. Hesse S, Werner C, von Frankenberg S, Bardelben A. Treadmill training with partial body weight support after stroke. Physical Medicine Rehabilitation Clinic North America 2003; 14:S111-S123. Kosak MC, Reding MJ. Comparison of partial body weight-supported treadmill gait training versus aggressive bracing assisted walking post stroke. Neurorehabil Neural Repair 2000; 14:13-19. Moseley AM, Stark A, Cameron ID, Pollock A. Treadmill training and body weight support for walking after stroke. Stroke 2003; 34: 3006. Nisson L, Carlsson J, Danielsson A, Fugl-Meyer A, Hellstrom K, Kristensen L, Sjolund B, Sunnerhagen K, Grimby G. Walking training of patients with hemiparesis at an early stage on a treadmill with body weight support and walking training on the group. Clinical Rehabilitation 2001; 15:515-527. Pohl M, Mehrholz J, Ritschel C, Ruchriem S. Speed-dependent treadmill training in ambulatory hemiparetic stroke patients. A randomized controlled trial. Stroke 2002; 33:553-558. Sullivan KJ, Brown DA, Klassen T, et al. Effects of task-specific locomotor and strength training in adults who were ambulatory after stroke: results of the STEPS randomized clinical trial. Phys Ther 2007; 87:1580-1602. Sullivan KJ, Dnowlton BJ, Dobkin BH. Step training ith body weight support: effect of treadmill and speed practice paradigms on poststroke locomotor recovery. Arch Phys Med Rehabil 2002;83:683-691. Teasell RW, Foley NC, Salter K, Bhogal SK, Jutai J, Speechley MR. Evidence-Based Review of Stroke Rehabilitation (11th edition). Canadian Stroke Network; 2008. Tong RK, Ng MF, Li LS. Effectiveness of gait training using an electromechanical gait trainer, with and without functional electric stimulation, in subacute stroke: a randomized controlled trial. Arch Phys Med Rehabil 2006; 87:1298-1304. Visintin M, Barbeau H, Korner-Bitensky N, Mayo NE. A new approach to retrain gait in stroke patients through body weight support and treadmill stimulation. Stroke 1998; 29:1122-1128. Werner C, von Frankenberg S, Treig T, Konrad M, Hesse S. Treadmill training with partial body weight support and an electromechanical gait trainer for restoration of gait in subacute stroke patients. A randomized crossover study. Stroke 2002; 33:2895-2901.

Page 41: C. Motor Recovery pictures removed - EBRSRPR).pdf · - 3 - C1. Assessment of Motor Function Post Stroke C1.1 Assessments of Motor Function Post Stroke Case Study A 65 year old male

- 41 - 41

Yagura H, Hatakenaka M, Miyai I. Does therapeutic facilitation add to locomotor outcome of body weight-supported treadmill training in nonambulatory patients with stroke? A randomized controlled trial. Arch Phys Med Rehabil 2006; 87(4):529-535.

Page 42: C. Motor Recovery pictures removed - EBRSRPR).pdf · - 3 - C1. Assessment of Motor Function Post Stroke C1.1 Assessments of Motor Function Post Stroke Case Study A 65 year old male

- 42 - 42

Key Study: Treadmill Training Post Stroke Pohl M, Mehrholz J, Ritschel C, Ruckriem S. Speed-dependent treadmill training in ambulatory hemiparetic stroke patients. A randomized controlled trial. Stroke 2002;33:553-558. Author / Year Country PEDro score

Methods Outcome

Pohl et al. 2002 Germany 6 (RCT)

60 ambulatory patients suffering from hemiparesis, for at least 4 weeks, caused by right or left supratentorial ischemic stroke or intracerebral hemorrhage, exhibiting impaired gait were randomized into 1 of 3 groups: structured speed-dependent treadmill training (STT) where walking speed was increased with each session; limited progressive treadmill training (LTT) in which training speed was increased by no more than 5% of the maximum initial walking speed each week; and, conventional gait therapy (CGT) involving physiotherapeutic gait therapy based on latest PNF and Bobath concepts. Patients were evaluated before training, 2 and 4 weeks post-training on over ground walking speed, cadence, stride length and FAC.

After 4 weeks training period, the STT group scored significantly higher than the LTT and CGT for all outcome measures.

Speed-dependent treadmill training in ambulatory hemiparetic stroke patients

FAC Score for 3 Rehabilitation Groups at the End of the Study

p<.001

3.5 4 4.5 5 5.5

STT

LTT

CGT

FAC score

Stride Length for 3 Rehabilitation Groups at the End of the Study

p<.001

0 50 100 150

STT

LTT

CGT

Stride Length (m)

Overground Walking Speed for 3 Rehabiltation Groups at the End of

the Study

0 0.5 1 1.5 2

STT

LTT

CGT

Overground Walking Speed (m/s)

Cadence for 3 Rehabilitation Groups at the End of the Study

p<.001

0 0.2 0.4 0.6 0.8

STT

LTT

CGT

Cadence (steps/m)

CGT= Conventional Gait therapy

LTT= Limited Progressive Treadmill Training STT= Speed-dependent Treadmill Training

Page 43: C. Motor Recovery pictures removed - EBRSRPR).pdf · - 3 - C1. Assessment of Motor Function Post Stroke C1.1 Assessments of Motor Function Post Stroke Case Study A 65 year old male

- 43 - 43

Importance: This RCT demonstrated that structured speed-dependent treadmill training, where walking was increased with each session, resulted in significant improvements in many gait-related outcomes when compared to a less aggressive progressive treadmill training program or conventional gait therapy. Relevant SREBR Conclusion: There is conflicting evidence that treadmill training is superior to conventional therapy. Related References Ada L, Dean CM, Hall JM, Bampton J, Cromptom S. A treadmill and overground walking program improves walking in persons residing in the community after stroke: a placebo-controlled, randomized trial. Arch Phys Med Rehabil; 2003;84:1486-91. Dean CM, Richards CL, Malouin F. Task-related circuit training improves performance of locomotor tasks in chronic stroke: a randomized, controlled pilot trial. Arch Phys Med Rehabil 2000; 81:409-417. Hesse S, Bertelt C, Schaffrin A, Malezic M, Mauritz KH. Restoration of gait in nonambulatory hemiparetic patients by treadmill training with partial body-weight support. Arch Phys Med Rehabil 1994;75:1087-1093. Hesse S, Malezic M, Schaffrin A, Mauritz KH. Restoration of gait by combined treadmill training and multichannel electrical stimulation in non-ambulatory hemiparetic patients. Scand J Rehabil Med 1995;27:199-204. Jaffe DL, Brown DA, Pierson-Carey CD, Buckley EL, Lew HL. Stepping over obstacles to improve walking in individuals with poststroke hemiplegia. J Rehabil Res Dev 2004;41(3A):283-292. Laufer Y. Effects of one-point and four-point canes on balance and weight distribution in patients with hemiparesis. Clinical Rehabilitation 2002;16:141-148. Liston R, Mickelborough J, Harris B, Hann AW, Tallis RC. Conventional physiotherapy and treadmill re-training for higher-level gait disorders in cerebrovascular disease. Age Ageing 2000;29:311-318. Macko RF, DeSouza CA, Tretter LD, Silver KH, Smith GV, Anderson PA, Tomoyasu N, Gorman P, Dengel DR. Treadmill aerobic exercise training reduces the energy expenditure and cardiovascular demands of hemiparetic gait in chronic stroke patients. A preliminary report. Stroke 1997;28:326-330. Richards CL, Malouin F, Wood-Dauphinee S, Williams JI, Bouchard JP, Brunet D: Task-specific physical therapy for optimization of gait recovery in acute stroke patients. Arch Phys Med Rehabil 1993;74:612-620. Silver KH, Macko RF, Forrester LW, Goldberg AP, Smith GV. Effects of aerobic treadmill training on gait velocity, cadence, and gait symmetry in chronic hemiparetic stroke: a preliminary report. Neurorehabil Neural Repair 2000;4(1):65-71. Smith GV, Macko RF, Silver KHC, Goldberg AP. Treadmill aerobic exercise improves quadriceps strength in patients wit chronic hemiparesis following stroke: a preliminary report. J Neuro Rehab 1998;12:111-117. Smith GV, Forrester LW, Silver KH, Macko RF. Effects of treadmill training on translational balance perturbation responses in chronic hemiparetic stroke patients. J Stroke Cerebrovascular Diseases 2000;9:238-245.

Page 44: C. Motor Recovery pictures removed - EBRSRPR).pdf · - 3 - C1. Assessment of Motor Function Post Stroke C1.1 Assessments of Motor Function Post Stroke Case Study A 65 year old male

- 44 - 44

Key Study: Partial Body Weight Support and Treadmill Training Visintin M, Barbeau H, Korner-Bitensky N, Mayo NE. A new approach to retrain gait in stroke patients through body weight support and treadmill stimulation. Stroke 1998;29:1122-1128. Author / Year Country PEDro score

Methods Outcome

Visintin et al. 1998 Canada 6 (RCT)

100 patients were randomized to be trained to walk with up to 40% of their body weight supported by a BWS system with overhead harness (BWS group) or to be trained to walk bearing full body weight (no-BWS group). Stroke onset was > 6 months.

Significant effect in favour of BWS group on balance; motor control; overhead walking speed and over-ground endurance at the end of the training period compared to the no-BWS group. At the 3-month follow-up, BWS group was found to have maintained its effect for motor recovery and over-ground walking speed.

Visintin et al. (1998) Comparing Body Weight Support (BWS) with no BWS

0 50 100 150 200

Balance ScaleScore

Motor Recovery(STREAM-Scale

Score)

OvergroundWalking Endurance

(m)

Valu

es a

t Pos

t-Tra

inin

g

BWS no BWS

p = .018

p = .001

p = .018

0 0.1 0.2 0.3 0.4

Overground Walking Speed (m/s)

p = .006

Page 45: C. Motor Recovery pictures removed - EBRSRPR).pdf · - 3 - C1. Assessment of Motor Function Post Stroke C1.1 Assessments of Motor Function Post Stroke Case Study A 65 year old male

- 45 - 45

Importance: This RCT was the key initial study supporting the concept that treadmill training with partial weight support results in improved gait performance. However, the training is very labour intensive and other RCTs as well as a meta-analyses by Moseley et al. (2003) have presented a more mixed picture. Relevant SREBR Conclusion: There is conflicted evidence that the combination of partial body weight support and treadmill training result in improved gait performance when compared to other physiotherapy interventions. Related References Eich HJ, Mach H, Werner C, Hesse S. Aerobic treadmill plus Bobath walking training improves walking in subacute stroke: a randomized controlled trial. Clinical Rehabilitation 2004;18(6):640-651. Moseley AM, Stark A, Cameron ID, et al. Treadmill training and body weight support for walking after stroke. Stroke 2003;34(12):3006. Nilsson L, Carlsson J, Danielsson A, et al. Walking training of patients with hemiparesis at an early stage after stroke: a comparison of walking training on a treadmill with body weight support and walking training on the ground. Clinical Rehabilitation 2001;15(5):515-527. Werner C, von Frankenberg S, Treig T, Konrad M, Hesse S. Treadmill training with partial body weight support and an electromechanical gait trainer for restoration of gait in subacute stroke patients. A randomized crossover study. Stroke 2002;33:2895-2901.

Page 46: C. Motor Recovery pictures removed - EBRSRPR).pdf · - 3 - C1. Assessment of Motor Function Post Stroke C1.1 Assessments of Motor Function Post Stroke Case Study A 65 year old male

- 46 - 46

C8. Functional Electrical Stimulation of Lower Extremity

Page 47: C. Motor Recovery pictures removed - EBRSRPR).pdf · - 3 - C1. Assessment of Motor Function Post Stroke C1.1 Assessments of Motor Function Post Stroke Case Study A 65 year old male

- 47 - 47

C8. Functional Electrical Stimulation in the Lower Extremity Q1. Describe the use of functional electrical stimulation (FES) in the lower extremity. Answer 1. FES of the common peroneal nerve has been used to enhance ankle dorsiflexion during the

swing phase of gait. 2. Although weak ankle dorsiflexion with plantarflexion hypertonicity is typically corrected by an

ankle foot orthosis, FES may be a suitable alternative for highly motivated patients who are able to walk independently or with minimal assistance.

3. Although not widely used or available, there is growing evidence that FES combined with gait training improves hemiplegic gait.

Discussion Functional electrical stimulation (FES) in the lower extremity has been used to enhance ankle dorsiflexion during the swing phase of gait. Weak ankle dorsiflexion with plantarflexion hypertonicity results in a drop foot, which is typically corrected by an ankle foot orthosis (AFO). FES of the common peroneal nerve during the swing phase of gait would appear to be a suitable alternative. Although not widely used or universally available, for highly motivated patients, able to walk independently or with minimal assistance, there is growing evidence that treatment with FES can improve dropped foot, which in turn can improve gait. Both implantable and surface electrodes may be used. A meta-analysis by Glanz et al. (1996) including four RCTs (Bowman et al. 1979, Winchester et al. 1983, Merletti et al. 1978, Levin 1992), reported a favourable treatment effect associated with FES compared to a no treatment control. The effect size associated with a statistically significant change in paretic muscle force of contraction was 0.63 (95% CI 0.29, 0.98), although the clinical significance of this outcome is unclear. There were no other common outcomes among the four included studies. A more recent systematic review (Kottink et al. 2004) also evaluated the effect of FES treatment on gait recovery. This review included eight studies, only one of which was an RCTs, evaluating both implanted and transcutaneous stimulators (Table 9.35). The primary outcomes of this review were self-selected walking speed and the physiological cost index (PCI). Pooled estimates of treatment effect only included point estimates from three studies. The result suggested that FES treatment was associated with a 38% (95% CI, 22%, 54%) improvement in walking speed. Only two studies were included in the evaluation of improvement of PCI and the results were inconclusive. A systematic review by Robbins et al. (2006) evaluated the effect of TENS and FES on gait speed and included the results from 8 studies, 4 of which were RCTs. FES was associated with a significant treatment effect although the effect size was larger in studies, which used multichannel FES compared to single-channel FES. A recent Cochrane review (2006) examined the use of all forms of electrostimulation (ES) in the recovery of functional ability following stroke. This review assessed the efficacy of functional electrical stimulation (both as a form of neuromuscular retraining and as a form of neuroprosthesis/orthosis), transculaneous electrical nerve stimulation, EMG and electroacupuncture. Twenty-four RCTs evaluating the efficacy of treatment on both the upper and lower extremities were included. Only three outcomes yielded

Page 48: C. Motor Recovery pictures removed - EBRSRPR).pdf · - 3 - C1. Assessment of Motor Function Post Stroke C1.1 Assessments of Motor Function Post Stroke Case Study A 65 year old male

- 48 - 48

a benefit of treatment that reached statistical significance: i) ES improved motor impairment active joint range of movement in the lower limb, compared to no stimulation; ii) improved co-contraction of agonist and antagonist muscles for the comparison of ES compared with placebo and improved Fugl-Meyer scores for the comparison of ES vs. conventional therapy. The authors concluded that further research is required to confirm a benefit of treatment. Generalizations of the effectiveness of the treatment are difficult to make due to variations in the type of stimulation (single-channel vs. multichannel units), intensity of treatment, patient acceptability and compliance, additional treatments provided (ie. routine physiotherapy) as well as the timing and choice of outcome measurement. Cozean et al. (1988) found that FES when combined with biofeedback produced better results than standard physical therapy or FES or biofeedback alone. MacDonnel et al. (1994), Yan et al. (2004) and Daly et al. (2004) found that FES when combined with physiotherapy was superior to physiotherapy alone in improving certain elements of ambulation. Daly et al. (2006) evaluated the effect of intramuscular functional neuromuscular stimulation when combined with overground walking training and body-weight supported treadmill training. However, electrical stimulation did not result in additional benefits of cycling training. Three studies examined the benefit of using the Odstock Dropped Foot Stimulator (ODFS) (Burridge et al. 1997, Taylor et al. 1999, Sheffler et al. 2006). Burridge et al. (1997) found that FES when combined with physiotherapy was superior to physiotherapy alone in significant improving gait speed while reducing energy costs; however, the benefit was only evident when the stimulator was being used. There was no carryover effect. More recently, Sheffler et al. (2006) reported that a traditional AFO was most effective in improving walking performance compared with either no device or the ODFS. The authors speculated that patients likely needed a longer period of time to become accustomed to the ODFS in order to realize a benefit of treatment. Kottnik et al. (2007) also reported that peroneal nerve stimulation was found to improve gait speed. Summary of RCTs Examining FES in Lower Extremity

Author, Year PEDro Score n Outcome

Daly et al. 2006 8 32 + Tinetti gait scale

Yavuzer et al. 2007 7 30 - Gait kinematics, Brunnstrom scores

Cozean et al. 1988 6 36 + BFB + FES group for indexes of knee flexion & dorisflexion, stride length and gait cycle time

Bogataj et al. 1995 6 20 + MFES for all outcome measures

Daly et al. 2004 6 16 + Gait kinematics

Yan et al. 2005 6 46 + Discharge home following inpatient rehabilitation

MacDonnel et al. 1994 5 35 + Functional Ambulation Scores

Kottnik et al. 2008 5 29 +

Page 49: C. Motor Recovery pictures removed - EBRSRPR).pdf · - 3 - C1. Assessment of Motor Function Post Stroke C1.1 Assessments of Motor Function Post Stroke Case Study A 65 year old male

- 49 - 49

Walking speed Newsam & Baker 2004 5 20 +

Motor unit recruitment Burridge et al. 1997 5 32 +

FES with use of stimulator in Physiological Cost Index Yavuzer et al. 2006 5 25 -

Gait kinematics, Brunnstrom scores Chen et al. 2005 4 24 +

Walking speed, lower limb spasticity Heckermann et al. 1997

4 28 Ankle range of motion (+)

Tong et al. 2006 (RCT)

4 46 Five-Meter Walking Speed Test (+) Berg Balance Scale (-)

Elderly Mobility Scale (+) Barthel Index (-)

Motricity Index Leg Subscale (+) FIM Instrument Score (-)

Functional Ambulatory Category (+)

The SREBR (11th edition) noted that there is strong evidence that FES and gait retraining results in improvements in hemiplegic gait (Teasell et al. 2008).

References Bogataj U, Gros N, Kljajic M, Acimovic R, Malezic M. The rehabilitation of gait in patients with hemiplegia: a comparison between conventional therapy and multichannel functional electrical stimulation therapy. Phys Ther 1995; 75:490-502. Bowman BR, Baker LL, Waters RL. Positional feedback and electrical stimulation: an automated treatment for the hemiplegic wrist. Arch Phys Med Rehabil 1979; 60:497–502. Burridge JH, Taylor PN, Hagan SA, Wood DE, Swain ID. The effects of common peroneal stimulation on the effort and speed of walking: a randomized controlled trial with chronic hemiplegic patients. Clin Rehabil 1997; 11:201-210. Chen SC, Chen YL, Chen CJ, Lai CH, Chiang WH, Chen WL. Effects of surface electrical stimulation on the muscle-tendon junction of spastic gastrocnemius in stroke patients. Disabil Rehabil 2005; 27(3):105-110. Cozean CD, Pease WS, Hubbell SL. Biofeedback and functional electric stimulation in stroke rehabilitation. Arch Phys Med Rehabil 1988; 69:401-405. Daly JJ, Roenigk KL, Butler KM, et al. Response of sagittal plane gait kinematics to weight supported treadmill training and functional neuromuscular stimulation following stroke. J Rehabil Res Dev 2004;41:807-820. Daly JJ, Roenigk K, Holcomb J, Rogers JM, Butler K, Gansen J, McCabe J, Frederickson E, Marsolais EB, Ruff RL. A Randomized Controlled Trial of Functional Neuromuscular Stimulation in Chronic Stroke Subjects. Stroke 2006; 37:172-178. Glanz M, Klawansky S, Stason W, Berkey C, Chalmers TC. Functional electrostimulation in

Page 50: C. Motor Recovery pictures removed - EBRSRPR).pdf · - 3 - C1. Assessment of Motor Function Post Stroke C1.1 Assessments of Motor Function Post Stroke Case Study A 65 year old male

- 50 - 50

poststroke rehabilitation: a meta-analysis of the randomized controlled trials. Arch Phys Med Rehabil 1996; 77:549-553. Heckmann J, Mokrusch T, Krockel A, Warnke S, von Stockert T, Neundorfer B. EMGtriggered electrical muscle stimulation in the treatment of central hemiparesis after a stroke. Eur J Phys Med Rehabil 1997; 7(5):138-141. Kottink AI, Hermens HJ, Nene AV et al. Therapeutic effect of an implantable peroneal nerve stimulator in subjects with chronic stroke and footdrop: a randomized controlled trial. Physical Therapy 2008; 88(4):437-448. Kottink AI, Hermens HJ, Nene AV, et al. A randomized controlled trial of an implantable 2-channel peroneal nerve stimulator on walking speed and activity in poststroke hemiplegia. Arch Phys Med Rehabil 2007; 88:971-978. Kottink AI, Oostendorp LJ, Buurke JH, Nene AV, Hermens HJ, IJzerman MJ. The orthotic effect of functional electrical stimulation on the improvement of walking in stroke patients with a dropped foot: a systematic review. Artif Organs 2004; 28:577-586. Levin MF, Hui-Chan CW. Relief of hemiparetic spasticity by TENS is associated with improvement in reflex and voluntary motor functions. Electroencephalogr Clin Neurophysiol 1992; 85:131-142. MacDonell RA, Triggs WJ, Leikauskas J, Bourque M, Robb K, Day BJ, Shahani BT. Functional electrical stimulation to the affected lower limb and recovery after cerebral infarction. Cerebrovasc Dis 1994; 4:155-160. Merletti R, Zelaschi F, Latella D, et al. Control study of muscle force recovery in hemiparetic patients during treatment with functional electrical-stimulation. Scandinavian Journal of Rehabilitation Medicine 1978; 10(3):147-154. Newsam CJ, Baker LL. Effect of an electric stimulation facilitation program on quadriceps motor unit recruitment after stroke. Arch Phys Med Rehabil 2004; 85:2040-2045. Robbins SM, Houghton PE, Woodbury MG, Brown JL. The therapeutic effect of functional and transcutaneous electric stimulation on improving gait speed in stroke patients: a meta-analysis. Arch Phys Med Rehabil 2006; 87(6):853-859. Sheffler LR, Hennessey MT, Naples GG, Chae J. Peroneal nerve stimulation versus an ankle foot orthosis for correction of footdrop in stroke: impact on functional ambulation. Neurorehabil Neural Repair 2006; 20:355-360. Taylor PN, Burridge JH, Dunkerley AL, Wood DE, Norton JA, Singleton C, Swain ID. Clinical use of the Odstock dropped foot stimulator: its effect on the speed and effort of walking. Arch Phys Med Rehabil 1999; 80:1577-1583. Teasell RW, Foley NC, Salter K, Bhogal SK, Jutai J, Speechley MR. Evidence-Based Review of Stroke Rehabilitation (11th edition). Canadian Stroke Network; 2008. Tong RK, Ng MF, Li LS. Effectiveness of gait training using an electromechanical gait trainer, with and without functional electric stimulation, in subacute stroke: a randomized controlled trial. Arch Phys Med Rehabil 2006; 87:1298-1304. Winchester P, Montgomery J, Bowman B, Hislop H. Effects of feedback stimulation training and cyclical electrical stimulation on knee extension in hemiparetic patients. Phys Ther 1983; 63:1096-1103.

Page 51: C. Motor Recovery pictures removed - EBRSRPR).pdf · - 3 - C1. Assessment of Motor Function Post Stroke C1.1 Assessments of Motor Function Post Stroke Case Study A 65 year old male

- 51 - 51

Yan T, Hui-Chan CW, Li LS. Functional electrical stimulation improves motor recovery of the lower extremity and walking ability of subjects with first acute stroke: a randomized placebo-controlled trial. Stroke 2005;36:80-85. Yan T, Hui-Chan CWY. Transcutaneous electrical never stimulation (TENS) improves motor recovery and walking ability of subjects with acute stroke: a randomized controlled trial. Stroke 2004; 35(6):E320. Yavuzer G, Geler-Külcü D, Sonel-Tur B, Kutlay S, Ergin S, Stam HJ. Neuromuscular Electric Stimulation Effect on Lower-Extremity Motor Recovery and Gait Kinematics of Patients With Stroke:A Randomized Controlled Trial. Arch Phys Med Rehabil 2006; 87:536-540. Yavuzer G, Oken O, Atay MB, Stam HJ. Effect of sensory-amplitude electric stimulation on motor recovery and gait kinematics after stroke: a randomized controlled study. Arch Phys Med Rehabil 2007; 88:710-714.

Page 52: C. Motor Recovery pictures removed - EBRSRPR).pdf · - 3 - C1. Assessment of Motor Function Post Stroke C1.1 Assessments of Motor Function Post Stroke Case Study A 65 year old male

- 52 - 52

C9. Rehabilitation of the Upper Extremity

Page 53: C. Motor Recovery pictures removed - EBRSRPR).pdf · - 3 - C1. Assessment of Motor Function Post Stroke C1.1 Assessments of Motor Function Post Stroke Case Study A 65 year old male

- 53 - 53

C9. Rehabiliation of the Upper Extremity C9.1 Motor Recovery in Severe Stroke Nakayama et al. (1994) reported that of stroke patients with severe arm paresis, with little or no movement at hospital admission, 14% make a complete motor recovery while 30% make a partial recovery. The odds of recovery for this patient, with no recovery at 14 days post-stroke, are less than that reported by Nakayama. Q1. For the severely affected hemiplegic upper extremity should the therapists continue to treat the upper extremity with a goal of maximizing recovery or palliate the upper extremity, minimizing contractures and pain? Answers 1. Barreca et al. (2001) recommended patients with a Chedoke-MacMaster score of less than

4 have a poor prognosis – should focus on palliation with minimizing contractures and pain. Discussion Reporting on an evidence-based consensus panel, Barreca et al. (2001) recommended that patients with a poor prognosis (defined as a Chedoke-MacMaster Score of less than stage 4) should receive treatment that focuses on minimizing contractures and pain. Barreca et al. (2001) noted that, “For the client with severe motor, sensory and functional deficits in the involved limb after stroke, the effectiveness literature indicates that additional treatment for the upper limb will not result in any significant neurological change. The evidence to date suggests that interventions may not lead to meaningful functional use of the affected limb at this stage of motor recovery.” Therefore, based on the consensus panel report, it is recommended that the therapist should focus on maintaining a comfortable, pain-free, mobile arm and hand. To maximize functional independence, stroke survivors with persistent motor and sensory deficits and their caregivers should be taught compensatory techniques and environmental adaptations that enable performance of important tasks and activities with the less affected arm and hand. References Barreca S. et al. Management of the Post Stroke Hemiplegic Arm and Hand: Treatment Recommendations of the 2001 Consensus Panel. Heart and Stroke Foundation of Ontario, 2001. Nakayama H, Jorgensen HS, Raaschou HO, Olsen TS. Recovery of upper extremity function in stroke patients: the Copenhagen Stroke Study. Arch Phys Med Rehabil 1994;75:394-398.

Page 54: C. Motor Recovery pictures removed - EBRSRPR).pdf · - 3 - C1. Assessment of Motor Function Post Stroke C1.1 Assessments of Motor Function Post Stroke Case Study A 65 year old male

- 54 - 54

C10. Constraint-Induced Movement Therapy

Page 55: C. Motor Recovery pictures removed - EBRSRPR).pdf · - 3 - C1. Assessment of Motor Function Post Stroke C1.1 Assessments of Motor Function Post Stroke Case Study A 65 year old male

- 55 - 55

C10. Constraint-Induced Movement Therapy Case Study

Q1. Describe CIMT and the criteria necessary for him to be considered for CIMT. Answers 1. CIMT is designed to reduce functional deficits in the more affected upper extremity. 2. The key features of CIMT are restraint of the unaffected hand/arm and increased

practice/use of the affected hand/arm. 3. It is designed to overcome learned non-use by promoting cortical reorganization (Taub et al.

1999). 4. CIMT works best for patients with active wrist and hand movements. Q2. What is the evidence for CIMT post-stroke for the upper extremity? 1. Based on the results from two RCTs, there is conflicting evidence of benefit of CIMT in

comparison to traditional therapies in the acute stage of stroke. 2. Based on the results from 12 RCTs, there is strong evidence of benefit of CIMT in

comparison to traditional therapies in the chronic stage of stroke. Benefits appear to be confined to stroke patients with some active wrist and hand movements, particularly those with sensory loss and neglect.

A 44-year-old man suffered a significant intracerebral hemorrhage with a right hemiplegia and right sensory loss to the point of the right arm having no sensation. He experienced a significant motor recovery in his right upper extremity including the hand but failed to use his right hand (he was right hand dominant) because of the significant and nearly complete sensory loss. Six weeks following the onset of his stroke, the uninvolved arm was subsequently restrained, initially for 4 hours in the mornings and progressing over the next week to 8-10 hours per day. Upon restraining the uninvolved arm and continuing with rehabilitation therapies he exhibited a dramatic recovery in the use of his right arm and hand. The upper extremity function test improved from 31/99 to 70/99 over 3.5 weeks. The 9-hole modified peg test improved from 210.3 seconds to 38.6 seconds over 3 weeks. Grip strength improved from 11 to 19 kg while lateral pinch strength improved from 4.0 to 6.0 kg over 2 weeks. Chedoke-McMaster Stroke Assessement Score for Motor Recovery did not improve over the course of treatment (6/7).

Page 56: C. Motor Recovery pictures removed - EBRSRPR).pdf · - 3 - C1. Assessment of Motor Function Post Stroke C1.1 Assessments of Motor Function Post Stroke Case Study A 65 year old male

- 56 - 56

3. The most compelling evidence in favour of a benefit of CIMT comes from the EXCITE trial (Wolf et al. 2006), an RCT that assigned 222 patients who were between 3 and 9 months post stroke to receive either CIMT (n=106) or usual care (no treatment, home care or outpatient programs) (n=116). The CIMT group wore a mitt on the less-affected hand while performing repetitive task practice and behavioural shaping with the hemiplegic hand. The CIMT group significantly improved on the Wolf Motor Function Test (log performance time, functional ability 0-5 scale (p<0.001), the Motor Activity Log (MAL) Amount of Use (p<0.001) and the MAL Quality of Movement (p<0.001) and caregiver MAL (Wolf et al. 2006).

Discussion Constraint-Induced Movement Therapy (CIMT) refers to a new set of rehabilitation techniques designed to reduce functional deficits in the more affected upper extremity of stroke survivors. The two key features of CIMT are restraint of the unaffected hand/arm and increased practice /use of the affected hand/arm (Fritz et al. 2005). Since stroke survivors may experience “learned non-use” of the upper extremity within a short period of time (Taub 1980), CIMT is designed to overcome learned non-use by promoting cortical reorganization (Taub et al. 1999). While the biological mechanism(s) responsible for the benefit are unknown and the contribution from intense practice is difficult to disassociate from the contribution of constraining the good limb, this form of treatment shows promise, especially for survivors with moderate disability following stroke. Several reviews have been published on the effectiveness of CIMT (Taub & Morris, 2001, Barreca et 2003, Hakkennes & Keating 2005, Bonaiuti et al. 2007) and while the results have been generally positive, uncertainty of its effectiveness remain due to the small number of trials published, the small sample sizes of the studies and heterogeneous patient characteristics, duration and intensity of treatment, and outcomes assessed. A meta-analysis conducted by Van Peppen et al. (2004) concluded that CIT was associated with improvements in dexterity, measured by the Arm Motor Activity Test or the Action Research Arm test, but not in performance of ADL, measured by FIM or Barthel Index scores. A more recent review by Hakkenes and Keating (2006) included the results from 14 RCTs and concluded that there was a benefit associated with treatment although larger well-designed studies are still required. Several treatment contrasts were examined including traditional CIT vs. alternative therapy or control, modified CIT vs. alternative therapy or control and traditional CIT vs. modified CIT, although pooled estimates of the treatment sizes for the subgroups were not provided. The associated pooled effect sizes for all of the included RCTs were: Action Research Arm test 1.51 (95% CI 0.27, 2.74), Fugl Meyer Assessment 1.16 (95% CI –0.18, 2.52) and the Wolf Motor Function Test 0.50 (95% CI –0.28, 1.27). Taub et al. (2003) noted that constraint-induced movement therapy has limitations in that the improvement seen does not restore the stroke patients’ movement to their motor status prior to the stroke. The same authors note that constraint-induced movement therapy “produces a variable outcome that depends on the severity of initial impairment. If patients with residual motor function are categorized on the basis of their active range of motion, the higher functioning individuals tend to improve more than persons who are more disabled (Taub et al. 1999)…. For patients with the lowest motor functioning, constraint-induced therapy does improve movement at the shoulder and elbows. Because these people have little or no ability to move the fingers, there is no adequate motor basis for carrying out training of hand function. Consequently, because most daily activities that are carried out by the upper extremity are

Page 57: C. Motor Recovery pictures removed - EBRSRPR).pdf · - 3 - C1. Assessment of Motor Function Post Stroke C1.1 Assessments of Motor Function Post Stroke Case Study A 65 year old male

- 57 - 57

performed by the hand, there is relatively little translation of the therapy induced movement in proximal joint function into an increase in the actual amount of use of the more affected extremity in the real life situation… Thus, constraint-induced therapy is clearly not a complete answer to motor deficits after stroke. The work so far does show that motor function in a large percentage of patients with chronic stroke is substantially modifiable” (Taub et al. 2003). Van der Lee (2001) suggests that the positive results attributed to CIMT may simply reflect a greater intensity of training of the affected arm and questions the concept of non-use implying that it may not be a distinct entity, but rather the result of sensory disorders or hemineglect. According to Dromerick et al. (2000), constraint of the unaffected arm by use of a mitten (6 hours per day for 14 days), and ‘forced use’ of the affected arm soon after stroke (mean six days), is feasible. However, trials reporting small, but significant reductions in arm impairment, especially for patients with sensory disorders and hemi-neglect (van der Lee et al. 1999, Ploughman & Corbett 2004), have also reported a high number of deviations from the randomized treatment schedule, due to patients’ non compliance. This has led to trials investigating the effectiveness of modified or shorter periods of constraint induced therapy treatment. There is promising evidence that the drawbacks to stroke patient participation in CIMT (i.e., required practice intensity and duration of restraint) may be overcome through modifications to the basic procedures. They include a less intense, modified CIMT (mCIMT) that combines structured functional practice sessions with restricted use of the less affected upper limb (Page et al., 2004), and also forced-use therapy (FUT) which employs constraint without intensive training of the affected arm (“shaping”) (Ploughman & Corbett, 2004). Page et al. (2002, 2004 and 2005) provide one example of the distinction between CIMT and mCIMT: CIMT is defined by the i) restriction of a patient’s less affected upper-limb during up to 90% of waking hours during a 2-week period and ii) participation in an intensive upper-extremity therapy program for 6 hours per day, using the affected limb during the same 2-week period. In contrast, mCIMT involves the restriction of the unaffected limb for periods of 5 hours a day, 5 days a week for 2 weeks combined with structured, ½ hour therapy sessions, 3 days a week. However, other criteria for defining mCIMT have also been used, which overlap with CIMT, blurring the distinction. Lin et al. (2007) cite mCIMT as providing 2 hours of therapy a day for 10-15 consecutive weekdays + restraint for 6 hours per day. There are also examples of trials, presented in the following tables, which provided the intervention for periods of up to 10 weeks. There is some evidence, too, for the beneficial effect of donepezil, a primarily central acetylcholinesterase inhibitor, as an adjuvant therapy (Nadeau et al., 2004; Richards et al. 2006). Taub et al. (2005) recently reported that the benefits associated with CIMT could be achieved with the use of an automated device (AutoCITE). The optimal timing of treatment remains uncertain. While there is evidence that patients treated in the acute phase of stroke may benefit preferentially (Taub & Morris 2001), Grotta et al. (2004) suggest that the greatest benefit is likely to be conferred during the chronic stages of stroke and that the treatment has been shown to be harmful in animal studies of “forced use” immediately post stroke. The results from the largest and most rigorously conducted trial-The Extremity Constraint Induced Therapy Evaluation (EXCITE), may provide the strongest evidence of a benefit of CIMT treatment, to date. The study recruited 222 subjects with moderate disability 3 to 9 months following stroke, over 3 years from 7 institutions in the US. Treatment was provided for up to 6 hours a day, 5 days a week for 2 weeks. Patients were reassessed up to 24 months following

Page 58: C. Motor Recovery pictures removed - EBRSRPR).pdf · - 3 - C1. Assessment of Motor Function Post Stroke C1.1 Assessments of Motor Function Post Stroke Case Study A 65 year old male

- 58 - 58

treatment. At 12 months, compared with the control group who received usual care, subjects in the treatment group had significantly higher scores on sections of the Wolf Motor Function test and the Motor Activity Log. At 24 months these gains were maintained. While these results are encouraging, as Cramer (2007) points out, the number of patients for whom this treatment may be suitable for, remains uncertain. In the EXCITE trial, only 6.3% of patients screened were eligible. While larger estimates of 20-25% have been suggested, it remains uncertain if subjects with greater disability would benefit from treatment. The SREBR (11th edition) noted that there is conflicting evidence of benefit of CIMT in comparison to traditional therapies in the acute stage of stroke. There is strong evidence of benefit of CIMT and mCIMT in comparison to traditional therapies in the chronic stage of stroke. Benefits appear to be confined to stroke patients with some active wrist and hand movements, particularly those with sensory loss and neglect (Teasell et al. 2008). References Barreca S, Wolf S L, Fasoli S, Bohannon R. Treatment interventions for the paretic upper limb of stroke survivors: a critical review. Neurorehabil Neural Repair 2003; 17(4):220-226. Bonaiuti D, Rebasti L, Sioli P. The constraint induced movement therapy: a systematic review of randomised controlled trials on the adult stroke patients. Eura Medicophys 2007; 43:139-146. Cramer SC. The EXCITE trial: a major step forward for restorative therapies in stroke. Stroke 2007; 38:2204-2205. Dromerick AW, Edwards DF, Hahn M. Does the application of constraint-induced movement therapy during acute rehabilitation reduce arm impairment after ischemic stroke? Stroke 2000; 31:2984-2988. Fritz SL, Light KE, Patterson TS, Behrman AL, Davis SB. Active finger extension predicts outcomes after constraint-induced movement therapy for individuals with hemiparesis after stroke. Stroke 2005;36:1172-1177. Grotta JC, Noser EA, Ro T, et al. Constraintinduced movement therapy. Stroke 2004; 35:2699-2701. Hakkenes S, Keating JL. Constraint-Induced movement therapy following stroke systematic review of randomised controlled trials. Australian Journal of Physiotherapy 2005; 51:221-231. Lin KC, Wu CY, Wei TH, Lee CY, Liu JS. Effects of modified constraint-induced movement therapy on reach-to-grasp movements and functional performance after chronic stroke: a randomized controlled study. Clin Rehabil 2007; 21:1075-1086. Nadeau SE, Behrman AL, Davis SE, Reid K, Wu SS, Stidham RN, Helms KM, Gonzalez LJ. Donepezil as an adjuvant to constraintinduced therapy for upper-limb dysfunction after stroke: an exploratory randomized clinical trial. 2004; 41:525-534. Page SJ, Sisto S, Johnston MV, Levine P. Modified constraint-induced therapy after subacute stroke: a preliminary study. Neurorehabil Neural Repair 2002; 16(3):290- 295. Page SJ, Sisto S, Levine P, McGrath RE. Efficacy of modified constraint-induced movement therapy in chronic stroke: a single-blinded randomized controlled trial. Arch Phys Med Rehabil. 2004; 85(1):14-8. Page SJ, Levine P, Leonard AC. Modified constraint-induced therapy in acute stroke: a randomized controlled pilot study. Neurorehabil Neural Repair 2005; 19:27-32.

Page 59: C. Motor Recovery pictures removed - EBRSRPR).pdf · - 3 - C1. Assessment of Motor Function Post Stroke C1.1 Assessments of Motor Function Post Stroke Case Study A 65 year old male

- 59 - 59

Ploughman M, Corbett D. Can forced-use therapy be clinically applied after stroke? An exploratory randomized controlled trial. Arch Phys Med Rehabil. 2004; 85(9):1417-23. Richards L, Rothi LJG, Davis S, Wu AA, Nadeau SE. Limited dose response to Constraint-Induced Movement therapy in patients with chronic stroke. Clin Rehabil 2006; 20:1066- 1074. Taub E. Somatosensory deafferentation research with monkeys: Implications for rehabilitation medicine. In: LP Ince (Ed.), Behavioral Psychology in Rehabilitation Medicine: Clinical Applications (pp.371-401). Baltimore: Williams and Wilkins, 1980. Taub E, Uswatte G, Pidikiti R. Constraint-Induced Movement Therapy: a new family of techniques with broad application to physical rehabilitation--a clinical review. J Rehabil Res Dev 1999;36(3):237-51. Taub E, Morris DM. Constraint-induced movement therapy to enhance recovery after stroke. Curr Atheroscler Rep 2001; 3(4):279-86. Taub E, Uswatte G, Morris DM. Improved motor recovery after stroke and massive cortical reorganization following Constraint-Induced Movement therapy. Phys Med Rehabil Clin N Am 2003; 14(1 Suppl):S77-91, ix. Taub E, Lum PS, Hardin P, Mark VW, Uswatte G. AutoCITE: automated delivery of CI therapy with reduced effort by therapists. Stroke 2005; 36:1301-1304. Teasell RW, Foley NC, Salter K, Bhogal SK, Jutai J, Speechley MR. Evidence-Based Review of Stroke Rehabilitation (11th edition). Canadian Stroke Network; 2008. Van Peppen RP, Kwakkel G, Wood-Dauphinee S, Hendriks HJ, Van der Wees PJ, Dekker J. The impact of physical therapy on functional outcomes after stroke: what's the evidence? Clin Rehabil 2004; 18:833-862. van der Lee JH, Wagenaar RC, Lankhorst GJ, Vogelaar TW, Deville WL, Bouter LM. Forced use of the upper extremity in chronic stroke patients: results from a single-blind randomized clinical trial. Stroke 1999; 30:2369-2375. van der Lee JH. Constraint-induced therapy for stroke: more of the same or something completely different? Curr Opin Neurol 2001; 14:741-744. Wolf SL, Winstein CJ, Miller JP, Taub E, Uswatte G, Morris D, Giuliani C, Light KE, Nichols-Larsen D. Effect of Constraint-Induced Movement Therapy on Upper Extremity Function 3 to 9 months after stroke. JAMA 2006; 296: 2095-2104.

Page 60: C. Motor Recovery pictures removed - EBRSRPR).pdf · - 3 - C1. Assessment of Motor Function Post Stroke C1.1 Assessments of Motor Function Post Stroke Case Study A 65 year old male

- 60 - 60

Key Study: Constraint-Induced Movement Therapy Taub E, Miller NE, Novack TA, Cook EW, Fleming WC, Nepomuceno CS, Connell JS, Crago JE. Technique to improve chronic motor deficit after stroke. Arch Phys Med Rehabil 1993;74:347-354 Author / Year Country PEDro score

Methods Outcome

Taub et al. 1993 USA 6 (RCT)

9 patients randomized to either have their unaffected upper extremity restrained in a sling during waking hours for 14 days with 10 of those 10 days patients given 6 hours of practice in using impaired upper extremity or to receive several procedures designed to focus attention on use of the impaired upper extremity (control).

Restraint group showed significantly greater improvement in quality of movement and functional ability compared to control on Emory Test and the Arm Motor Activity Rest test at the end of treatment. Motor Activity Log indicates that the restraint group showed a marked increase in their ability to use their affected upper extremity. Gains made during treatment period were maintained during 2 year follow up.

Importance: Constraint-Induced Movement Therapy (CIMT) involves restraint of the unaffected hand/arm and increased practice/use of the affected hand arm (Fritz et al. 2005). Despite being a median of over 4 years post-stroke, the treatment group showed a marked increase in their upper extremity use. Relevant SREBR Conclusions: There is strong evidence of the benefit of CIMT in comparison to traditional therapies in the chronic stage of stroke. Benefits appear to be confined to stroke patients with some active wrist and hand movements, particularly those with sensory loss and neglect. Related References Page SJ, Sisto SA, Levine P. Modified constraint-induced therapy in chronic stroke. Am J Phys Med Rehabil 2002; 81(11):870-875. Page SJ, Sisto SA, Levine P, et al. Efficacy of modified constraint-induced movement therapy in chronic stroke: A single-blinded randomized controlled trial. Arch Phys Med Rehabil 2004;85(1):14-18.

Page 61: C. Motor Recovery pictures removed - EBRSRPR).pdf · - 3 - C1. Assessment of Motor Function Post Stroke C1.1 Assessments of Motor Function Post Stroke Case Study A 65 year old male

- 61 - 61

Key Study: Constraint-Induced Movement Therapy Van der Lee JH, Wagenaar RC, Lankhorst GJ, Vogelaar TW, Deville WL, Bouter LM. Forced use of the upper extremity in chronic stroke patients: results from a single-blind randomized clinical trial. Stroke 1999;30:2369-2375. Author / Year Country PEDro score

Methods Outcome

van der Lee et al. 1999 Netherlands 7 (RCT)

In an observer blind trial, 66 patients were randomized to receive either forced use therapy with immobilization of the unaffected arm combined with intensive treatment or to receive intensive bimanual training based on Neuro-Development Treatment.

Mean improvement on Action Research Arm in patients with sensory disorder was significantly greater in those receiving forced use rather than bimanual training. During treatment, forced use patients also showed greater clinical significant improvement on Motor Activity Log than bimanual training patients.

Action Research Arm (ARA) for Forced Use vs. Bimanual

33.4

39.2 38 38.5

28.330 30.8 30.7

0

5

10

15

20

25

30

35

40

45

0 weeks 3 weeks 6 weeks 1 year

ARA

valu

e (ra

nge

0-57

poi

nts)

Forced Use Bimanual

p=.001

Importance: This RCT examined Constraint Induced Movement Therapy (CIMT) and intensive therapy and compared it to intensive bimanual training based on NDT in chronic stroke patients. CIMT-treated patients showed significantly greater improvement. Relevant SREBR Conclusions: There is strong evidence of the benefit of CIMT in comparison to traditional therapies in the chronic stage of stroke. Benefits appear to be confined to stroke patients with some active wrist and hand movements, particularly those with sensory loss and neglect. Related References Basmajian JV, Gowland CA, Finlayson MA, Hall AL, Swanson LR, Stratford PW, Trotter JE, Brandstater ME. Stroke treatment: comparison of integrated behavioral-physical therapy vs traditional physical therapy programs. Arch Phys Med Rehabil 1987;68(5 Pt 1):267-272.

Page 62: C. Motor Recovery pictures removed - EBRSRPR).pdf · - 3 - C1. Assessment of Motor Function Post Stroke C1.1 Assessments of Motor Function Post Stroke Case Study A 65 year old male

- 62 - 62

Dickstein R, Hocherman S, Pillar T, Shaham R. Stroke rehabilitation. Three exercise therapy approaches. Phys Ther 1986; 66(8):1233-1238. Gelber DA. Comparison of two therapy approaches in the rehabilitation of the pure motor hemiparetic stroke patient. J Neurol Rehabil 1995,9:191-196.

Langhammer B, Stanghelle JK. Bobath or motor relearning programme? A comparison of two different approaches of physiotherapy in stroke rehabilitation: a randomized controlled study. Clin Rehabil 2000;14(4):361-9. Langhammer B, Stanghelle JK. Bobath or motor relearning programme? A follow-up one and four years post stroke. Clin Rehabil 2003;17(7):731-734. Logigian MK, Samuels MA, Falconer J, Zagar R. Clinical exercise trial for stroke patients. Arch Phys Med Rehabil 1983;64(8):364-367. Lord JP, Hall K. Neuromuscular reeducation versus traditional programs for stroke rehabilitation. Arch Phys Med Rehabil 1986;67(2):88-91. Platz T, Eickhof C, van Kaick S, et al. Impairment-oriented training or Bobath therapy for severe arm paresis after stroke: a single-blind, multicentre randomized controlled trial. Clin Rehabil 2005;19:714-724. van Vliet PM, Lincoln NB, Foxall A. Comparison of Bobath based and movement science based treatment for stroke: a randomised controlled trial. J Neurol Neurosurg Psychiatry 2005;76:503-508.

Wagenaar RC, Meijer OG, van Wieringen PC, Kuik DJ, Hazenberg GJ, Lindeboom J, Wichers F, Rijswijk H. The functional recovery of stroke: a comparison between neuro-developmental treatment and the Brunnstrom method. Scand J Rehabil Med 1990;22(1):1-8.

Page 63: C. Motor Recovery pictures removed - EBRSRPR).pdf · - 3 - C1. Assessment of Motor Function Post Stroke C1.1 Assessments of Motor Function Post Stroke Case Study A 65 year old male

- 63 - 63

C11. Mental Practice

Page 64: C. Motor Recovery pictures removed - EBRSRPR).pdf · - 3 - C1. Assessment of Motor Function Post Stroke C1.1 Assessments of Motor Function Post Stroke Case Study A 65 year old male

- 64 - 64

C11. Mental Practice Q1. What is mental practice and what is the evidence that it is useful in post-stroke recovery? Answers 1. Mental imagery involves rehearsing a specific task or series of tasks mentally before actually

performing the task. 2. The theory is stored motor plans for executing movements can be accessed and reinforced

during mental practice 3. There is strong evidence that mental practice can improve upper extremity function and

ADLs following stroke. Discussion The use of mental imagery or mental practice as a means to enhance performance following stroke was adapted from the field of sports psychology were the technique has been shown to improve athletic performance, when used as an adjunct to standard training methods. The technique, as the name suggests, involves rehearsing a specific task or series of tasks, mentally. A series of small trials have adapted and evaluated the effects of mental practice as a treatment following stroke. The ability of the treatment to improve motor function or ADL performance is the outcome most frequently assessed in these studies. The most plausible mechanism to explain the success of the technique is that stored motor plans for executing movements can be accessed and reinforced during mental practice (Page et al. 2001). Mental practice can be used to supplement conventional therapy and can be used at any stage of recovery. A systematic review (Braun et al. 2006) included the results from 10 studies, three of which were RCTs. Sample sizes ranged from 1 to 46. The patient characteristics, interventions and outcomes assessed of the included studies were sufficiently heterogeneous to preclude pooled analysis. The authors were unable to draw conclusions based on the available evidence and suggested more research is needed. More recently, Zimmerman-Schlatter et al. (2008) also assessed the efficacy of motor imagery in recovery post stroke. Theses authors included the results from only 4 RCTs (Liu et al. 2004, Page et al. 2001, 2006 2007) in which the duration and frequency of treatment lasted from 10 minutes to one-hour a day, with 3 to 5 sessions per week for 3 to 6 weeks. Mean time of stroke onset ranged from several days to several years. Three of these studies reported improvements in the mean Action Research Arm Test and Fugl-Meyer scores. Two of these studies also found higher mean change scores than the minimally clinically relevant difference in the ARAT and FM scores. These authors concluded that although there was evidence of benefit of treatment, larger and more rigorous studies are required to confirm these findings. A study protocol which aims to evaluate the effectiveness and cost-effectiveness of this treatment has recently been published (Verbunt et al. 2008). Page et al. (2000, 2001, 2005, 2007) used a tape-recorded (guided) imagery intervention to enable mental practice, whereby patients would sit in a room quietly and listen to a male voice

Page 65: C. Motor Recovery pictures removed - EBRSRPR).pdf · - 3 - C1. Assessment of Motor Function Post Stroke C1.1 Assessments of Motor Function Post Stroke Case Study A 65 year old male

- 65 - 65

encouraging them to first relax (warm-up) and then to mentally perform a series of tasks (reaching for a cup). Patients mentally practised both at home and during supervised therapy sessions. Patients in both the control and intervention groups also received occupational therapy. Page et al. reported significant improvement in Fugl-Meyer scores (2000; 2007) and Action Research Arm tests (2005; 2007) between treatment and control subjects. Patients in the study by Dijkerman et al. (2004) were asked to read a set of instructions directing them through a series of tasks. A placebo mental imagery condition was also used where patients were asked to describe a series of pictures, which had been presented previously. There was no difference between groups with respect to ADL performance, measured by the Barthel Index. The study by Liu et al. (2004) had patients in the mental practice group practice a different series of mental tasks each week (ie. week 1: wash the dishes, prepare tea, fold laundry) while patients in the control group (functional training program) practiced the same tasks having the therapist first demonstrate the task. This study reported a benefit of treatment in terms of improvement in ADL performance. Page et al. (2005) also demonstrated that mental practice, where patients cognitively rehearse ADL activities, improved motor function in the affected upper limbs of chronic stroke patients. The author suggests that the technique induces use-dependent brain reorganization to achieve the improvements in motor function. The SREBR (11th edition) noted that there is strong evidence that mental practice can improve upper-extremity motor and ADL performance following stroke (Teasell et al. 2008). References Braun SM, Beurskens AJ, Borm PJ, Schack T, Wade DT. The effects of mental practice in stroke rehabilitation: a systematic review. Archives of Physical Medicine & Rehabilitation. 2006; 87(6):842-852. Dijkerman HC, Letswaart M, Johnston M, MacWalter RS. Does motor imagery training improve hand function in chronic stroke patients? A pilot study. Clinical Rehabilitation. 2004; 18(5):538-549. Liu K.P., Chan C.C., Lee T.M., Hui-Chan C.W. et al. Mental imagery for promoting relearning for people after stroke: A Randomized Controlled Trial. Arch Phys Med Rehabil 2004; 85(9);1403-1408. Page S.J. Imagery improves upper extremity motor function in chronic stroke patients: A pilot study. The Occupational Therapy Journal of Research 2000; 20(3):200-213. Page SJ, Levine P. Back from the brink: electromyography-triggered stimulation combined with constraint-induced movement therapy in chronic stroke. Arch Phys Med Rehabil 2006; 87:27-31. Page SJ, Levine P, Leonard A. Mental practice in chronic stroke: results of a randomized, placebo-controlled trial. Stroke 2007;38:1293-1297 Page SJ, Levine P, Leonard AC. Effects of mental practice on affected limb use and function in chronic stroke. Arch Phys Med Rehabil 2005; 86(3):399-402. Page J.S., Levine P., Sisto S., & Johnston M.V. Mental practice combined with physical practice for upper-limb motor deficit in sub-acute stroke. Physical Therapy 2001; 81(8):1455-1462. Teasell RW, Foley NC, Salter K, Bhogal SK, Jutai J, Speechley MR. Evidence-Based Review of Stroke Rehabilitation (11th edition). Canadian Stroke Network; 2008. Verbunt JA, Seelen HA, Ramos FP, Michielsen BH, Wetzelaer WL, Moennekens M. Mental practice-based rehabilitation training to improve arm function and daily activity performance in stroke patients: a randomized clinical trial. BMC Neurol 2008;8:7

Page 66: C. Motor Recovery pictures removed - EBRSRPR).pdf · - 3 - C1. Assessment of Motor Function Post Stroke C1.1 Assessments of Motor Function Post Stroke Case Study A 65 year old male

- 66 - 66

Zimmermann-Schlatter A, Schuster C, Puhan MA, Siekierka E, Steurer J. Efficacy of motor imagery in post-stroke rehabilitation: a systematic review. J Neuroeng Rehabil 2008;5:8.

Page 67: C. Motor Recovery pictures removed - EBRSRPR).pdf · - 3 - C1. Assessment of Motor Function Post Stroke C1.1 Assessments of Motor Function Post Stroke Case Study A 65 year old male

- 67 - 67

C12. Spasticity Post Stroke

Page 68: C. Motor Recovery pictures removed - EBRSRPR).pdf · - 3 - C1. Assessment of Motor Function Post Stroke C1.1 Assessments of Motor Function Post Stroke Case Study A 65 year old male

- 68 - 68

C12. Spasticity Post Stroke C12.1 Spasticity of the Hemiplegic/Hemiparetic Limbs Post Stroke Q1. Describe spasticity post-stroke. Answer Spasticity Post-Stroke Definition: Usually seen days to weeks post-ischemic stroke. Usually follows classic upper extremity flexor and lower extremity extensor patterns during

ambulation. Velocity-dependent resistance to passive movement of affected muscles at rest. Q2. Describe one positive aspect of spasticity on a stroke patient. Answer Positive Aspect of Spasticity in Stroke: Allows hemiplegic patient to weight-bear on affected leg during stance phase. Q3. Describe a number of potential treatments for spasticity in a patient with upper motor neuron syndrome. Answers Potential Treatments for Spasticity in UMN Disorder: Conservative Treatment: stretching, splints/orthotics, serial casting, electrical stimulation,

local application of cold. Medications: benzodiazepine, baclofen, dantrolene, tizmidine. Injections: Bo-Tox and phenol. Intrathecal Baclofen. Surgical procedures. C12.2 Spasticity in the Hemiplegic Lower Extremity Case Study A 38 year old male was admitted to hospital with a large left MCA infarct. CT scan showed a dense left MCA infarct, which was felt to be cardioembolic in etiology. 3 weeks

Page 69: C. Motor Recovery pictures removed - EBRSRPR).pdf · - 3 - C1. Assessment of Motor Function Post Stroke C1.1 Assessments of Motor Function Post Stroke Case Study A 65 year old male

- 69 - 69

after admission to acute care and following the onset of his stroke he was admitted to the rehabilitation unit. He required set-up assistance with his meals and tolerated a minced, honey-fluid diet. He required one person to assist him with dressing, grooming and bathing. He required in-and-out catheterizations for urinary retention. He was able to complete a 2 person pivot transfer despite problems with his dynamic balance. The patient could stand only in the parallel bars and used a manual wheelchair with a laptray for mobility. He was unable to communicate verbally but was able to gesture to make his needs known. Premorbidly he was quite active and was fully employed. This gentleman was eventually able to ambulate independently with a cane. However, he had problems with a spastic hemiplegic gait. He tended to walk on his toes with his ankle/foot in plantar inversion which was only partially compensated for by an ankle-foot orthosis. This would make his gait more inefficient, causing him to circumduct to clear the foot and sometimes throwing him off balance. Q4. When Should One Treat Spasticity of Hemiplegic Lower Extremity? Answers 1. Spasticity in the hemiplegic lower extremity is generally not treated. 2. Primary treatment is for spastic equinovarus, caused by spasticity of the gastrocnemius and

tibialis posterior muscles. Discussion Spasticity is common in stroke patients although it does not always require treatment. Treatments are likely to be most effective in the subset of stroke patients with severe spasticity Gresham et al. (1995) notes that, “Contractures that restrict movement of the involved joint or are painful will impede rehabilitation and may limit the patient’s potential for recovery. Paretic limbs with muscle spasticity are at especially high risk of developing contracture. Prevention is the key to effective management. Spastic equinovarus foot is a frequent complication following stroke, caused by spasticity of the gastrocnemius and tibialis posterior muscles. Q5. What treatment options are available for spasticity in the lower extremity? Answers 1. Typical treatment options include antispastic medication, orthotic devices (splints), physical

therapy, neurolysis with alcohol, phenol or botulinum toxin, as well as surgical options (Deltombe et al. 2004).

Page 70: C. Motor Recovery pictures removed - EBRSRPR).pdf · - 3 - C1. Assessment of Motor Function Post Stroke C1.1 Assessments of Motor Function Post Stroke Case Study A 65 year old male

- 70 - 70

Discussion The SREBR (11th edition) (Teasell et al. 2008) found the following:

1. Strong evidence that deinnervating muscles, in the lower extremity, with Botulinum toxin reduces spasticity.

2. Conflicting evidence whether such deinnervation improves functional outcomes. 3. Moderate evidence that nerve blocking (thermocoagulation of nerve) improves spasticity

compared to the use of an AFO. 4. Conflicting evidence that Dantrolene sodium is effective in treating post-stroke

spasticity compared to placebo. 5. Moderate evidence that ketazolam, diazepam and tolperisone are more effective when

compared to placebo in treating post-stroke spasticity. 6. Limited evidence that Tizanidine is not superior to oral Baclofen. 7. Moderate evidence that Tolperisone reduces spasticity. 8. Moderate evidence that intrathecal baclofen can reduce spasticity in the chronic stages

of stroke. 9. Strong evidence that electrical stimulation can reduce ankle plantarflexion spasticity

post stroke. 10. Moderate evidence that therapeutic ultrasound can reduce alpha motoneuron

excitability associated with ankle plantarflexor spasticity. Case Study (continued) The patient has heard about botulinum toxin as a “cure” for spasticity and wants to know if it will help with the spasticity in his lower leg. Q6. Explain to the patient the mechanism and goals of Botulinum toxin treatment (BT) in the lower extremity. Answers 1. Botulinum toxin works by weakening spastic muscles by blocking the neuromuscular

junction. 2. Works for up to 6 months. 3. In lower extremity most treatment is focused on spastic equinovarus deformity. 4. Primary goals of treatment are improvement in gait velocity and quality, reduced pain and

improved posture. Discussion Botulinum toxin works by weakening spastic muscles through selectively blocking the release of acetylcholine at the neuromuscular junction. The benefits of botulinum injections are generally dose-dependent and last approximately 2 to 4 months (Simpson et al. 1996). One of the advantages of botulinum is that it is safe to use on small, localized areas or muscles, such as those in the upper extremity. Unlike chemical neurolysis with either phenol or alcohol, botulinum toxin is not associated with skin sensory loss or dysesthesia (Suputtitada &

Page 71: C. Motor Recovery pictures removed - EBRSRPR).pdf · - 3 - C1. Assessment of Motor Function Post Stroke C1.1 Assessments of Motor Function Post Stroke Case Study A 65 year old male

- 71 - 71

Sunanwela 2005). The main goals of BT of muscles of the leg and foot are improvement in active function, increased participation, improved gait velocity and quality, and reduced reliance on walking aids. Secondary goals may include increased comfort, reduced pain, reduced burden of care, and facilitation of brace use or appropriate seating (Gracies et al. 2007). Treatment of the gastrocsoleus muscle when there is excessive spastic plantarflexion when in the stance phase of walking is an appropriate indication for botulinum toxin and will improve the safety, speed and efficiency (effort) of the patient’s gait. Q7. Describe the impact of botulinum toxin in the spastic lower extremity. Answer 1. Botulinum toxin in the lower extremity has been shown to reduce spasticity but not

necessarily function. Discussion With respect to the treatment of spasticity in the lower extremity post stroke, there are fewer studies available than in the upper extremity. Only two RCTs compared the effects of BTx to a placebo (Burdaud et al. 1996, Pittock et al. 2003). Both reported an improvement in spasticity, but not function. Two studies compared BTx injection to a phenol block, with mixed results in terms of measures of spasticity, while function was not evaluated. The SREBR (11th edition) found strong evidence that deinnervating muscles, in the lower extremity, with Botulinum toxin reduces spasticity. However, there is conflicting (Level 4) evidence whether such deinnervation improves functional outcomes (Teasell et al. 2008). Q8. What are some of the common indications for use of botulinum toxin in the spastic lower extremity, which muscles are commonly involved and what is the functional impact of these spastic muscles? Answers 1. Hip adductors (adductor longus/brevis/magnus) to reduce scissoring thighs and improve

hygiene. 2. Flexed knee (hamstrings/gastrocnemius) to improve swing step length. 3. Extended knee (gluteus medius/quadriceps) to improve knee flexion on early swing phase. 4. Equinovarus foot (gastrocsoleus/tibialis posterior) to improve dorsiflexion and eversion. 5. Extended big toe (extensor hallucis longus) to reduce hyperextension and improve ability to

wear shoes. Discussion

Page 72: C. Motor Recovery pictures removed - EBRSRPR).pdf · - 3 - C1. Assessment of Motor Function Post Stroke C1.1 Assessments of Motor Function Post Stroke Case Study A 65 year old male

- 72 - 72

Muscles Involved Functional Impact Adducted thighs Adductor

longus/brevis/magnus Gracilis Iliopsoas (weak) Pectineus (weak)

Scissoring thighs when sitting and walking Difficulty with hygiene, catheterization, urinal use, dressing and mobility

Flexed Hip

Iliopsoas Hamstrings

Flexed Knee Hamstrings medial/lateral Gastrocnemius

Knee remains flexed throughout swing and stance phases of gait Limited limb advancement with resultant short step length

Extended Knee Gluteus maximus Rectus femoris Vastus lateralis/medialis /intermedius Ilipsoas (weak)

Knee remains extended throughout gait cycle Toe drag in early swing phase may cause tripping and falling

Equinovarus Foot Gastrocnemius medial/lateral Soleus Tibialis posterior Extensor hallucis longus Long toe flexors Peroneus longus

Foot and ankle turned in – dynamic spastic equinus deformity of ankle and inverted foot at stance or swing phases During stance phase, contact occurs at the forefoot; weight is borne primarily on lateral border

Striatal Toe Extensor hallucis longus Inability to wear a shoe When wearing a shoe, pain at tip of toe and under first metatarsal during stance

Toe Clawing and Curling

Long or short toe flexors

C12.3 Spasticity in the Hemiplegic Upper Extremity Case Study (continued) The patient notes that when he walks his arm goes up into forward flexed posture, with prominent spastic flexion of the elbow and wrist. He finds this posture bothersome, it gets in the way of his dressing, it makes him appear more disabled and can be bothersome by the end of the day. Q9. Describe the benefits of botulinum toxin in improving function in the spastic upper extremity. Answers

Page 73: C. Motor Recovery pictures removed - EBRSRPR).pdf · - 3 - C1. Assessment of Motor Function Post Stroke C1.1 Assessments of Motor Function Post Stroke Case Study A 65 year old male

- 73 - 73

1. Botulinum toxin has been shown to reduce spasticity in the upper extremity. 2. However, botulinum toxin has not been shown to necessarily improve function likely

because underlying weakness more than spasticity results in the limitation of function. 3. Modest improvements in the dressing, grooming and eating on the Barthel Index score have

been reported following botulinum toxin injections. Discussion While many controlled studies have demonstrated a reduction in spasticity in the upper extremity following treatment with botulinum toxin (usually using BTX-A (Botox or Dysport) and measured with the Modified Ashworth Scale or range of motion), it is less clear whether this treatment is associated with an improvement in upper extremity function. Francis et al. (2004) suggested several reasons why this might be so. These authors suggested that underlying muscle weakness and not spasticity contributes to the limitation in function. However, they also speculated that the most likely reason for this was insufficiently sensitive outcome measures and under-powered studies. A meta-analysis by the same authors, which included the results from two RCTs (Bakheit et al. 2000, 2001), lead Francis et al. (2004) to suggest that BTX-A does have a modest beneficial affect on function. The authors of this review pooled the data and assessed the effect on the arm section of the Barthel Index (dressing, grooming and eating) and reported a modest improvement in upper arm function following botulinum toxin. Pooling was only possible for two RCTs due to heterogeneity of interventions and outcomes. The SREBR (11th edition) notes that there is strong evidence that treatment with botulinum toxin injection alone or in combination with therapy significantly decreases spasticity in the upper extremity in stroke survivors. However, it is not clear that the improvements are sustained, nor is there strong evidence that they are associated with improved function and quality of life. There is moderate evidence that electrical stimulation combined with botulinum toxin injection is associated with reductions in muscle tone. There is strong evidence that a combination of physiotherapy and botulinum toxin injection is associated with improved upper extremity function (Teasell et al. 2008). Q10. What are some of the common indications for use of botulinum toxin in the spastic upper extremity, which muscles are commonly involved and what is the functional impact of these spastic muscles? Answers 1. Adducted/internally rotated shoulder (subscapularis/pectoralis major) to improve on

adduction and internally rotated shoulder tightness/contracture and pain. 2. Flexed elbow (brachioradialis/biceps/brachialis) to make ADLs and hygiene easier as well

as improve cosmesis. 3. Pronated forearm (pronator quadratus/pronator teres) to improve hand orientation. 4. Flexed wrist (flexor carpi radialis/brevis/ulnaris/extrinsic finger flexors) to improve ADLs and

reduce pain. 5. Clenched fist (flexor digitorum profundus/sublimis) to improve hygiene. 6. Thumb in palm deformity (adductor pollicis/flexor pollicis longus/thenar group) to improve

thumb for key grasp.

Page 74: C. Motor Recovery pictures removed - EBRSRPR).pdf · - 3 - C1. Assessment of Motor Function Post Stroke C1.1 Assessments of Motor Function Post Stroke Case Study A 65 year old male

- 74 - 74

Discussion Muscles Involved Functional Impact Adducted/Internally Rotated Shoulder

Pectoralis major Latissimus dorsi Teres major Subscapularis

Arm adducted tightly; forearm lies against middle of chest Severely restricted ability to reach targets/apply force/push objects Frozen shoulder Dressing problems

Flexed Elbow Brachioradialis Biceps Brachialis

Bent elbow inadvertently hooks onto things Difficulty dressing and reaching for objects Can lead to skin maceration and breakdown

Pronated Forearm Pronator quadrates Pronator teres

Impairs ability to orient hand

Flexed Wrist Flexor carpi radialis/brevis/ulnaris Extrinsic finger flexors

Difficulty inserting hands into narrow openings (eg, sleeves) May have pain on passive range of motion Carpal tunnel symptoms may occur

Clenched Fist Flexor digitorum profundus Flexor digitorum sublimis

Fingers clasped in palm Unable to wash palm Skin maceration, breakdown and noxious odour

Thumb-In-Palm Deformity

Adductor pollicis Flexor pollicis longus Thenar group

Thumb held within palm Distal interphalangeal joint flexed Thumb unable to function during key grasp

Case Study (continued) In particular, this patient notes that when he walks his arm goes up into forward flexed posture, with prominent spastic flexion of the elbow. Q11. The patient specifically asks if botulinum toxin will improve the appearance of his spastic elbow flexion which becomes more prominent when he ambulates. Which of the muscles would you inject? Answer 1. Botulinum toxin into the elbow flexors (brachioradialis, biceps and brachialis muscles) has

been shown to reduce spasticity. It may not last and will likely not be associated with

Page 75: C. Motor Recovery pictures removed - EBRSRPR).pdf · - 3 - C1. Assessment of Motor Function Post Stroke C1.1 Assessments of Motor Function Post Stroke Case Study A 65 year old male

- 75 - 75

significant functional improvement although it will improve the cosmetic effect. References Bakheit AM, Thilmann AF, Ward AB, Poewe W, Wissel J, Muller J, Benecke R, Collin C, Muller F, Ward CD, Neumann C. A randomized, double-blind, placebo-controlled, dose-ranging study to compare the efficacy and safety of three doses of botulinum toxin type A (Dysport) with placebo in upper limb spasticity after stroke. Stroke. 2000 Oct;31(10):2402-6. Bakheit AM, Pittock S, Moore AP, Wurker M, Otto S, Erbguth F, Coxon L. A randomized, double-blind, placebo-controlled study of the efficacy and safety of botulinum toxin type A in upper limb spasticity in patients with stroke. Eur J Neurol. 2001;8(6):559-65. Burbaud P, Wiart L, Dubos JL, Gaujard E, Debelleix S, Joseph PA, Mazaux JM, Bioulac B, Marat M, Lagueny A. A randomised, double blind placebo controlled trial of botulin toxin in the treatment of spastic foot in hemiparetic patients. J Neurology, Neurosurgery and Psychiatry 1996;61:265-269. Deltombe T, De Wispelaere JF, Gustin T, Jamart J, Hanson P. Selective blocks of the motor nerve branches to the soleus and tibialis posterior muscles in the management of the spastic equinovarus foot. Arch Phys Med Rehabil 2004; 85(1):54-8. Francis HP, Wade DT, Turner-Stokes L, Kingswell RS, Dott CS, Coxon EA. Does reducing spasticity translate into functional benefit? An exploratory meta-analysis. J Neurol Neurosurg Psychiatry 2004; 75(11):1547-1551. Gracies JM, Singer BJ, Dunne JW. The role of botulinum toxin injections in the management of muscle overactivity of the lower limb. Disability and Rehabilitation 2007; 29: 1789-805. Pittock SJ, Moore AP, Hardiman O, Ehler E, Kovac M, Bojakowski J, al Khawaja I, Brozman M, Kanovsky P, Skorometz A, Slawek J, Reichel G, Stenner A, timberbaeva S, Stelmasiak Z, Zifko UA, Bhakta B, Coxon E. A double-blind randomised placebo-controlled evaluation of three doses of botulinum toxin type A (Dysport®) in the treatment of spastic equinovarus deformity after stroke. Cerebrovasc dis 2003;15:289-30. Simpson DM, Alexander DN, O'Brien CF, Tagliati M, Aswad AS, Leon JM, Gibson J, Mordaunt JM, Monaghan EP. Botulinum toxin type A in the treatment of upper extremity spasticity: a randomized, double-blind, placebo-controlled trial. Neurology 1996;46:1306-1310. Suputtitada A, Suwanwela NC. The lowest effective dose of botulinum A toxin in adult patients with upper limb spasticity. Disabil Rehabil 2005;27:176-184. Teasell RW, Foley NC, Salter K, Bhogal SK, Jutai J, Speechley MR. Evidence-Based Review of Stroke Rehabilitation (11th edition). Canadian Stroke Network; 2008.

Page 76: C. Motor Recovery pictures removed - EBRSRPR).pdf · - 3 - C1. Assessment of Motor Function Post Stroke C1.1 Assessments of Motor Function Post Stroke Case Study A 65 year old male

- 76 - 76

Key Study: Botulinum Toxin for Spastic Equinovarus Deformity Post Stroke Pittock SJ, Moore AP, Hardiman O, Ehler E, Kovac M, Bojakowski J, al Khawaja I, Brozman M, Kanovsky P, Skorometz A, Slawek J, Reichel G, Stenner A, timberbaeva S, Stelmasiak Z, Zifko UA, Bhakta B, Coxon E. A double-blind randomised placebo-controlled evaluation of three doses of botulinum toxin type A (Dysport®) in the treatment of spastic equinovarus deformity after stroke. Cerebrovasc Dis 2003;15:289-30. Author / Year Country PEDro score

Methods Outcome

Pittock et al. 2003 UK 8 (RCT)

In a double-blind, placebo-controlled, dose-ranging study, 234 patients with hemiparesis with spastic equinovarus deformity of the ankle after stroke were randomized to one of 4 treatment groups: 500 units of Dysport; 1000 units of Dysport; 1500 units of Dysport and placebo. Patients were assessed every 4 weeks over a 12-week period.

Distance covered during 2-minuate walking test significantly increased in each group, but there were no differences between groups. Significant improvement in calf spasticity, limp pain reduction in use of walking was noted in the Dysport groups relative to the control group.

Walking Parameters at Baseline and 12 Weeks: Placebo vs.

Different Treatment Levels

No significant differences were found between

groups.

0

10

20

30

40

50

60

70

80

Baseline 12 Weeks Baseline 12 Weeks Baseline 12 Weeks

2-min Walking Test, m Difference in Step Length, cm Stepping Rate, steps/min

Walking Parameter

Val

ue

Placebo Dysport 500 units Dysport 1000 units Dysport 1500 units

Importance: This RCT compared the effect of Botox to placebo on various gait parameters. Although gait quality was improved, 2-minute walking speed was not improved over the control. Relevant SREBR Conclusions: There is strong evidence that deinnervating muscles with Botulinum toxin in the lower extremity reduces spasticity. There is conflicting evidence whether such deinnervation improves functional outcomes. Related Reference Burbaud P, Wiart L, Dubos JL, Gaujard E, Debelleix S, Joseph PA, Mazaux JM, Bioulac B, Marat M, Lagueny A. A randomised, double blind placebo controlled trial of botulin toxin in the treatment of spastic foot in hemiparetic patients. J Neurology, Neurosurgery and Psychiatry 1996;61:265-269.

Page 77: C. Motor Recovery pictures removed - EBRSRPR).pdf · - 3 - C1. Assessment of Motor Function Post Stroke C1.1 Assessments of Motor Function Post Stroke Case Study A 65 year old male

- 77 - 77

Key Study: Botulinum Toxin for Spasticity Upper Extremity Post Stroke Brashear A, McAfee AL, Kuhn ER, Fyffe J. Botulinum toxin type B in upper-limb poststroke spasticity: a double-blind, placebo-controlled study. Arch Phys Med Rehabil 2004;85:705-709. Author / Year Country PEDro score

Methods Outcome

Brashear et al. 2004 USA 7 (RCT)

15 stroke patients were randomized to receive a single Botox type B injection in the elbow, wrist, finger and thumb (n=10) or placebo (n=5). Measures were recorded at 2, 4, 8, 12 and 16 weeks.

There was no significant decrease in muscle tone in the elbow, wrist, or finger. A decrease in Ashworth scale scores was observed at the wrist at week 2 in the treatment group. Improvement was also observed at week 4 for the elbow (p=.039), wrist (p=.002), finger (p=.001) and thumb (p=.002) in the treatment gr. Improvements were not sustained.

Importance: Only two studies have evaluated the effectiveness of botulinum toxin B for upper-limb spasticity associated with stroke. Relevant SREBR Conclusions: There is strong that treatment with botulinum toxin either alone or in combination with therapy significantly decreases spasticity in the upper extremity in stroke survivors. However, it is not clear that the improvements are sustained, nor is there strong evidence that they are associated with improved function and quality of life. Related References Brashear A, Gordon MF, Elovic E et al. Intramuscular injection of botulinum toxin for the treatment of wrist and finger spasticity after a stroke. N Engl J Med 2002; 347(6):395-400. Brashear A, McAfee AL, Kuhn ER, Ambrosius WT. Treatment with botulinum toxin type B for upper-limb spasticity. Arch Phys Med Rehabil 2003; 84:103-107.

Page 78: C. Motor Recovery pictures removed - EBRSRPR).pdf · - 3 - C1. Assessment of Motor Function Post Stroke C1.1 Assessments of Motor Function Post Stroke Case Study A 65 year old male

- 78 - 78

C13. Hemiplegic Shoulder Pain

Page 79: C. Motor Recovery pictures removed - EBRSRPR).pdf · - 3 - C1. Assessment of Motor Function Post Stroke C1.1 Assessments of Motor Function Post Stroke Case Study A 65 year old male

- 79 - 79

C13. Hemiplegic Shoulder Pain C13.1 The Early Flaccid Upper Extremity Case Study A 75-year old gentleman suffers a moderately severe left middle cerebral artery territory stroke and is admitted to an acute stroke unit. As a rehabilitation clinician you are asked to assess him two days following admission. You notice that he has a flaccid left upper extremity. Q1. You are asked to immediately provide a management plan for the upper extremity designed to protect the affected shoulder. Answers 1. Flaccid hemiplegic arm at 2 days post stroke 2. Careful positioning of the hemiplegic shoulder toward abduction, external rotation and

flexion of the shoulder. 3. Ensure the flaccid arm is continuously supported when the patient is sitting or transferring –

use arm trough, lap tray or arm sling. 4. Avoid pulling on the flaccid arm during transfers. 5. Very gentle range of motion exercises with physiotherapy. Discussion Ideally, measures should be taken immediately following a stroke to minimize the likelihood of developing hemiplegic shoulder pain in the future. This includes early passive shoulder range of motion, and supporting and protecting the involved shoulder in the initial flaccid stage. Positioning of the Hemiplegic Shoulder: Careful positioning of the shoulder serves to minimize subluxation and later contractures. It has been suggested that through careful and correct positioning the development of shoulder pain can be prevented. Bender and McKenna (2001) noted that the “recommended position for the upper limb is towards abduction, external rotation and flexion of the shoulder”; however, there is no consensus regarding the exact degree of positioning (Carr and Kenny 1992). The Stroke Rehabilitation Evidence-Based Review (11th edition) (Teasell et al. 2008) noted that there was consensus opinion that proper positioning of the hemiplegic shoulder helps to avoid shoulder subluxation. However, there was conflicting evidence that prolonged positioning did not influence active and passive range of motion or level of pain. Hemiplegic Arm Support: The flaccid hemiplegic arm must be carefully supported at all times, particularly when the patient is sitting up in a chair or wheelchair. Otherwise there is risk of

Page 80: C. Motor Recovery pictures removed - EBRSRPR).pdf · - 3 - C1. Assessment of Motor Function Post Stroke C1.1 Assessments of Motor Function Post Stroke Case Study A 65 year old male

- 80 - 80

subluxation of the glenohumeral joint. There are several means to support the hemiplegic arm including arm troughs, lap trays and arm slings. Arm slings are often used in the initial stages following a stroke to support the affected arm. Advantages:

Best method of supporting flaccid hemiplegic arm while the patient is standing or transferring

Disadvantages: Encourage flexor synergies Inhibit arm swing Contracture formation Decrease body image causing the patient to further avoid using that arm

As tone returns to the shoulder muscles, the risk of shoulder subluxation decreases and slings can be withdrawn. Slings tend to hold the limb in a poor position, which may accentuate the adduction and internal rotation posture and may contribute to shortening of tonically active muscles. An alternative for when the patient is sitting in a wheelchair is a lapboard or arm trough.

The Stroke Rehabilitation Evidence-Based Review (11th edition) (Teasell et al. 2008) noted that there was limited evidence that shoulder slings prevent subluxation associated with hemiplegic shoulder. There was limited evidence that one device was no better than the other. C13.2 The Later Spastic Upper Extremity Case Study (continued) The same patient is admitted to the rehabilitation unit within 10 days of suffering their moderate-sized left middle cerebral artery infarct. On examining his shoulder he has no motor movement involving the upper extremity apart from being able to shrug his shoulder. Passively you are able to forward flex his shoulder to 90 degrees, externally rotate it to a full 90 degrees with mild discomfort. He has reasonably good internal rotation and abduction is restricted to 100 degrees. Clinical examination of this gentleman’s shoulder also reveals mild subluxation of the shoulder joint. Three weeks later the physiotherapist comments that this gentleman is suffering from quite significant shoulder pain. You examine him and his physical examination has changed. He has made some mild motor recovery in that he now has a Chedoke McMaster Outcome Score of 3 in the arm, and 2 in the hand. His tone has increased significantly involving the upper extremity. He is now able to only obtain 70 degrees of abduction, 40 degrees of external rotation with significant pain at end range, particularly when it is held in the end range adducted position (70 degrees) and he still had reasonably good internal rotation. The patient himself does not express himself well but when you ask him if it hurts he points to his right anterior shoulder. Treatment with anti-inflammatory drugs and Tylenol

Page 81: C. Motor Recovery pictures removed - EBRSRPR).pdf · - 3 - C1. Assessment of Motor Function Post Stroke C1.1 Assessments of Motor Function Post Stroke Case Study A 65 year old male

- 81 - 81

#3’s do not appear to be assisting him and the nurses report that pain is beginning to keep him up at night and that they find him quite restless. Q2. What are the possible pathophysiological causes of this gentleman’s pain? Answers 1. Shoulder contracture (due to subscapularis/pectoralis hypertonicity) 2. Shoulder subluxation 3. Rotator cuff disorder 4. Fracture Q3. Assuming he has had no pain premorbidly and little pain at time of admission, what is the most likely cause of this gentleman’s pain? Answer 1. Most likely cause is spasticity – muscle imbalance (subscapularis and pectoralis

hypertonicity) – shoulder contracture. Discussion The most likely cause of pain is spasticity, muscle imbalance, and subsequent shoulder contraction. Hemiplegia post-stroke is characterized by typical posturing which is reflective of hypertonic muscle patterns. Flexor tone predominates and results in scapular retraction and depression as well as internal rotation and adduction of the shoulder. With the predominance of internal rotation of the affected shoulder, these muscles become contracted, shortening and preventing external rotation. External rotation of the humerus is necessary to allow the shoulder to abduct beyond 90 degrees. The two muscles which have been implicated in excessive internal rotation of the shoulder are the subscapularis and pectoralis muscles. Q4. Describe a management plan for this gentleman’s pain. Management: X-ray if history of fall Analgesics Support the arm Gentle ROM exercises Botulinum toxin injection if shoulder tightness and pain persists

Page 82: C. Motor Recovery pictures removed - EBRSRPR).pdf · - 3 - C1. Assessment of Motor Function Post Stroke C1.1 Assessments of Motor Function Post Stroke Case Study A 65 year old male

- 82 - 82

Case Study (continued) The therapist asks if it would be appropriate to try functional electrical stimulation. Q5. What would you advise regarding the use of functional electrical stimulation? Answer At present there is conflicting evidence that FES reduces pain, improves function and reduces subluxation following stroke. One study suggested it may be harmful; other studies have shown a benefit. Discussion Functional electrical stimulation is designed to compensate or facilitate flaccid shoulder muscles, which in turn should reduce the risk of shoulder subluxation. The ideal intensity of treatment is thought to be 6 hours daily, 5 days a week for 6 weeks (Paci et al. 2005). Most of the RCTs reviewed reported a benefit associated with electrical stimulation, although there was variability in the outcomes assessed: range of motion, muscle tone, EMG activity, shoulder subluxation, shoulder pain and muscle function. The results of these studies suggest that FES can reduce pain in the affected shoulder and also improve upper extremity function. However, the results from the largest and most methodological rigorous trial (Church et al. 2006) suggests that FES treatment might actually be associated with harm and may worsen arm function, especially among those with severe paresis. The authors proposed several mechanisms to potentially explain this finding including over-use and over-stimulation of the affected arm and the possibility that stimulation interfered with the motor re-learning processes, influencing and impeding upper limb recovery. The authors did not evaluate the benefit of FES with respect to reducing shoulder subluxation or spasticity. The Stroke Rehabilitation Evidence-Based Review (11th edition) (Teasell et al. 2008) noted that there was conflicting evidence that functional electrical stimulation reduced pain, improved function and reduced subluxation following stroke. Case Study (continued) The patient tells you that the shoulder pain is increasing and the therapist has been particularly aggressive, trying to get him to stretch out his tight shoulder even though it is very painful at the time.

Page 83: C. Motor Recovery pictures removed - EBRSRPR).pdf · - 3 - C1. Assessment of Motor Function Post Stroke C1.1 Assessments of Motor Function Post Stroke Case Study A 65 year old male

- 83 - 83

Q6. What would you advise with the overaggressive therapist and the progressively increasing shoulder pain? Answer 1. The therapist needs to discontinue the aggressive therapy, particularly if painful during the

process. Discussion The only RCT (Kumar et al. 1990) to look at shoulder exercises post-stroke found that an aggressive range of motion exercise program resulted in significantly more pain than a gentle range of motion approach. The Stroke Rehabilitation Evidence-Based Review (11th edition) (Teasell et al. 2008) demonstrated that there was moderate evidence that aggressive range of motion therapies, using overhead pullies results in increased rates of shoulder pain. Q7. What is the evidence for using botulinum toxin to treat this gentleman’s painful hemiplegic shoulder? 1. Based on the results of 3 studies there is conflicting evidence as to the benefit of using

botulinum toxin to reduce hemiplegic shoulder pain. 2. However, options are limited and side effects minimal for use of botulinum toxin for the

hemiplegic shoulder. Discussion Subscapularis spasticity is characterized by shoulder range of motion being most limited with pain being reproduced on external rotation. This appears to correlate well with hemiplegic shoulder pain that is now thought to be a consequence of spastic muscle imbalance about the shoulder in many cases. Pectoralis muscle spasticity, characterized by limitation of range and pain on shoulder abduction, is seen to a lesser extent, causing a similar muscle imbalance. Intra-articular injections of steroids, botulinum toxin and other agents have been used in an effort to treat spastic muscles, redress the imbalance and to relieve hemiplegic shoulder pain. The Stroke Rehabilitation Evidence-Based Review (11th edtion) (Teasell et al. 2008) has noted that three RCTs examined the potential benefit of botulinum toxin in the treatment of hemiplegic shoulder (Kong et al. 2007, Yelnik et al. 2007, De Boer et al. 2008). All trials compared botulinum toxin (either 500 or 1,000 MU) with placebo. Based on the results from these trials, there was conflicting evidence that botulinum toxin injected into the subscapularis muscle reduces spastic shoulder pain and improves passive range of motion of the hemiplegic shoulder. However, in a case such as this, options are limited and the potential side-effects of botulinum toxin small. References

Page 84: C. Motor Recovery pictures removed - EBRSRPR).pdf · - 3 - C1. Assessment of Motor Function Post Stroke C1.1 Assessments of Motor Function Post Stroke Case Study A 65 year old male

- 84 - 84

Bender L, Mckenna K. Hemiplegic shoulder pain: Defining the problem and its management. Disabil Rehabil 2001;23(16):698-705. Carr EK, Kenney FD. Positioning of the stroke patient: a review of the literature. Int J Nurs Stud 1992;29(4):355-369. Church C, Price C, Pandyan AD, Huntley S, Curless R, Rodgers H. Randomized Controlled Trial to Evaluate the Effect of Surface Neuromuscular Electrical Stimulation to the Shoulder After Acute Stroke. Stroke 2006;37:2995-3001. De Boer KS, Arwert HJ, de Groot JH, Meskers CG, Mishre AD, Arendzen JH. Shoulder pain and external rotation in spastic hemiplegia do not improve by injection of botulinum toxin A into the subscapular muscle. J Neurol Neurosurg Psychiatry 2008;79:581-583. Kong KH, Neo JJ, Chua KS. A randomized controlled study of botulinum toxin A in the treatment of hemiplegic shoulder pain associated with spasticity. Clin Rehabil. 2007;21:28-35. Kumar R, Metter EJ, Mehta AJ, Chew T. Shoulder pain in hemiplegia. The role of exercise. American J Phys Med Rehabil 1990; 69(4):205-208. Paci M. Physiotherapy based on the Bobath concept for adults with post-stroke hemiplegia: a review of effectiveness studies. J Rehabil Med 2003;35:2-7. Teasell RW, Foley NC, Salter K, Bhogal SK, Jutai J, Speechley MR. Evidence-Based Review of Stroke Rehabilitation (11th edition). Canadian Stroke Network; 2008. Yelnik AP, Colle FM, Bonan IV, Vicaut E. Treatment of shoulder pain in spastic hemiplegia by reducing spasticity of the subscapular muscle : A randomized, double-blind, placebo-controlled study of botulinum toxin A. J Neurol Neurosurg Psychiatry 2007;78:845-848.

Page 85: C. Motor Recovery pictures removed - EBRSRPR).pdf · - 3 - C1. Assessment of Motor Function Post Stroke C1.1 Assessments of Motor Function Post Stroke Case Study A 65 year old male

- 85 - 85

Key Study: Exercises for Hemiplegic Shoulder Pain Kumar R, Metter EJ, Mehta AJ, Chew T. Shoulder pain in hemiplegia. The role of exercise. Am J Phys Med Rehabil 1990;69(4):205-208. Author / Year Country PEDro score

Methods Outcome

Kumar et al. 1990 USA 5 (quasi-randomized controlled trial)

28 patients were randomized to receive a rehabilitation program of range of motion by therapist (ROMT) once a day, 5 days a week; or a rehabilitation program with use of skate board once a day, 5 days a week; or a rehabilitation program with use of overhead pulley once a day, 5 days a week while an inpatient on a stroke rehabilitation unit.

Significant difference in the incidence of pain reported between the groups. Shoulder pain was more common in the overhead pulley (63%) group than in the ROMT group (8%). ROM was significantly reduced in those patients who developed shoulder pain when compared to those who did not develop shoulder pain motion abduction, forward flexion, internal rotation and external rotation. Shoulder subluxation was found in 46% of all patients with no significant difference between treatment groups.

0

2

4

6

8

10

12

Num

ber o

f Pat

ient

s

ROMT SB OP

The Relationship Between the Incidence of Pain in 3 Exercise Groups

Pain No Pain ROMT=range of motion by therapist; SB=skateboard; OP=overhead pulley

Importance: This study was one of the first to investigate the effectiveness of three different exercise approaches aimed at the reduction of shoulder pain and the first to suggest that certain forms of aggressive physiotherapy might be harmful. Relevant SREBR Conclusions: There is moderate evidence that aggressive range of motion therapies, using overhead pullies results in increased rates of shoulder pain. There is moderate evidence that gentle exercises to improve range of motion help to reduce shoulder pain. Related References Inaba MK, Piorkowski M. Ultrasound in treatment of painful shoulder in patients with hemiplegia. J Phys Ther 1972;52:737-741.

Page 86: C. Motor Recovery pictures removed - EBRSRPR).pdf · - 3 - C1. Assessment of Motor Function Post Stroke C1.1 Assessments of Motor Function Post Stroke Case Study A 65 year old male

- 86 - 86

Partridge CJ, Edwards SM, Mee R, Van Langenberghe HV. Hemiplegic shoulder pain: A study of two methods of phsyiotherapy treatment. Clinical Rehabilitation 1990;4:43-49. Poduri KR. Shoulder pain in stroke patients and its effects on rehabilitation. J Stroke Cerebrovasc Dis 1993;3:261-266. Tyson SF and Chissim C. The immediate effect of handling technique on range of movement in the hemiplegic shoulder. Clinical Rehabilitation 2002;16:137-140. Lynch D, Ferraro M, Krol J, Trudell CM, Christos P, Volpe BT. Continuous passive motion improves shoulder joint integrity following stroke. Clinical Rehabilitation 2005;19:594-599.

Page 87: C. Motor Recovery pictures removed - EBRSRPR).pdf · - 3 - C1. Assessment of Motor Function Post Stroke C1.1 Assessments of Motor Function Post Stroke Case Study A 65 year old male

- 87 - 87

C14. Case Study: Post-Stroke Motor Recovery

Page 88: C. Motor Recovery pictures removed - EBRSRPR).pdf · - 3 - C1. Assessment of Motor Function Post Stroke C1.1 Assessments of Motor Function Post Stroke Case Study A 65 year old male

- 88 - 88

C14. Case Study: Post-Stroke Motor Recovery Case Study 63 year old female was admitted with a right middle cerebral infarct, involving the frontal lobe, as a consequence of a cardiac emboli. As a consequence she is suffering from a right hemiparesis. At the time of admission to the stroke rehabilitation unit, 14 days after the onset of her stroke, she was admitted with a Berg Balance score of only 8/56; the patient is still experiencing a largely flaccid hemiplegia with Chedoke McMaster Staging scores on the right side of 1/7 in the hand and arm, 1/7 in the leg and 1/7 in the foot, and 1/7 for posture. There were no sensory problems noted. Q1. The patient asks you what the potential for recovery of the upper extremity is. What is your response? Answer 1. Prognosis for recovery of upper extremity is poor as patient has a largely flaccid upper

extremity with essentially no motor recovery at 14 days. Discussion Nakayama et al. (1994) reported that of stroke patients with severe arm paresis, with little or no movement at hospital admission, 14% make a complete motor recovery while 30% make a partial recovery. The odds of recovery for this patient, with no recovery at 14 days post-stroke, are less than that reported by Nakayama. Q2. The occupational therapist asks you how they should manage the hemiplegic upper extremity. Should they continue to treat the upper extremity with a goal of maximizing recovery or palliate the upper extremity, minimizing contractures and pain? Answers 1. Barreca (2001) recommended patients with a Chedoke-MacMaster score of less than 4

have a poor prognosis – should focus on palliation of minimizing contractures and pain. Discussion Reporting on an evidence-based consensus panel, Barreca et al. (2001) recommended that patients with a poor prognosis (defined as a Chedoke-MacMaster Score of less than stage 4)

Page 89: C. Motor Recovery pictures removed - EBRSRPR).pdf · - 3 - C1. Assessment of Motor Function Post Stroke C1.1 Assessments of Motor Function Post Stroke Case Study A 65 year old male

- 89 - 89

should receive treatment that focuses on minimizing contractures and pain. Barreca et al. (2001) noted that, “For the client with severe motor, sensory and functional deficits in the involved limb after stroke, the effectiveness literature indicates that additional treatment for the upper limb will not result in any significant neurological change. The evidence to date suggests that interventions may not lead to meaningful functional use of the affected limb at this stage of motor recovery.” Therefore, based on the consensus panel report, it is recommended that the therapist should focus on maintaining a comfortable, pain-free, mobile arm and hand. To maximize functional independence, stroke survivors with persistent motor and sensory deficits and their caregivers should be taught compensatory techniques and environmental adaptations that enable performance of important tasks and activities with the less affected arm and hand. Case Study (continued) The patient improves with rehabilitation. Although there is only limited recovery in her upper extremity she improves in her lower extremity to a CMS of 3 in her leg and 2 in her foot. She is able to ambulate, albeit with a spastic gait, a cane, and supervision. Unfortunately, because her ankle/foot keeps going into plantarflexion she has trouble clearing her foot or getting her heel down on the ground. The outpatient physiotherapist thinks that she would be a good candidate for botulinum toxin injections. Q3. Explain to the patient the mechanism and goals of Botulinum toxin treatment (BT) in the lower extremity. Answers 1. Botulinum toxin works by weakening selected spastic muscles by selectively blocking the

neuromuscular junction. 2. It works for up to 6 months. 3. The goal is to selectively block the spastic gastrocsoleus and tibialis posterior muscles to

eliminate the plantarflexion deformity and allow for greater ease of ambulation with less effort and greater gait velocity.

Discussion Botulinum toxin works by weakening spastic muscles through selectively blocking the release of acetylcholine at the neuromuscular junction. The benefits of botulinum injections are generally dose-dependent and last approximately 2 to 4 months (Simpson et al. 1996). One of the advantages of botulinum is that it is safe to use on small, localized areas or muscles, such as those in the upper extremity. Unlike chemical neurolysis with either phenol or alcohol, botulinum toxin is not associated with skin sensory loss or dysesthesia (Suputtitada & Sunanwela 2005).

Page 90: C. Motor Recovery pictures removed - EBRSRPR).pdf · - 3 - C1. Assessment of Motor Function Post Stroke C1.1 Assessments of Motor Function Post Stroke Case Study A 65 year old male

- 90 - 90

The main goals of BT of muscles of the leg and foot are improvement in active function, increased participation, improved gait velocity and quality, and reduced reliance on walking aids. Secondary goals may include increased comfort, reduced pain, reduced burden of care, and facilitation of brace use or appropriate seating (Gracies et al. 2007). Treatment of the gastrocsoleus muscle when there is excessive spastic plantarflexion when in the stance phase of walking is an appropriate indication for botulinum toxin and will improve the safety, speed and efficiency (effort) of the patient’s gait. Q4. Describe the benefits of botulinum toxin in improving function in the spastic upper extremity. Answers 1. Botulinum toxin has been shown to reduce spasticity in the upper extremity. 2. However, botulinum toxin has not been shown to necessarily improve function likely

because underlying weakness more than spasticity results in the limitation of function. 3. Modest improvements in the dressing, grooming and eating on the Barthel Index score have

been reported following botulinum toxin injections. Discussion While many controlled studies have demonstrated a reduction in spasticity in the upper extremity following treatment with botulinum toxin (usually using botulinum toxin A (Botox or Dysport) and measured with the Modified Ashworth Scale or range of motion), it is less clear whether this treatment is associated with an improvement in upper extremity function. Francis et al. (2004) suggested several reasons why this might be so. These authors suggested that underlying muscle weakness and not spasticity contributes to the limitation in function. However, they also speculated that the most likely reason for this was insufficiently sensitive outcome measures and under-powered studies. A meta-analysis by the same authors, which included the results from two RCTs (Bakheit et al. 2000, 2001), lead Francis et al. (2004) to suggest that BTX-A does have a modest beneficial affect on function. The authors of this review pooled the data and assessed the effect on the arm section of the Barthel Index (dressing, grooming and eating) and reported a modest improvement in upper arm function following botulinum toxin. Pooling was only possible for two RCTs due to heterogeneity of interventions and outcomes. References Bakheit AM, Pittock S, Moore AP, Wurker M, Otto S, Erbguth F, Coxon L. A randomized, double-blind, placebo-controlled study of the efficacy and safety of botulinum toxin type A in upper limb spasticity in patients with stroke. Eur J Neurol. 2001;8(6):559-65. Bakheit AM, Thilmann AF, Ward AB, Poewe W, Wissel J, Muller J, Benecke R, Collin C, Muller F, Ward CD, Neumann C. A randomized, double-blind, placebo-controlled, dose-ranging study to compare the efficacy and safety of three doses of botulinum toxin type A (Dysport) with placebo in upper limb spasticity after stroke. Stroke. 2000 Oct;31(10):2402-6. Barreca S. et al. Management of the Post Stroke Hemiplegic Arm and Hand: Treatment Recommendations of the 2001 Consensus Panel. Heart and Stroke Foundation of Ontario, 2001.

Page 91: C. Motor Recovery pictures removed - EBRSRPR).pdf · - 3 - C1. Assessment of Motor Function Post Stroke C1.1 Assessments of Motor Function Post Stroke Case Study A 65 year old male

- 91 - 91

Francis HP, Wade DT, Turner-Stokes L, Kingswell RS, Dott CS, Coxon EA. Does reducing spasticity translate into functional benefit? An exploratory meta-analysis. J Neurol Neurosurg Psychiatry 2004; 75(11):1547-1551. Gracies JM, Singer BJ, Dunne JW. The role of botulinum toxin injections in the management of muscle overactivity of the lower limb. Disability and Rehabilitation 2007; 29: 1789-805. Nakayama H, Jorgensen HS, Raaschou HO, Olsen TS. Recovery of upper extremity function in stroke patients: the Copenhagen Stroke Study. Arch Phys Med Rehabil 1994;75:394-398. Simpson DM, Alexander DN, O'Brien CF, Tagliati M, Aswad AS, Leon JM, Gibson J, Mordaunt JM, Monaghan EP. Botulinum toxin type A in the treatment of upper extremity spasticity: a randomized, double-blind, placebo-controlled trial. Neurology 1996;46:1306-1310. Suputtitada A, Suwanwela NC. The lowest effective dose of botulinum A toxin in adult patients with upper limb spasticity. Disabil Rehabil 2005;27:176-184.

Page 92: C. Motor Recovery pictures removed - EBRSRPR).pdf · - 3 - C1. Assessment of Motor Function Post Stroke C1.1 Assessments of Motor Function Post Stroke Case Study A 65 year old male

- 92 - 92

C15. Case Study: Lower Extremity and Mobility

Page 93: C. Motor Recovery pictures removed - EBRSRPR).pdf · - 3 - C1. Assessment of Motor Function Post Stroke C1.1 Assessments of Motor Function Post Stroke Case Study A 65 year old male

- 93 - 93

C15. Case Study: Lower Extremity and Mobility Case Study A 38 year old male was admitted to hospital with a large left MCA infarct. CT scan showed a dense left MCA infarct, which was felt to be cardioembolic in etiology. 3 weeks after admission to acute care and following the onset of his stroke he was admitted to the rehabilitation unit. He required set-up assistance with his meals and tolerated a minced, honey-fluid diet. He required one person to assist him with dressing, grooming and bathing. He required in-and-out catheterizations for urinary retention. He was able to complete a 2 person pivot transfer despite problems with his dynamic balance. The patient could stand only in the parallel bars and used a manual wheelchair with a laptray for mobility. He was unable to communicate verbally but was able to gesture to make his needs known. Premorbidly he was quite active and fully employed. Case Study (continued) Initial assessment on the stroke rehabilitation unit reveals the following: Admission 3 weeks post-stroke 2 Minute Walk Test Not walking Berg Balance Scale 5/56 COVS 30/91 CMS Right Leg 1/7 CMS Right Foot 1/7 Postural Control 4/7 CMS Arm 1/7 CMS Hand 1/7 Sensation Right Side Decreased Q1. Try to predict this gentleman’s clinical course. Presuming no medical complications will he walk again and if so how much assistance is he likely to require. Answer 1. Given this gentleman’s age, he will likely return to walking with an aid and supervision or

one person assist.

Page 94: C. Motor Recovery pictures removed - EBRSRPR).pdf · - 3 - C1. Assessment of Motor Function Post Stroke C1.1 Assessments of Motor Function Post Stroke Case Study A 65 year old male

- 94 - 94

Discussion The vast majority of stroke patients are able to return to some form of ambulation. In this case, given the patients age, he will likely return to walking, probably with an aid (cane) and one person assistance (Marini et al. 2001, Nedeltchev et al. 2005). Q2. The Question of whether this patient would benefit from partial body-weight support and treadmill training early on arises. Discuss the evidence for benefits of this treatment approach and any potential drawbacks. Answers 1. The evidence of PBWS and treadmill training is mixed but the weight of the evidence is

moving towards supporting PBWS. 2. Is supported by the general trend towards task-specific therapies. 3. Does require some equipment and can be therapist intensive. Discussion Partial body weight support combined with treadmill training is a recent innovation that allows non-ambulatory patients to practice the complex physiological movements of gait cycles by minimizing weight bearing demands. However, despite a strong neurological and physiological basis, there has been conflicting evidence regarding its relative effectiveness. For example, in a recent meta-analysis, Moseley et al. (2003) found that PWBS combined with treadmill training was no more effective than treadmill training alone. Interestingly, the authors also noted that PWBS did appear to be associated with a non-significant benefit for patients who were independent walkers at the start of treatment. Furthermore, there has also been some indication that more aggressive training may be associated with greater improvements (Eich et al. 2004, Sullivan et al. 2002). Thus, while there is continued uncertainty regarding the effectiveness of PWBS, further research is required to determine whether or not it is beneficial and in which circumstances. Case Study (continued) This patient remained on the rehabilitation unit for almost 3 months. He was discharged to home with the help of his spouse. Admission Discharge 2 Minute Walk Test Not walking 60 meters Greatest Distance Walked Before Requiring a Rest - 525 meters

Time to Walk Greatest Distance - 22 min, 42 sec Assistance Required - Supervision for walking Walking Device Used - Single point cane Berg Balance Scale 5/56 51/56 COVS 30/91 75/91

Page 95: C. Motor Recovery pictures removed - EBRSRPR).pdf · - 3 - C1. Assessment of Motor Function Post Stroke C1.1 Assessments of Motor Function Post Stroke Case Study A 65 year old male

- 95 - 95

CMS Right Leg 1/7 4/7 CMS Right Foot 1/7 2/7 Postural Control 4/7 5/7 CMS Arm 1/7 2/7 CMS Hand 1/7 2/7 Sensation Right Side Decreased Decreased

Q3. What is the risk of this gentleman falling based on the Berg Balance Score? Answer 1. This patient has a low risk of falling despite limited motor recovery in the lower extremity. 2. Berg Balance Score is 51/56 with any score greater than 45 indicating a much lower risk of

falling. Q4. This gentleman showed a marked improvement in terms of ambulation and mobility despite only modest improvements in his Chedoke McMaster or Brunnstrom recovery stages. What factors would account for this improvement? Answers 1. The patient was young. 2. Improvements in balance (51/56 on the Berg), likely because he was young and fit

premorbidly, helped to compensate for the modest motor recovery. 3. Sufficient recovery was seen in the all important proximal leg muscles which help to propel

the leg forward. 4. The increased tone which comes with recovery, is associated with increased extensor tone

in the lower extremity which helps to facilitate walking, especially be locking the knee in this case.

References Eich HJ, Mach H, Werner C, Hesse S. Aerobic treadmill plus Bobath walking training improves walking in subacute stroke: a randomized controlled trial. Clin Rehabil 2004;18:640-651. Marini C, Totaro R, De Santis F, Ciancarelli I, Baldassarre M, Carolei A. Stroke in Young Adults in the Community-Based L’Aquila Registry: Incidence and Prognosis. Stroke 2001;32:52-56. Moseley AM, Stark A, Cameron ID, Pollock A. Treadmill training and body weight support for walking after stroke. Stroke 2003; 34: 3006. Nedeltchev K, der Maur TA, Georgiadis D, et al. Ischaemic stroke in young adults: predictors of outcome and recurrence. J Neurol Neurosurg Psychiatry 2005;76:191-195.

Page 96: C. Motor Recovery pictures removed - EBRSRPR).pdf · - 3 - C1. Assessment of Motor Function Post Stroke C1.1 Assessments of Motor Function Post Stroke Case Study A 65 year old male

- 96 - 96

Sullivan KJ, Dnowlton BJ, Dobkin BH. Step training with body weight support: effect of treadmill and speed practice paradigms on poststroke locomotor recovery. Arch Phys Med Rehabil 2002;83:683-691.

Page 97: C. Motor Recovery pictures removed - EBRSRPR).pdf · - 3 - C1. Assessment of Motor Function Post Stroke C1.1 Assessments of Motor Function Post Stroke Case Study A 65 year old male

- 97 - 97

C16. Case Study: Upper Extremity Post-Stroke

Page 98: C. Motor Recovery pictures removed - EBRSRPR).pdf · - 3 - C1. Assessment of Motor Function Post Stroke C1.1 Assessments of Motor Function Post Stroke Case Study A 65 year old male

- 98 - 98

C16. Case Study: Upper Extremity Post-Stroke Case Study A 67 year old male suffered a small subcortical left hemispheric stroke, with subsequent right hemiparesis. He was admitted to rehabilitation 14 days following the onset of his stroke. The Chedoke McMaster Score of the lower extremity was 5 in the leg, 3 in the foot, 5 in the arm, and 2 in the hand. Q1. The occupational therapist is new and has not done much in the way of stroke rehabilitation in the past. She asks you how they should manage the hemiplegic upper extremity. How aggressively should she treat the upper extremity? Answer 1. Given that the arm is a CMS 5 in the arm and only 2 in the hand, nevertheless treatment

should be a more aggressive restorative program geared towards regaining function in the affected upper extremity.

Discussion Barreca et al. (2001), reporting on the 2001 Consensus Panel Recommendations for Patients with Moderate Impairment, noted that patients with less severe initial impairment (defined as a Chedoke-McMaster score > stage 4) should receive an aggressive restorative program geared towards regaining function in the affected upper extremity. The authors of the report noted that “patients with moderate impairments who demonstrate high motivation and potential for functional motor gains” need to engage in repetitive and intense use of novel tasks that challenge them to acquire necessary motor skills and to use the involved upper extremity during functional tasks and activities. These patients will also need to engage in motor-learning training, including the use of imagery. Therefore, for this patient, the therapist needs to provide an intensive rehabilitation therapy regimen with the goal of maximizing recovery. Q2. Your patient tells you that he knows he is going to improve because he frequently imagines moving his arm. What can you tell him about mental practice? Answers 1. There is strong evidence that mental practice may improve upper extremity motor and ADL

performance following stroke.

Page 99: C. Motor Recovery pictures removed - EBRSRPR).pdf · - 3 - C1. Assessment of Motor Function Post Stroke C1.1 Assessments of Motor Function Post Stroke Case Study A 65 year old male

- 99 - 99

2. Mental practice is designed to access and reinforce stored motor plans for executing movements

Discussion The technique of mental practice involves rehearsing a specific task or series of tasks mentally. The most plausible mechanism to explain the success of the technique is that stored motor plans for executing movements can be accessed and reinforced during mental practice. (Page et al 2001). It can be used to supplement conventional therapy and can be used at any stage of recovery. Zimmerman-Schlatter et al. (2008) assessed the efficacy of motor imagery in recovery post stroke and found that three of four RCTs (Liu et al. 2004, Page et al. 2001, 2007, Butler et al. 2006) reported improvements in the mean Action Research Arm Test and the Fugl-Meyer scores. The Stroke Rehabilitation Evidence-Based Review (11th edition) (Teasell et al. 2008) demonstrated that there was a strong level of evidence that mental practice may improve upper extremity motor and ADL performance following stroke. References Barreca S. et al. Management of the Post Stroke Hemiplegic Arm and Hand: Treatment Recommendations of the 2001 Consensus Panel. Heart and Stroke Foundation of Ontario, 2001. Butler AJ, Page SJ. Mental practice with motor imagery: evidence for motor recovery and cortical reorganization after stroke. Arch Phys Med Rehabil 2006; 87:S2-11. Page SJ, Levine P, Leonard A. Mental Practice in Chronic Stroke: Results of a Randomized, Placebo-Controlled Trial. Stroke 2007; 38:1293-1297. Page SJ, Levine P, Sisto SA, Johnston MV. Mental practice combined with physical practice for upper-limb motor deficit in subacute stroke. Physical Therapy 2001; 81(8):1455-62. Teasell RW, Foley NC, Salter K, Bhogal SK, Jutai J, Speechley MR. Evidence-Based Review of Stroke Rehabilitation (11th edition). Canadian Stroke Network; 2008. Zimmermann-Schlatter A, Schuster C, Puhan MA, Siekierka E, Steurer J. Efficacy of motor imagery in post-stroke rehabilitation: a systematic review. J Neuroeng Rehabil 2008; 5:8.

Page 100: C. Motor Recovery pictures removed - EBRSRPR).pdf · - 3 - C1. Assessment of Motor Function Post Stroke C1.1 Assessments of Motor Function Post Stroke Case Study A 65 year old male

- 100 - 100

References Ada L, Dean CM, Lindley R, Lloyd G. Improving community ambulation after stroke: the AMBULATE trial. BMC Neurology 2009; 9(8). Alexander MP. Stroke rehabilitation outcome: A potential use of predictive variables to establish levels of care. Stroke 1994; 25(1):128-134. Allison R, Dennett R. Pilot randomized controlled trial to assess the impact of additional supported standing practice on functional ability post stroke. Clin Rehabil 2007; 21:614-619. Bagg S, Pombo AP, Hopman W. Effect of age on functional outcomes after stroke rehabilitation. Stroke 2002; 33(1):179-185. Bakheit AM, Thilmann AF, Ward AB, Poewe W, Wissel J, Muller J, Benecke R, Collin C, Muller F, Ward CD, Neumann C. A randomized, double-blind, placebo-controlled, dose-ranging study to compare the efficacy and safety of three doses of botulinum toxin type A (Dysport) with placebo in upper limb spasticity after stroke. Stroke. 2000 Oct;31(10):2402-6.

Bakheit AM, Pittock S, Moore AP, Wurker M, Otto S, Erbguth F, Coxon L. A randomized, double-blind, placebo-controlled study of the efficacy and safety of botulinum toxin type A in upper limb spasticity in patients with stroke. Eur J Neurol. 2001;8(6):559-65. Barbeau H, Visintin M. Optimal outcomes obtained with body-weight support combined with treadmill training in stroke subjects. Arch Phys Med Rehabil 2003; 84:1458-65. Barclay-Goddard R. Physical function outcome measurement in acute neurology. Physiotherapy Canada 2000;52:138-145. Barreca S. et al. Management of the Post Stroke Hemiplegic Arm and Hand: Treatment Recommendations of the 2001 Consensus Panel. Heart and Stroke Foundation of Ontario, 2001. Barreca S, Wolf S L, Fasoli S, Bohannon R. Treatment interventions for the paretic upper limb of stroke survivors: a critical review. Neurorehabil Neural Repair 2003; 17(4):220-226. Berg KO, Wood-Dauphinee S, Williams JL, Maki B. Measuring balance in the elderly: preliminary development of an instrument. Physiotherapy Canada 1989; 41:304-311. Berg KO, Wood-Dauphinee S, Williams JL. The Balance Scale: Reliability assessment with elderly residents and patients with acute stroke. Scan J Rehab Med 1995; 27:27-36. Berg KO, Wood-Dauphinee S, Williams JL, Maki B. Measuring balance in the elderly: validation of an instrument. Can J Public Health 1992; 83:S7-S11. Bender L, Mckenna K. Hemiplegic shoulder pain: Defining the problem and its management. Disabil Rehabil 2001;23(16):698-705. Bobath B. Hemiplegia: evaluation and treatment. London: Butterworth-Heinemann, 1978.

Page 101: C. Motor Recovery pictures removed - EBRSRPR).pdf · - 3 - C1. Assessment of Motor Function Post Stroke C1.1 Assessments of Motor Function Post Stroke Case Study A 65 year old male

- 101 - 101

Bobath, B. (Ed.). Adult Hemiplegia: evaluation and treatment. London: Heinemann Medical Books; 1990. Bogataj U, Gros N, Kljajic M, Acimovic R, Malezic M. The rehabilitation of gait in patients with hemiplegia: a comparison between conventional therapy and multichannel functional electrical stimulation therapy. Phys Ther 1995; 75:490-502. Bogousslavsky J, Pierre P. Ischemic stroke in patients under age 45. Neurol Clin 1992; 10(1):113-124. Bonaiuti D, Rebasti L, Sioli P. The constraint induced movement therapy: a systematic review of randomised controlled trials on the adult stroke patients. Eura Medicophys 2007; 43:139-146. Bonan IV, Yelnik AP, Colle FM et al. Reliance on visal information after stroke. Part II: Effectiveness of a balance rehabilitation program with visual cue deprivation after stroke: A randomized controlled trial. Arch Phys Med Rehabil 2004; 85:274-278. Bourbonnais D, Bilodeau S, Lepage Y, Beaudoin N, Gravel D, Forget R. Effect of force-feedback treatments in patients with chronic motor deficits after a stroke. Am J Phys Med Rehabil 2002; 81: 890-897. Bowman BR, Baker LL, Waters RL. Positional feedback and electrical stimulation: an automated treatment for the hemiplegic wrist. Arch Phys Med Rehabil 1979; 60:497–502. Braun SM, Beurskens AJ, Borm PJ, Schack T, Wade DT. The effects of mental practice in stroke rehabilitation: a systematic review. Archives of Physical Medicine & Rehabilitation. 2006; 87(6):842-852. Brown AM, Marlowe KJ, Bjelke B. Age effect on motor recovery in a post-acute animal stroke model. Neurobiol Aging 2003; 24(4):607-614. Brunnstrom S. Movement Therapy in Hemiplegia. New York: Harper & Row; 1970. Burbaud P, Wiart L, Dubos JL, Gaujard E, Debelleix S, Joseph PA, Mazaux JM, Bioulac B, Marat M, Lagueny A. A randomised, double blind placebo controlled trial of botulin toxin in the treatment of spastic foot in hemiparetic patients. J Neurology, Neurosurgery and Psychiatry 1996;61:265-269. Burridge JH, Taylor PN, Hagan SA, Wood DE, Swain ID. The effects of common peroneal stimulation on the effort and speed of walking: a randomized controlled trial with chronic hemiplegic patients. Clin Rehabil 1997; 11:201-210. Carr EK, Kenney FD. Positioning of the stroke patient: a review of the literature. Int J Nurs Stud 1992;29(4):355-369. Carr JH, Shepherd RB, Nordholm L, Lynne D. Investigation of a new motor assessment scale for stroke patients. Physical Therapy 1985; 65: 175-80.

Page 102: C. Motor Recovery pictures removed - EBRSRPR).pdf · - 3 - C1. Assessment of Motor Function Post Stroke C1.1 Assessments of Motor Function Post Stroke Case Study A 65 year old male

- 102 - 102

Chen SC, Chen YL, Chen CJ, Lai CH, Chiang WH, Chen WL. Effects of surface electrical stimulation on the muscle-tendon junction of spastic gastrocnemius in stroke patients. Disabil Rehabil 2005; 27(3):105-110. Cheng PT, Wu SH, Liaw MY, Wong AM, Tang FT. Symmetrical body-weight distribution training in stroke patients and its effect on fall prevention. Arch Phys Med Rehabil 2001; 82:1650-1654. Church C, Price C, Pandyan AD, Huntley S, Curless R, Rodgers H. Randomized Controlled Trial to Evaluate the Effect of Surface Neuromuscular Electrical Stimulation to the Shoulder After Acute Stroke. Stroke 2006;37:2995-3001. Classen J, Liepert J, Wise SP, Hallett M, Cohen LG. Rapid plasticity of human cortical movement representation induced by practice. J Neurophysiol 1998;79(2):1117-23. Cotman CW, Anderson KJ. Synaptic plasticity and functional stabilization in the hippocampal formation: Possible role in Alzheimer’s disease. Adv Neurol 1988; 47:313-335. Cozean CD, Pease WS, Hubbell SL. Biofeedback and functional electric stimulation in stroke rehabilitation. Arch Phys Med Rehabil 1988; 69:401-405. Cramer SC. The EXCITE trial: a major step forward for restorative therapies in stroke. Stroke 2007; 38:2204-2205. da Cunha, I. T., Jr., Lim, P. A., Qureshy, H., Henson, H., Monga, T., & Protas, E. J. Gait outcomes after acute stroke rehabilitation with supported treadmill ambulation training: a randomized controlled pilot study. Arch Phys Med Rehabil 2002; 83(9):1258-1265. Daly JJ, Roenigk K, Holcomb J, Rogers JM, Butler K, Gansen J, McCabe J, Frederickson E, Marsolais EB, Ruff RL. A randomized controlled trial of functional Neuromuscular Stimulation in chronic stroke subjects. Stroke 2006; 37:172-178. Daly JJ, Roenigk KL, Butler KM, et al. Response of sagittal plane gait kinematics to weight supported treadmill training and functional neuromuscular stimulation following stroke. J Rehabil Res Dev 2004;41:807-820. Dean CM, Richards CL, Malouin F. Task-related circuit training improves performance of locomotor tasks in chronic stroke: a randomized, controlled pilot trial. Arch Phys Med Rehabil 2000; 81:409-417. De Boer KS, Arwert HJ, de Groot JH, Meskers CG, Mishre AD, Arendzen JH. Shoulder pain and external rotation in spastic hemiplegia do not improve by injection of botulinum toxin A into the subscapular muscle. J Neurol Neurosurg Psychiatry 2008;79:581-583. Deltombe T, De Wispelaere JF, Gustin T, Jamart J, Hanson P. Selective blocks of the motor nerve branches to the soleus and tibialis posterior muscles in the management of the spastic equinovarus foot. Arch Phys Med Rehabil 2004; 85(1):54-8. De Seze M, Wiart L, Bon-Saint-Come A, Debelleix S, de Seze M, Joseph PA, Mazaux JM, Barat M. Rehabilitation of postural disturbances of hemiplegic patients by using trunk control retraining during exploratory exercises. Arch Phys Med Rehabil 2001; 82:793-800.

Page 103: C. Motor Recovery pictures removed - EBRSRPR).pdf · - 3 - C1. Assessment of Motor Function Post Stroke C1.1 Assessments of Motor Function Post Stroke Case Study A 65 year old male

- 103 - 103

Dias D, Lains J, Pereira A, et al. Can we improve gait skills in chronic hemiplegics? A randomised control trial with gait trainer. Eura Medicophys 2007; 43:499-504. Dijkerman HC, Letswaart M, Johnston M, MacWalter RS. Does motor imagery training improve hand function in chronic stroke patients? A pilot study. Clinical Rehabilitation. 2004; 18(5):538-549. Dromerick AW, Edwards DF, Hahn M. Does the application of constraint-induced movement therapy during acute rehabilitation reduce arm impairment after ischemic stroke? Stroke 2000; 31:2984-2988. Duncan P, Richards L, Wallace D, et al. A randomized, controlled pilot study of a home-based exercise program for individuals with mild and moderate stroke. Stroke 1998; 29:2055-2060. Duncan PW, Sullivan KJ, Behrman AL, et al. Protocol for the Locomotor Experience Applied Post-stroke (LEAPS) trial: a randomized controlled trial. BMC Neurol 2007; 7:39. Eich HJ, Mach H, Werner C, Hesse S. Aerobic treadmill plus Bobath walking training improves walking in subacute stroke: a randomized controlled trial. Clin Rehabil 2004;18:640-651. Eser F, Yavuzer G, Karakus D, Karaoglan B. The effect of balance training on motor recovery and ambulation after stroke: a randomized controlled trial. Eur J Phys Rehabil Med 2008; 44:19-25. Finch E, Brooks D, Stratford PW, Mayo NE. Physical Rehabilitations Outcome Measures. A Guide to Enhanced Clinical Decision-Making., 2nd ed. Toronto, Ontario: Canadian Physiotherapy Association, 2002. Flansbjer UB, Miller M, Downham D, Lexell J. Progressive resistance training after stroke: effects on muscle strength, muscle tone, gait performance and perceived participation. J Rehabil Med 2008; 40:42-48. Francis HP, Wade DT, Turner-Stokes L, Kingswell RS, Dott CS, Coxon EA. Does reducing spasticity translate into functional benefit? An exploratory meta-analysis. J Neurol Neurosurg Psychiatry 2004; 75(11):1547-1551. French B, Thomas LH, Leathley MJ, et al. Repetitive task training for improving functional ability after stroke. Cochrane Database Syst Rev 2007;CD006073.

Fritz SL, Light KE, Patterson TS, Behrman AL, Davis SB. Active finger extension predicts outcomes after constraint-induced movement therapy for individuals with hemiparesis after stroke. Stroke 2005;36:1172-1177. Frost SB, Barbay S, Griel KM, et al. Reorganization of remote corical regions after ishchemic brain injury: a potential substrate for stroke recovery. J Neurophysiol 2003; 89(6):3205-3214. Glanz M, Klawansky S, Stason W, Berkey C, Chalmers TC. Functional electrostimulation in poststroke rehabilitation: a meta-analysis of the randomized controlled trials. Arch Phys Med Rehabil 1996; 77:549-553.

Page 104: C. Motor Recovery pictures removed - EBRSRPR).pdf · - 3 - C1. Assessment of Motor Function Post Stroke C1.1 Assessments of Motor Function Post Stroke Case Study A 65 year old male

- 104 - 104

Glasser. Effects of isokinetic training on the rate of movement during ambulation in hemiparetic patients. Physical Therapy 1986; 66(5):673-676. Gowland C, Stratford P, Ward M, Moreland J, Torresin W, Van Hullenaar S, Sanford J, Barreca S, Vanspall B, Plews N. Measuring physical impairment and disability with the Chedoke-McMaster Stroke Assessment. Stroke 1993; 24(1):58-63. Gracies JM, Singer BJ, Dunne JW. The role of botulinum toxin injections in the management of muscle overactivity of the lower limb. Disability and Rehabilitation 2007; 29: 1789-805. Grant T, Brouwer B, Culham E, Vandervoort A. Balance retraining following acute stroke: a comparison of two methods. Canadian Journal of Rehabilitation 1997; 11:69-73. Grotta JC, Noser EA, Ro T, et al. Constraintinduced movement therapy. Stroke 2004; 35:2699-2701. Hafsteinsdottir TB, Kappelle J, Grypdonck MH, Algra A. Effects of Bobath-based therapy on depression, shoulder pain and health-related quality of life in patients after stroke. J Rehabil Med 2007;39:627-632. Hakkenes S, Keating JL. Constraint-Induced movement therapy following stroke systematic review of randomised controlled trials. Australian Journal of Physiotherapy 2005; 51:221-231. Heckmann J, Mokrusch T, Krockel A, Warnke S, von Stockert T, Neundorfer B. EMGtriggered electrical muscle stimulation in the treatment of central hemiparesis after a stroke. Eur J Phys Med Rehabil 1997; 7(5):138-141. Herman JM, Culpepper L, Franks P. Patterns of utilization, disposition, and length of stay among stroke patients in a community hospital setting. J AM Geriatr Soc 1984; 32(6):421-426. Hesse S, Werner C, von Frankenberg S, Bardelben A. Treadmill training with partial body weight support after stroke. Physical Medicine Rehabilitation Clinic North America 2003; 14:S111-S123. Howe TE, Taylor I, Finn P, Jones H. Lateral weight transference exercises following acute stroke: a preliminary study of clinical effectiveness. Clin Rehabil 2005; 19:45-53. Huijbregts M. The physiotherapy clinical outcome variables (COVS) reliability testing videotape. Physiotherapy Canada 1996;48:285. Inaba M, Edberg E, Montgomery J, Gillis MK. Effectiveness of functional training, active exercise and resistive exercise for patients with hemiplegia. Physical Therapy 1973; 53:28-35. Jorgensen HS, Nakayama H, Raaschou HO, Olsen TS. Recovery of walking function in stroke patients: The Copenhagen Stroke Study. Arch Phys Med Rehabil 1995(a); 76(1):27-32. Jorgensen HS, Nakayama H, Raaschou HO, et al. Outcome and time course of recovery in stroke. Part I: Outcome. The Copenhagen Stroke Study. Arch Phys Med Rehabil 1995(b); 76(1):27-32.

Page 105: C. Motor Recovery pictures removed - EBRSRPR).pdf · - 3 - C1. Assessment of Motor Function Post Stroke C1.1 Assessments of Motor Function Post Stroke Case Study A 65 year old male

- 105 - 105

Jorgensen HS, Nakayama H, Raaschou HO, et al. Outcome and time course of recovery in stroke. Part II: Time course of recovery. The Copenhagen Stroke Study. Arch Phys Med Rehabil 1995(c); 76(5):406-412. Juneja G, Czyrny JJ, Linn RT. Admission balance and outcomes of patients admitted for acute inpatient rehabilitation. Am J Phys Med Rehabil 1998; 77:388-393. Kalra L. Does age affect benefits of stroke unit rehabilitation? Stroke 1994; 25(2):346-351. Kim CM, Eng JJ, MacIntyre DL, Dawson AS. Effects of isokinetic strength training on walking in persons with stroke: a double-blind controlled pilot study. Journal of Stroke and Cerebrovascular Disease 2001; 10(6):265-273. Kong KH, Neo JJ, Chua KS. A randomized controlled study of botulinum toxin A in the treatment of hemiplegic shoulder pain associated with spasticity. Clin Rehabil. 2007;21:28-35. Kosak MC, Reding MJ. Comparison of partial body weight-supported treadmill gait training versus aggressive bracing assisted walking post stroke. Neurorehabil Neural Repair 2000; 14:13-19. Kotila M, Waltimo O, Niemi ML, et al. The profile of recovery from stroke and factors influencing outcome. Stroke 1984; 15(6):1039-1044. Kottink AI, Hermens HJ, Nene AV et al. Therapeutic effect of an implantable peroneal nerve stimulator in subjects with chronic stroke and footdrop: a randomized controlled trial. Physical Therapy 2008; 88(4):437-448. Kottink AI, Hermens HJ, Nene AV, et al. A randomized controlled trial of an implantable 2-channel peroneal nerve stimulator on walking speed and activity in poststroke hemiplegia. Arch Phys Med Rehabil 2007; 88:971-978. Kottink AI, Oostendorp LJ, Buurke JH, Nene AV, Hermens HJ, IJzerman MJ. The orthotic effect of functional electrical stimulation on the improvement of walking in stroke patients with a dropped foot: a systematic review. Artif Organs 2004; 28:577-586. Kugler C, Altenhoner T, Lochner P, Ferbert A. Does age influence early recovery from ischemic stroke? A study from the Hessian Stroke Data Bank. J Neurol 2003; 250(6):676-681. Kumar R, Metter EJ, Mehta AJ, Chew T. Shoulder pain in hemiplegia. The role of exercise. American J Phys Med Rehabil 1990; 69(4):205-208.

Langhammer B, Stanghelle JK. Bobath or motor relearning programme? A comparison of two different approaches of physiotherapy in stroke rehabilitation: a randomized controlled study. Clin Rehabil 2000;14(4):361-9. Levin MF, Hui-Chan CW. Relief of hemiparetic spasticity by TENS is associated with improvement in reflex and voluntary motor functions. Electroencephalogr Clin Neurophysiol 1992; 85:131-142.

Page 106: C. Motor Recovery pictures removed - EBRSRPR).pdf · - 3 - C1. Assessment of Motor Function Post Stroke C1.1 Assessments of Motor Function Post Stroke Case Study A 65 year old male

- 106 - 106

Lin KC, Wu CY, Wei TH, Lee CY, Liu JS. Effects of modified constraint-induced movement therapy on reach-to-grasp movements and functional performance after chronic stroke: a randomized controlled study. Clin Rehabil 2007; 21:1075-1086. Liu K.P., Chan C.C., Lee T.M., Hui-Chan C.W. et al. Mental imagery for promoting relearning for people after stroke: A Randomized Controlled Trial. Arch Phys Med Rehabil 2004; 85(9);1403-1408. Low Choy N, Kuys S, Richards M, Isles R. Measurement of functional ability following traumatic brain injury using the Clinical Outcomes Variable Scale: A reliability study. Australian Journal of Physiotherapy 2002;48:35-39. MacDonell RA, Triggs WJ, Leikauskas J, Bourque M, Robb K, Day BJ, Shahani BT. Functional electrical stimulation to the affected lower limb and recovery after cerebral infarction. Cerebrovasc Dis 1994; 4:155-160. Mao HF, Hsueh IP, Tang PF, Sheu CF, Hsieh CL. Analysis and comparison of the psychometric properties of three balance measures for stroke patients. Stroke 2002; 33:1022-1027. Marini C, Totaro R, De Santis F, Ciancarelli I, Baldassarre M, Carolei A. Stroke in Young Adults in the Community-Based L’Aquila Registry: Incidence and Prognosis. Stroke 2001;32:52-56. Mead GE, Greig CA, Cunningham I, et al. Stroke: a randomized trial of exercise or relaxation. J Am Geriatr Soc 2007; 55:892-899. Merletti R, Zelaschi F, Latella D, et al. Control study of muscle force recovery in hemiparetic patients during treatment with functional electrical-stimulation. Scandinavian Journal of Rehabilitation Medicine 1978; 10(3):147-154. Moreland JD, Goldsmith CH, Huijbregts MP, Anderson RE, Prentice DM, Brunton KB, O’Brien A, Torresin WD. Progressive resistance strengthening exercises after stroke: a single-blind randomized controlled trial. Arch Phys Med Rehabil 2003; 84:1433-40. Morioka S, Yagi F. Effects of perceptual learning exercises on standing balance using a hardness discrimination task in hemiplegic patients following stroke: a randomized controlled pilot trial. Clinical Rehabilitation 2003; 17:600-607. Moseley AM, Stark A, Cameron ID, Pollock A. Treadmill training and body weight support for walking after stroke. Stroke 2003; 34: 3006. Mudie MH. Training symmetry of weight distribuation after stroke: a randomized controlled pilot study comparing task-related reach, Bobath and feedback training approaches. Clinical Rehabilitation 2002;16:582-592. Myers BJ. Proprioceptive neuromuscular facilitation approach. In Trombly CA, ed. (Ed.), Occupational Therapy for Physical Dysfunction: 474-498. Baltimore, MD: Williams & Wilkins; 1995.

Page 107: C. Motor Recovery pictures removed - EBRSRPR).pdf · - 3 - C1. Assessment of Motor Function Post Stroke C1.1 Assessments of Motor Function Post Stroke Case Study A 65 year old male

- 107 - 107

Nadeau SE, Behrman AL, Davis SE, Reid K, Wu SS, Stidham RN, Helms KM, Gonzalez LJ. Donepezil as an adjuvant to constraintinduced therapy for upper-limb dysfunction after stroke: an exploratory randomized clinical trial. 2004; 41:525-534. Nakayama H, Jorgensen HS, Raaschou HO, Olsen TS. Recovery of upper extremity function in stroke patients: the Copenhagen Stroke Study. Arch Phys Med Rehabil 1994;75:394-398. Nedeltchev K, der Maur TA, Georgiadis D, et al. Ischaemic stroke in young adults: predictors of outcome and recurrence. J Neurol Neurosurg Psychiatry 2005;76:191-195. Newsam CJ, Baker LL. Effect of an electric stimulation facilitation program on quadriceps motor unit recruitment after stroke. Arch Phys Med Rehabil 2004; 85:2040-2045. Nisson L, Carlsson J, Danielsson A, Fugl-Meyer A, Hellstrom K, Kristensen L, Sjolund B, Sunnerhagen K, Grimby G. Walking training of patients with hemiparesis at an early stage on a treadmill with body weight support and walking training on the group. Clinical Rehabilitation 2001; 15:515-527. Nudo RJ. Adaptice plasticity in motor cortex: implication for rehabilitation after bain injury. J Rehabil Med 2003; (41Suppl):7-10. Ouellette MM, LeBrasseur NK, Bean JF, et al. High-intensity resistance training improves muscle strength, self-reported function, and disability in long-term stroke survivors. Stroke 2004; 35:1404-1409. Paci M. Physiotherapy based on the Bobath concept for adults with post-stroke hemiplegia: a review of effectiveness studies. J Rehabil Med 2003;35:2-7.

Paci M, Nannetti L, Rinaldi LA. Glenohumeral subluxation in hemiplegia: An overview. J Rehabil Res Dev 2005; 42(4):557-568. Page S.J. Imagery improves upper extremity motor function in chronic stroke patients: A pilot study. The Occupational Therapy Journal of Research 2000; 20(3):200-213. Page SJ, Levine P. Back from the brink: electromyography-triggered stimulation combined with constraint-induced movement therapy in chronic stroke. Arch Phys Med Rehabil 2006; 87:27-31. Page SJ, Levine P, Leonard AC. Modified constraint-induced therapy in acute stroke: a randomized controlled pilot study. Neurorehabil Neural Repair 2005; 19:27-32. Page SJ, Levine P, Leonard A. Mental Practice in Chronic Stroke: Results of a Randomized, Placebo-Controlled Trial. Stroke 2007;38:1293-1297. Page J.S., Levine P., Sisto S., & Johnston M.V. Mental practice combined with physical practice for upper-limb motor deficit in sub-acute stroke. Physical Therapy 2001; 81(8):1455-1462. Page SJ, Levine P, Teepen J, Hartman EC. Resistance-based, reciprocal upper and lower limb locomotor training in chronic stroke: a randomized, controlled crossover study. Clin Rehabil 2008; 22:610-617.

Page 108: C. Motor Recovery pictures removed - EBRSRPR).pdf · - 3 - C1. Assessment of Motor Function Post Stroke C1.1 Assessments of Motor Function Post Stroke Case Study A 65 year old male

- 108 - 108

Page SJ, Sisto S, Johnston MV, Levine P. Modified constraint-induced therapy after subacute stroke: a preliminary study. Neurorehabil Neural Repair 2002; 16(3):290- 295. Page SJ, Sisto S, Levine P, McGrath RE. Efficacy of modified constraint-induced movement therapy in chronic stroke: a single-blinded randomized controlled trial. Arch Phys Med Rehabil. 2004; 85(1):14-8. Pittock SJ, Moore AP, Hardiman O, Ehler E, Kovac M, Bojakowski J, al Khawaja I, Brozman M, Kanovsky P, Skorometz A, Slawek J, Reichel G, Stenner A, timberbaeva S, Stelmasiak Z, Zifko UA, Bhakta B, Coxon E. A double-blind randomised placebo-controlled evaluation of three doses of botulinum toxin type A (Dysport®) in the treatment of spastic equinovarus deformity after stroke. Cerebrovasc dis 2003;15:289-30. Platz T, Eickhof C, van Kaick S, et al. Impairment-oriented training or Bobath therapy for severe arm paresis after stroke: a single-blind, multicentre randomized controlled trial. Clin Rehabil 2005;19:714- 724. Ploughman M, Corbett D. Can forced-use therapy be clinically applied after stroke? An exploratory randomized controlled trial. Arch Phys Med Rehabil. 2004; 85(9):1417-23. Pohl M, Mehrholz J, Ritschel C, Ruchriem S. Speed-dependent treadmill training in ambulatory hemiparetic stroke patients. A randomized controlled trial. Stroke 2002; 33:553-558. Pohl PS, Perera S, Duncan PW, Maletsky R, Whitman R, Studenski S. Gains in distance walking in a 3-month follow-up poststroke: what changes? Neurorehabil Neural Repair 2004;18:30-36. Popa-Wagner A, Schroder E, Schmoll H, et al. Upregulation of MAP1B and MAP2 in the rat brain after middle cerebral artery occlusion: Effect of age. J Cereb Blood Flow Metab 1999; 19(4):425-434. Price SJ, Reding MJ. Physical therapy philosophies and strategies. In Good DC, Couch JR Jr. (Ed.), Handbook of Neurorehabilitation (pp. 181-196). New York: Marcel Dekker, 1994. Richards L, Rothi LJG, Davis S, Wu AA, Nadeau SE. Limited dose response to Constraint-Induced Movement therapy in patients with chronic stroke. Clin Rehabil 2006; 20:1066- 1074. Robbins SM, Houghton PE, Woodbury MG, Brown JL. The therapeutic effect of functional and transcutaneous electric stimulation on improving gait speed in stroke patients: a meta-analysis. Arch Phys Med Rehabil 2006; 87(6):853-859. Sackley CM, Lincoln NB. Single blind randomized controlled trial of visual feedback after stroke: effects on stance symmetry and function. Disabil Rehabil 1997; 19:536-546. Salbach NM, Mayo NE, Wood-Dauphinee S, Hanley JA, Richards CL, Cote R. A task orientated intervention enhances walking distance and speed in the first year post 9. Mobility and the Lower Extremity pg. 107 of 110 www.ebrsr.com stroke: a randomized controlled trial. Clin Rehabil 2004;18:509-519.

Page 109: C. Motor Recovery pictures removed - EBRSRPR).pdf · - 3 - C1. Assessment of Motor Function Post Stroke C1.1 Assessments of Motor Function Post Stroke Case Study A 65 year old male

- 109 - 109

Schauwecker PE, Cheng HW, Serquinia RM, et al. Lesion-induced sprouting of commissural/associational axons and induction of GAP-43 mRNA in hilar and CA3 pyramidal neurons in the hippocampus are diminished in aged rats. J Neurosci 1995; 15(3 Pt 2):2462-2470. Scheff SW, Bernardo LS, Cotman CW. Decrease in adrenergic axon sprouting in the senescent rat. Science 1978; 202(4369):775-778. Seaby L, Torrance G. Reliability of a physiotherapy functional assessment used in a rehabilitation setting. Physiotherapy Canada 1989;41:264-271. Shapira S, Sapir M, Wengier A, et al. Aging has a complex effect on a rat model of ischemic stroke. Brain Res 2002; 925(2):148-158. Sheffler LR, Hennessey MT, Naples GG, Chae J. Peroneal nerve stimulation versus an ankle foot orthosis for correction of footdrop in stroke: impact on functional ambulation. Neurorehabil Neural Repair 2006; 20:355-360. Simpson DM, Alexander DN, O'Brien CF, Tagliati M, Aswad AS, Leon JM, Gibson J, Mordaunt JM, Monaghan EP. Botulinum toxin type A in the treatment of upper extremity spasticity: a randomized, double-blind, placebo-controlled trial. Neurology 1996;46:1306-1310. Skilbeck CE, Wade DT, Hewer RL, Wood VA. Recovery after stroke. J Neurol Neurosurg Psychiatry 1983; 46(1):5-8. Sullivan KJ, Brown DA, Klassen T, et al. Effects of task-specific locomotor and strength training in adults who were ambulatory after stroke: results of the STEPS randomized clinical trial. Phys Ther 2007; 87:1580-1602. Sullivan KJ, Dnowlton BJ, Dobkin BH. Step training with body weight support: effect of treadmill and speed practice paradigms on poststroke locomotor recovery. Arch Phys Med Rehabil 2002;83:683-691. Suputtitada A, Suwanwela NC. The lowest effective dose of botulinum A toxin in adult patients with upper limb spasticity. Disabil Rehabil 2005;27:176-184. Taub E. Somatosensory deafferentation research with monkeys: Implications for rehabilitation medicine. In: LP Ince (Ed.), Behavioral Psychology in Rehabilitation Medicine: Clinical Applications (pp.371-401). Baltimore: Williams and Wilkins, 1980.

Taub E, Uswatte G, Pidikiti R. Constraint-Induced Movement Therapy: a new family of techniques with broad application to physical rehabilitation--a clinical review. J Rehabil Res Dev 1999;36(3):237-51. Taub E, Morris DM. Constraint-induced movement therapy to enhance recovery after stroke. Curr Atheroscler Rep 2001; 3(4):279-86. Taub E, Uswatte G, Morris DM. Improved motor recovery after stroke and massive cortical reorganization following Constraint-Induced Movement therapy. Phys Med Rehabil Clin N Am 2003; 14(1 Suppl):S77-91, ix.

Page 110: C. Motor Recovery pictures removed - EBRSRPR).pdf · - 3 - C1. Assessment of Motor Function Post Stroke C1.1 Assessments of Motor Function Post Stroke Case Study A 65 year old male

- 110 - 110

Taub E, Lum PS, Hardin P, Mark VW, Uswatte G. AutoCITE: automated delivery of CI therapy with reduced effort by therapists. Stroke 2005; 36:1301-1304. Taylor PN, Burridge JH, Dunkerley AL, Wood DE, Norton JA, Singleton C, Swain ID. Clinical use of the Odstock dropped foot stimulator: its effect on the speed and effort of walking. Arch Phys Med Rehabil 1999; 80:1577-1583. Teasell RW, Foley NC, Salter K, Bhogal SK, Jutai J, Speechley MR. Evidence-Based Review of Stroke Rehabilitation (11th edition). Canadian Stroke Network; 2008. www.ebrsr.com Teixeira-Samela LF, Olney SJ, Nadeau S, Brower B. Muscle strengthening and physical conditioning to reduce impairment and disability in chronic stroke survivors. Arch Phys Med Rehabil 1999; 80:1211-8. Tong RK, Ng MF, Li LS. Effectiveness of gait training using an electromechanical gait trainer, with and without functional electric stimulation, in subacute stroke: a randomized controlled trial. Arch Phys Med Rehabil 2006; 87:1298-1304. van der Lee JH, Wagenaar RC, Lankhorst GJ, Vogelaar TW, Deville WL, Bouter LM. Forced use of the upper extremity in chronic stroke patients: results from a single-blind randomized clinical trial. Stroke 1999; 30:2369-2375. van der Lee JH. Constraint-induced therapy for stroke: more of the same or something completely different? Curr Opin Neurol 2001; 14:741-744. van Nes IJ, Latour H, Schils F, Meijer R, van Kuijk A, Geurts AC. Long-term effects of 6-week whole-body vibration on balance recovery and activities of daily living in the postacute phase of stroke: a randomized, controlled trial. Stroke 2006;37:2331-2335. Van Peppen RP, Kwakkel G, Wood-Dauphinee S, Hendriks HJ, Van der Wees PJ, Dekker J. The impact of physical therapy on functional outcomes after stroke: what's the evidence? Clin Rehabil 2004; 18:833-862. van Vliet PM, Lincoln NB, Foxall A. Comparison of Bobath based and movement science based treatment for stroke: a randomised controlled trial. J Neurol Neurosurg Psychiatry 2005;76:503-508. Verbunt JA, Seelen HA, Ramos FP, Michielsen BH, Wetzelaer WL, Moennekens M. Mental practice-based rehabilitation training to improve arm function and daily activity performance in stroke patients: a randomized clinical trial. BMC Neurol 2008;8:7 Visintin M, Barbeau H, Korner-Bitensky N, Mayo NE. A new approach to retrain gait in stroke patients through body weight support and treadmill stimulation. Stroke 1998; 29:1122-1128. Walker C, Brouwer BJ, Culham EG. Use of visual feedback in retraining balance following acute stroke. Phys Ther 2000; 80:886-895. Wee JY, Bagg SD, Palepu A. The Berg balance scale as a predictor of length of stay and discharge destination in an acute stroke rehabilitation setting. Archives of Physical Medicine and Rehabilitation 1999; 80: 448-52.

Page 111: C. Motor Recovery pictures removed - EBRSRPR).pdf · - 3 - C1. Assessment of Motor Function Post Stroke C1.1 Assessments of Motor Function Post Stroke Case Study A 65 year old male

- 111 - 111

Wee JYM, Wong H, Palepu A. Validation of the Berg Balance Scale as a predictor of length of stay and discharge destination in stroke rehabilitation. Arch Phys Med Rehabil 2003; 84:731-735. Werner C, von Frankenberg S, Treig T, Konrad M, Hesse S. Treadmill training with partial body weight support and an electromechanical gait trainer for restoration of gait in subacute stroke patients. A randomized crossover study. Stroke 2002; 33:2895-2901. Whitney SL, Poole JL, Cass SP. A review of balance instruments for older adults. Am J Occup Ther 1998; 52:666-671. Whittemore SR, Nieto-Sampedro M, Needles DL, Cotman CW. Neuronotrophic factors for mammalian brain neurons: Injury induction in neonatal, adult and aged rat brain. Brain Res 1985; 352(2):169-178. Winchester P, Montgomery J, Bowman B, Hislop H. Effects of feedback stimulation training and cyclical electrical stimulation on knee extension in hemiparetic patients. Phys Ther 1983; 63:1096-1103. Wolf SL, Winstein CJ, Miller JP, Taub E, Uswatte G, Morris D, Giuliani C, Light KE, Nichols-Larsen D. Effect of Constraint-Induced Movement Therapy on Upper Extremity Function 3 to 9 months after stroke. JAMA 2006; 296: 2095-2104. Wong AM, Lee MY, Kuo JK, Tang FT. The development and clinical evaluation of a standing biofeedback trainer. J Rehabil Res Dev 1997; 34:322-327. Yagura H, Hatakenaka M, Miyai I. Does therapeutic facilitation add to locomotor outcome of body weight-supported treadmill training in nonambulatory patients with stroke? A randomized controlled trial. Arch Phys Med Rehabil 2006; 87(4):529-535. Yan T, Hui-Chan CWY. Transcutaneous electrical never stimulation (TENS) improves motor recovery and walking ability of subjects with acute stroke: a randomized controlled trial. Stroke 2004; 35(6):E320. Yan T, Hui-Chan CW, Li LS. Functional electrical stimulation improves motor recovery of the lower extremity and walking ability of subjects with first acute stroke: a randomized placebo-controlled trial. Stroke 2005;36:80-85. Yavuzer G, Eser F, Karakus D et al. The effects of balance training on gait late after stroke: a randomized controlled trial. Clin Rehabil. 2006; 20:960-969. Yavuzer G, Geler-Külcü D, Sonel-Tur B, Kutlay S, Ergin S, Stam HJ. Neuromuscular Electric Stimulation Effect on Lower-Extremity Motor Recovery and Gait Kinematics of Patients With Stroke:A Randomized Controlled Trial. Arch Phys Med Rehabil 2006; 87:536-540. Yavuzer G, Oken O, Atay MB, Stam HJ. Effect of sensory-amplitude electric stimulation on motor recovery and gait kinematics after stroke: a randomized controlled study. Arch Phys Med Rehabil 2007; 88:710-714.

Page 112: C. Motor Recovery pictures removed - EBRSRPR).pdf · - 3 - C1. Assessment of Motor Function Post Stroke C1.1 Assessments of Motor Function Post Stroke Case Study A 65 year old male

- 112 - 112

Yelnik AP, Colle FM, Bonan IV, Vicaut E. Treatment of shoulder pain in spastic hemiplegia by reducing spasticity of the subscapular muscle : A randomized, double-blind, placebo-controlled study of botulinum toxin A. J Neurol Neurosurg Psychiatry 2007;78:845-848. Zimmermann-Schlatter A, Schuster C, Puhan MA, Siekierka E, Steurer J. Efficacy of motor imagery in post-stroke rehabilitation: a systematic review. J Neuroeng Rehabil 2008;5:8. Zwick D, Rochelle A, Choksi A, Domowicz J. Evaluation and treatment of balance in the elderly: A review of the efficacy of the Berg Balance Test and Tai Chi Quan. Neurorehabilitation 2000; 15:49-56.